You are on page 1of 140

Federal Income Tax

I. Introduction
4 Main Units:
1. Gross Income
2. Deductions
3. Characterization of Income (Capital Gain or Loss)
4. Computations and Credits
Sources of Tax Law
-The code, the cases, and administrative materials.
Administrative Materials
i. Treasury Regulations
1.
The Secretary of the Treasury is given general authority to prescribe all
needful rules and regulations for the enforcement of the Internal Revenue Code.
This is a lawful delegation of subordinate legislative authority.
2.
Ultimately, the judiciary has the right to determine whether the
regulations promulgated by the executive (the Treasury) conform to the statute
enacted by the legislature.
3.
Most regulations are sustained so long as they reflect a consistent, long
standing interpretation by the Treasury or if they comport with notions of sound
policy and reason.
ii. Revenue Rulings
1.
On application from a taxpayer, the Secretary of the Treasury may
answer a specific question concerning the taxpayers liability.
2.
These responses, dubbed Revenue Rulings, do not carry the force and
effect of regulations, but they do provide precedent for use in the disposition of
similar cases.
3.

The Treasury is not compelled to respond to applications for Rulings.

iii. Private Letter Rulings: Not binding and can be requested but have to pay They are
published.
Tax Court Decision
Secondary Tax Resources: Lexis and Westlaw, BNA Tax Management Portfolios, RIA
Checkpoint, Legal Tax Software

**REMEMBER Regulation and Code are binding law.


*Court gives extreme deference to regulations and are reluctant to say one in no good.
Mayo Foundation v. US
Facts: Social Security requires both employers and employees to pay taxes on wages
earned. There are exemptions for students working for universities as incident to their
course of study. Mayo has residency program for medical students are paid and work
between 50 to 80 hours. Treasury regulation also stated that a services of a full time
employee are not incident to and for the purpose a course of study, and it provided an
example using medical students.
-May sought a refund after the issued its new full time employee regulations. May said
the new regulation was invalid. Dist. Ct. said regulation was inconsistent and invalid. Ct.
of Appeals reversed citing Chevron.
-Court applies the Chevron two-part analysis:
1. Has Congress directly addressed the precise question at issue? No. Statute does
not define student and cant definitively say the statute applies to med students.
2.

Is the rule a reasonable interpretation of the enacted text?


-Mayo wants to use multi factor test from National Muffler case.
National Muffler, on the other hand, requires that reviewing courts take
into account "whether the regulation harmonizes with the plain language
of the statute, its origin, and its purpose" and states that courts may
examine a statutory interpretation made years after the passage of the
original act on the merits

-Court says must apply Chevron.


-Student is ambiguous and the regulation was a reasonable construction of what Congress
has said.
TAKEAWAY: Court gives a lot of deverence and unusual for them to be overturned. They
are as long as they are reasonable.

CHAPTER II. Gross Income


A. GROSS INCOME GNERALLY
Taxable Income- Gross income less certain authorized deductions.
*Alaways read the code first, then go to regulations and administrative materials, case
law, when the code isnt enough. Regulations will have the same number as of the code
section.

Code 61 defines gross income as all income from whatever source derived.
61. Gross Income Defined
(a) General Definiotn.Except as otherwise provided in this subtitle, gross income
means all income from whatever source derived, including (but not limited to)
the following items:
(1) Compensation for services, including fees, commissions, fringe benefits,
and similar items;
(2) Gross income derived from business;
(3) Gains derived from dealings in property;
(4) Interest;
(5) Rents;
(6) Royalties;
(7) Dividends;
(8) Alimony and separate maintenance payments;
(9) Annuities;
(10) Income from life insurance and endowment contracts;
(11) Pensions;
(12) Income from discharge of indebtedness;
(13) Distributive share of partnership gross income;
(14) Income in respect of a decedent; and
(15) Income from an interest in an estate or trust.
**Examples: -Scholarships are mostly not considered gross income. Life insurance
proceeds arent considered gross income.
-Pulitzer Prize is considered gross income.
-10k gratuitous gifts are not gross income.
Reg. 1.61-1 Gross Income.
(a) General Definition. Gross income means all income from whatever source
derived, unless excluded by law. Gross income includes income realized in any
form, whether in money, property, or services. Income may be realized, therefore,
in the form of services, meals, accommodations, stock, or other property, as well
as in cash. Section 61 lists the more common items of gross income for purposes
of illustrationGross income, however, is not limited to the items so
enumerated.
Reg. 1.61-2. Compensation for services, including fees, commissions, and similar
items.
(a) In General
(1) Wages, salaries, commissions paid salesmen, compensation for services on the
basis of a percentage of profits, commissions on insurance premiums, tips,
bonuses including Christmas bonuses), termination or severance pay, rewards,
jury fees, marriage fees and other contributions received by a clergyman for

services, pay of persons in the military or naval forces of the United States,
retired pay of employees, pensions, and retirement of allowances are income
to the recipients unless excluded by law. . . .
Reg. 1.61-2. Compensation for services, including fees, commissions, and similar
items.
(d) Compensation paid other than in cash
(1) In General. Except as otherwise provided in paragraph (d)(6)(i) of this
section (relating to certain property transferred after June 30, 1969), if services are
paid for in property, the fair market value of the property taken in payment must
be included in income as compensation. If services are paid for in exchange for
other services, the fair market value of such other services taken in payment must
be included in income as compensation. If the services are rendered at a stipulated
price, such price will be presumed to be the fair market value of the compensation
received in the absence of evidence to the contrary. . . .
Reg. 1.61-14. Miscellaneous items of gross income.
(a) In General. In addition to the items enumerated in section 61(a), there are many
other kinds of gross income. For example, punitive damages such as treble
damages under the antitrust laws and exemplary damages for fraud are gross
income. Another persons payment of the taxpayers income taxes constitutes
gross income to the taxpayer unless excluded by law. Illegal gains constitute gross
income. Treasure trove, to the extent of its value in Untied States currency,
constitutes gross income for the taxable year in which it is reduced to undisputed
possession.
Cersarini v. US
Facts: The Ceasarinis bought a used piano for $15.00, and they found $4,467 of cash
stashed inside. The filed the cash as income and paid more than $800 in taxes on it. They
later filed an amended return and demanded a refund of that $800+. They argued that
found money should not be taxed as income.
-Ps arguments: 1. The found money is not includable in gross income under Sec 61.
2. Even if it was gross income, it was due and owing the year the piano
was purchased and the SOL are up.
3. If it was gross income it was entitled to capital gains treatment.
-Court said no refund and no capital gains treatment. Rule 61 says gross income means
all income from whatever source derived including but not limited 15 items. Basically, all
income is taxable unless you can point to an exemption and Ps have failed to point to an
exemption.

-P also argues that found money must be construed as a gift. Court shoots it down.
-Ps and Ds miss regulation that that includes treasure trove as gross income.
-Addressing the SOL argument, the court looks to Ohio law to determine when the
money was reduced to undisputed possession. In this case it was reduced only upon
discovery.
CLASS NOTES:
*Everything is income unless otherwise excluded. Regulation 1.61-14 says treasure trove
is included in income. But when does it become taxable? When you discover it and claim
it as your own.
TAKEAWAY: Is treasure trove income? Yes, treasure trove or windfall income is taxable.
Old Colony Trust Co. v. Commissioner
Facts: Company passed resolution that stated the company would pay all the income
taxes for its President and other officers. The Company paid $681,169.88 in taxes for him
for 1918 and $351,179.27 in 1919. The Board of Tax Appeals determined that these
amounts constituted additional income for the years 1919 and 1920.
Issue: If an employer pays the income taxes of an employee, does the payment of the
taxes constitute taxable income?
Held: Yes it is taxable income. The payment of the tax by the employers was in
consideration of the services rendered by the employee and was a gain derived by the
employee from his labor. Doesnt matter that it was paid directly to the government. The
discharge by a third person of an obligation to him is equivalent to receipt by the person
taxed.
-Wasnt a gift either, and cant address the tax on a tax argument because the Treasury
had not yet imposed another tax.
CLASS NOTES:
-Taking the tax burden off of the president is still another form of compensation/ a benefit
in exchange for services. IT IS NOT A GIFT in this situation.
-Look to the context of the payment of someones taxes, if for services then its income
and taxable, if just a gift then its not.

Commissioner v. Glenshaw Glass Co.

Facts: -Two cases were consolidated. First case, The parties settled and Glenshaw
received $800,000. Of that, $324,529.94 represented punitive damages. Glenshaw did not
report that amount as income.
-Second case, William Goldman sued Loews Inc. for violations of antitrust law and
sought treble damages. William Goldman received $375,000 in treble damages but
claimed $250,000 of that represented punitive damages and did not report it as income.
Issue: Did punitive damages constitute taxable income?
Held: Yes. All damages are taxable income. The Court defined income as all undeniable
accessions to wealth, clearly realized, and over which the taxpayers have complete
dominion. The Court rejected the defendants arguments that the income wasnt taxable
because it was not associated with capital or labor. While those designations are typical
sources of income, income is not limited to those avenues. Generally, the Court gives a
liberal construction to the broad phrasing in the code definition, in recognition of the
intention of Congress to tax all gains except those specifically exempted.
CLASS NOTES:
-Punitive damages are taxable income. Loss profit damages are taxable. Scope of 61 is
very broad.
-Default rule: Presume its included unless you can show that it is out.
-Current law explicitly says that punitive damages are taxable.
Charley v. Commissioner
Facts: For company travel, Charley would bill the client for roundtrip, firstclass airfare
then instruct his travel agent to buy a coach ticket. Charley used frequent flyer miles to
upgrade to first class, but he pocketed the difference between the billed price and the
purchase price in a personal travel account. In 1988, this account accrued more than $3k
in travel credit. The tax court levied a deficiency for that amount.
Issue: Whether tax credits converted to cash in a personal travel account established by
an employer constitute gross income to the employee?
Held: Yes. Travel credits were, at base, additional compensation. Alternatively, one
could characterize the transaction as a disposition of property and gross income include
gains derived from dealings is property.
CLASS NOTES:
-Guy was wealthier after the transaction before and still apart of his job. Not explicitly
excluded so still taxable.
-Normal frequent fly miles for ordinary travelers wont be taxed. Consider it more like a
rebate.
*Notes on page 52.

Loans:
*Are loans considered income? Not income because you have to pay it back.
They are offset by an obligation to pay.
Illegal Ascensions to Wealth:
*Are taxable gross income.
Problems pg. 1-2 54
1. Would the results to the taxpayers in the Cesarini case be different if instead of
discovering $4,467 in old currency in the piano, they discovered that the piano, a
Steinway, was the first Steinway piano ever built and is worth $500,000? There is
a difference when you find something is changing value. Wont be taxed on it until
you have a recognition event. (Sold it or something). Dont tax the property when
the value changes.
2. Winner attends the opening of a new department store. All persons attending
are given free raffle tickets for a watch worth $200. Disregarding any possible
application of IRC section 74, must Winner include anything within gross income
when she wins the watch in the raffle? It is taxable as a prize. Prizes and awards
are taxable income. Not a gift.

INCOME WITHOUT RECEIPT OF CASH OR PROPERTY


-Income doesnt have to be money. It can be property or services.
Helvering v. Independent Life Ins. Co.
The IRS was attempting to tax as income the rental value of a building, which was
occupied by its owner. The IRSs theory was that the fair rental value of the building was
income to its owner. (Imputed income = because the company owned the building, they
did not have to pay rent in another building, constituting income.)
Held: Of course this is not income. This is a clear rule. A business cannot be taxed on the
rental value of its own building.
CLASS NOTES:
-Not income to choose not to rent out your own house. Not going to be taxed on forgone
income, dont want to force people to make their property productive.
Revenue Ruling 79-24
- Housepainter paid for legal services by painting his lawyers house. Artist paid his rent
in art.

-Per the IRS, both of these payments amount to income taxable to the lawyer/landlord
at the fair market value.
-The applicable section of the Internal Revenue Code. . . and the Income Tax Regulations
are 61(a) and 1.61-2, relating to compensation for services.

CLASS NOTES:
Barter:
Does Barter create income? Yes and you look to the fair market value. Still as if you paid
money for the trade or service.
*Section 1.61-2(d)(1) of the regulations provides that if services are paid for other than in
money, the fair market value of the property or services taken in payment must be
included in income. If the services were rendered at a stipulated price, such price will be
presumed to be the fair market value of the compensation received in the absence of
evidence to the contrary.
*But what if you do your own work, working for yourself? Not taxable. Can do services
for yourself wont be taxable.
Dean v. Commissioner
-Personal use of business property create income? Yes.
- Deans transferred their home to a closelyheld corporation so that it could serve as
collateral on a loan to the corporation, but they remained in the home rentfree. IRS
sought taxes on the fair market rental value of the home.
-Court of Appeals affirmed the Tax Court ruling that the Deans were liable to the IRS for
taxes on the rental value of the home. The corporation was paying the rent for the Deans.
CLASS NOTES:
-How is it different from living in your own home? It is being owned by the corporation
so youre receiving compensation, free use of corporate property.
Problem 1 pg. 57
Vegy grows vegetables in her garden. Does Vegy have gross income when:
(a)

Vegy harvests her crop? No. Havent received anything.

(b)

Vegy and her family consume $100 worth of vegetables? No. Can enjoy your own
property.

(c)

Vegy sells vegetables for $100? Yes. She has exchanged products for money.

(d)
Vegy exchanges $100 worth of vegetables for $100 worth of tuna ,which Charlie
caught? Yes that is recognition of income for both of them. It was barter.

Chapter 3: The Exclusions of Gifts and Inheritances


A. RULES OF INCLUSION AND EXCLUSION.
Gross income includes the receipt of essentially any financially benefit UNLESS
it is 1) merely a return of capital, 2) offset by the liability of binding obligation to
repay, or 3) specifically excluded.
-Code lists specific items that are included and excluded.
-Provisions in the code to induce behavior. We like people to buy houses, have
kids, and donate to charities.
-How do we define a gift? Different parts of the code of different meanings for
gifts.

B. GIFTS
-Exclude gifts because we assume they have already paid taxes on it. (Just one
reason).
102. Gifts and inheritances
(a) General rule.Gross income does not include the value of property acquired by
gift, bequest, devise, or inheritance.
(b) Income.Subsection (a) shall not exclude from gross income
(1) The income for any property referred to in subsection (a); (Example: Stock is
tax free but the dividends are taxable) or
(2) Where the gift, bequest, devise, or inheritance is of income form property, the
amount of such income. (Example: Given the dividends from stock for next 10
years, is still taxable).
(c) (1)- Gifts and Inheritances. Employee Gifts. In General.Subsection (a) shall
not exclude form gross income any amount transferred by or for an employer to,
or for the benefit of, an employee.

Reg. 1.102-1. Gifts and inheritances.

(a) General Rule. Property received as a gift, or received under a will or under
statutes of descent and distribution, is not includible in gross income, although the
income from such property is includible in gross income. An amount of principal
paid under a marriage settlement is a gift. However, see Section 71 and the
regulations thereunder for rules relating to alimony or allowances paid upon
divorce or separation. Section 102 does not apply to prizes and awards (see
section 74 and 1.74-1) nor to scholarships and fellowship grants (see section
117 and the regulations thereunder)
(b) Income from gifts and inheritances. The income from any property received as
a gift, or under a will or statue of descent and distribution shall not be excluded
form gross income under paragraph (a) of this section.
102- an exclusion- Gross income does not include the value of property acquired by gift,
bequest, devise or inheritance. These are specifically excluded from income.
-Not Excluded:

Income from property received as a gift.


A gift of income from property.

Proposed Reg. 1.102-1(f)


**Look up. Gives some exceptions.
Commissioner v. Duberstein
-Two cases: First case. President of a metal company did business with another
metal company and the other president personally. Duberstein would give the
other president names of potential customers. The other president gave Duberstein
a Cadillac as a present. Duberstein didnt include the value of the car in his taxes.
-Second case: Stanton v. US: Man worked for church and its subsidiary. Directors
passed a resolution that gave Stanton a gratuity in the amount of 22k provided that
the church was released from all right and claims to pension and retirement. There
was hint of some ill feeling between the directors and Stanton, suggesting that
they paid him to retire.
ISSUE: Did these transfers amount to gifts: within the meaning of the statute?
Holding: Gift is not defined on the common law sense. Just because there is not
legal or moral obligation to make a payment does not make it a gift.
-Court says the intention controls when determining whether something was a gift
or not.
-Duberstein: Clear that the car was compensation for the presidents information
despite what they called it or that there was no obligation. Not a gift.

-Whether a transfer of money or property constitutes a gift is an issue of fact to be


determined by the trier of fact.
CLASS NOTES: -Need to know the facts to figure out if it was a gift or not.
-Look for a transfer from a detached and disinterested generosity out of
affection, respect, admiration, charity or like impulses. Look at the intention of
transferor.

Problem pg. 69
2. At the Heads Eye Casino is Vegas, Lucky Louie gives the matre d a $50 tip to
assure a good table, and gives the croupier a toke after a good night with the
cubes. Does either the matre d or the croupier have gross income? Yes.
Intending to give them money in return for something. Tips are not gifts.
BONUS: Professor received an extra copy of a textbook as an unsolicited mailing
from publisher who wanted to recommend the book to students. The professor
gave the book to a student. Is the book income to the student? Was the textbook
income to the professor? The book is not income to the student. It is a gift.
-Yes? Because it was give to the professor hoping he would help them sell
it. However, could argue it has no fair market value.
Employee Gifts
**A transfer from an employer is very rarely a gift
102(c) Denial of exclusion from income for employee gifts*
274(b) Disallowance of deduction for business or profit-making activities for gifts that
exceed $25*

[74(c) Certain employee achievement awards excluded from income


132(e) Certain de minimus employee fringes excluded from income
274(j) Limitations on deductibility of certain employee achievement awards
Prop. Reg. 1.102-1(f) refining scope of 102 with respect to cross-references and
intrafamily transfers
102(c)(1)- Gifts and Inheritances. Employee Gifts. In General.Subsection (a)
shall not exclude form gross income any amount transferred by or for an employer to, or
for the benefit of, an employee.
274(b)- Disallowance of certain entertainment, etc., expenses. Gifts.

**Disallowance of deduction for business or profit making activities for gifts that exceed
$25.
(1) Limitations.No deductions shall be allowed under section 162 or section 212
for any expense for gifts made directly or indirectly to any individual to the extent
that such expense, when added to prior expenses of the taxpayer for gifts made to
such individual during the same taxable year, exceeds $25. For purposes of this
section, the term gift means any item excludable form gross income of the
recipient under section 102 which is not excludable from his gross income under
any other provision of this chapter, but such term does not include
(A) An item having a cost to the taxpayer not in excess of $4.00 on which the
name of the taxpayer is clearly and permanently imprinted and which is one
of a number of identical items distributed generally by the taxpayer, or
(B) A sign, display rack, or other promotional material to be used on the
business premises of the recipient.
(2) Special rules.
(A) In the case of a gift by a partnership, the limitation contained in paragraph (1)
shall apply to the partnership as well as to each member thereof.
(B) For purposes of paragraph (1), a husband and wife shall be treated as one
taxpayer.
**In general, 102(a) does not apply to the employer-employee relationship. Any
purported gift between them is taxed.
However, there are exceptions:
1. 74(c), certain Achievement Awards are excluded
2. 132(e), Traditional Retirement Gifts
74(c) Prizes and Awards: Exception for certain employee achievement awards.
(1) In General.Gross income shall not include the value of an employee
achievement award (as defined in section 274(j)) received by the taxpayer if the
cost to the employer of the employee achievement award does not exceed the
amount allowable as a deduction to the employer for the cost of the employee
achievement award.
(2) Excess deduction award.If the cost to the employer of the employee
achievement award received by the taxpayer exceeds the amount allowable as a
deduction to the employer, then gross income includes the greater of
(A) An amount equal to the portion of the cost to the employer of the award that is
not allowable as a deduction to the employer (but in excess of the value of the
award), or
(B) The amount by which the value of the award exceeds the amount allowable as
a deduction to the employer.

132(e). Certain fringe benefits.


De minimis fringe defined.For the purpose of this section
(1) In general.The term de minimis fringe means any property or service the
value of which is (after taking into account the frequency with which similar
fringes are provided by the employer to the employers employees) so small as to
make accounting for its unreasonably or administratively impracticable.
Problems pg. 70
1. Employer gives all of her employees, except her son, a case of wine at
Christmas, worth $120. She gives Son, who is also an employee, a case of
wine, worth $700. Does Son have gross income? First say is it gross
income? Yes. Is there exclusion? Yes 102 as a gift but employer/employee
gift so 102c and income. BUT the exception if they are related, so not
income. Look at the context and relationship.
2. The congregation for whom Reverend serves as a minister gives her a
check for $5,000 on her retirement. Does Reverend have gross income?
Congregation not the employer. So no employer-employee relationship.
Not taxable, so probably a gift.
3. Retiree receives a $5,000 trip on his retirement. To pay for the cost of
the trip, Employer contributed $2,000 and fellow employees of Retiree
contributed $3,000. Does Retiree have gross income? Not a gift from the
employer, considered compensation for past services. Probably a gift from
the employees.

C. BEQUESTS, DEVISES, AND INHERITANCES


102- an exclusion- Gross income does not include the value of property acquired by
gift, bequest, devise or inheritance.

Lyeth v. Hoey- Even if there is a settlement or controversy, the inheritance is free from
tax.
Facts: Woman died, and Petitioner is her grandson. She left four children as heirs;
petitioner and his brother are children of a deceased daughter. Deceased left more than
$3M in trust to benefit Christian Science. During probate, heirs objected that there was a
lack of testamentary capacity and undue influence. In settlement, Petitioner received $80
in cash and shares of a corporation valued at $141,484. The Commissioner treated this as
income and denied petitioners request for a refund.

Issue: Is the property he received taxable income when it is the result of a compromise
agreement to a will contest?
The United States Supreme Court held that the bequest should not be treated as income,
relying on his status as an heir. The Court rejected the arguments that the value was
taxable because it resulted from a court judgment and that the value was taxable because
it did not correlate to the amount bequeathed to the petitioner in her previous will.
Wolder v. Commissioner
Facts: Wolder, an attorney, entered into an agreement with a wealthy client whereby he
would provide a lifetime of legal services free of charge in return for a bequest of 500
shares of common stock in the clients will. The client ultimately left Wolder more than
$15k in cash and 750 shares of stock. Wolder argues that, because of the source, this is
not income. Tax Court found to the contrary.
Issue: Whether a will bequest that functioned as postponed payment for legal services
was taxable income?
Held: Yes. Applied Duberstein, is it a ona fide gift or a method for paying compensation?
Look at the intent. Woman chose to compensate the attorney through a bequest for his
legal services. Just because it was a bequest does hide the fact the she was intending to
compensate him. Congress intended to tax income comprehensively.
**Just because it is in a will doesnt mean that its gratuitous. Look beyond face of
document. Look to why is it there? Gratuitous, tax-free. If compensation, not tax free.
Problems pg. 77
1. Consider whether it is likely that 102 applies in the following circumstances
a) Father leaves Daughter $20,000 in his will. Inheritance. Tax-free.
b) Father dies intestate and Daughter receives $20,000 worth of real estate as
his heir. Still inheritance, hasnt received income from the gift yet. Tax-free.
c) Father leaves several family members out of his will and Daughter and
others attack the will. As a result of the settlement of the controversy
Daughter receives $20,000. Look at the Lyeth v. Hoey case. Still applies. Taxfree.
d) Father leaves Daughter $20,000 in his will stating that the amount is in
appreciation of Daughter's long and devoted services to him. Still probably
tax-free. In appreciation No real services.
e) Father leaves Daughter $20,000 pursuant to a written agreement under
which Daughter agreed to care for Father in his declining years Formal
agreement. Intended to compensate for services. Not tax-free.

f) Same agreement as in (e) above except that Father died intestate and
Daughter successfully enforced her $20,000 claim under the agreement
against the estate Still taxable.
g) Same as (f) above except that Daughter settles her $20,000 claim for a
$10,000 payment. Same outcome. Taxable income.
h) ) Father appointed Daughter executrix of his estate and Fathers will
provided Daughter was to receive $20,000 for services as executrix. Taxable
as services rendered.
i) Father appointed Daughter executrix of his estate and made a $20,000
bequest to her in lieu of all compensation or commissions to which she would
otherwise be entitled as executrix. Still taxable, because you have to do the
service to receive the payment.

2. Boyfriend who has a mental problem with marriage agrees with Taxpayer
that he will leave her everything at his death in return for her staying with
him without marriage. She does, he doesnt, she sues his estate on a theory
of quantum meruit and settles her claim. Is her settlement excludable under
section 102? No. Still compensation for a service. Theres a contract, not a
gratuitous transfer.
3. If the Wolder case arose today, would 102(c) apply to resolve the issue?
Employer-employee relationship? Probably not.

CHAPTER 4 EMPLOYEE BENEFITS


A. Exclusion for Fringe Benefits
-Section 61(a) specifically includes in gross income compensation for services.
Such compensation may come in the form of property as well as cash, and it can
be indirectly as well as directly paid. Fringe benefits are specifically excluded
however. Their value is conceptually income, however, by statute Congress has
excluded it and allowed the taxpayer not to report it.
-Example) Office coffee.
-At first fringe benefits were not codified, now Congress has passed legislation.
Reasons to clarify: -Want to encourage employers to use fringe benefits to help
their business.-Set forth clear boundaries.-Fear of new practices forming which
could shrink the tax base and put a burden on peoples who sole income is in cash.

-If an employee benefit is not specifically excluded from gross income, its value
must be included in gross income.
132. Certain fringe benefits.
(a) Exclusion from gross income.Gross income shall not include any fringe
benefit which qualifies as a(1) No-additional-cost service,
(2) Qualified employee discount,
(3) Working condition fringe,
(4) De minimis fringe,
(5) Qualified transportation fringe,
(6) Qualified moving expense reimbursement,
(7)Qualified retirement planning services
I.R.C. 132(b), NoAdditionalCost Services
-Service offered in employers line of business to employee, so long as employer occurs
no additional cost in providing the service, are tax-free.
-Same line of business.
-Offered broadly (nondiscrimination standard).
-Not displacing paying customers
Common Examples
1. Hotels often allow employees free use of unused hotel rooms.
2. Airlines often allow employees to fly standby for free.
Same Line of Business: Has to be the same line if business.
Displacement Ruleu-e of such benefits cannot displace nonemployee
customers (i.e. the hotel cant turn away customers and still give the
employee a room for free; the airline cant bump a passenger to allow the
employee to fly free).
Reciprocal Agreementstwo companies in the same line of business can
have written reciprocal agreements to allow employees of both companies to
use the services of the other taxfree as long as theres no substantial cost in
doing so.
Nondiscrimination Rulebenefits must be provided to a wide crosssection
of employees, not just the highly compensated ones.
I.R.C. 132(c), Qualified Employee Discounts
Discounts on Goodsan employee purchasing goods sold by her employer can
exclude a discount up to the employers gross profit percentage. [(aggregate sales
price aggregate sales cost)/aggregate sales cost].
Gross profit percentage shall be determined on the basis of--(i) all property
offered to customers in the ordinary course of the line of business of the employer
in which the employee is performing services (or a reasonable classification of
property

Discounts on Servicesan employee purchasing services sold by her employer


can exclude up to 20% of the market price.
Line of Business Ruleas with the noaddl cost exemption above, the exclusion
for qualified employee discounts is limited to goods or services sold in the line of
business in which the employee is providing services.
NonDiscrimination Ruleas with the no addl cost exemption above, the
discount must be available to a wide crosssection of employees, not just the
highly compensated ones.
I.R.C. 132(d), Working Condition Fringe
- A working condition fringe property or service offered by employer to
employee that would have been deductible by the employee as a business expense
or depreciation. Theoretically a wash, but business deductions have their
limitations, so this does add benefits for many.
- Common Examples
Use of a company car or jet for business purposes; Employers subscription to a
business or trade journal for use by the employee; Bodyguard provided to an
employee for security reasons; Onthejob training provided by an employer.
I.R.C. 132(e), De Minimis Fringe
-A de minimis fringe is any property or service the value of which is so small as
to make accounting for it unreasonable or administratively impracticable.
Frequency is considered to determine if de minimis.
-Frequency considered as to whether benefit is de minimus.
-Detail in Code and Regs as to some specific examples.
- Common Examples
1.Use of secretarial services for personal letters;
2. Occasional personal use of company copy machines/printing facilities;
3. Occasional cocktail parties/picnics;
4. Occasional supper money of taxi fare advanced because of overtime
work;
5. Occasional tickets to sporting events or the theatre;

6. Coffee, donuts, drinks;


7. Local telephone calls;
8. Lowvalue holiday gifts;
9. Traditional retirement gifts presented to an employee upon retirement
after lengthy service.
I.R.C. 132(f), Qualified Transportation Fringe
i.
A qualified transportation fringe includes expenses for
1.
Qualified parking, up to $250/mo
2.
Use of a commuter highway vehicle, up to $130/mo
3.
Transit passes, up to $175/mo
4.
Use of a bicycle, up to $20/mo
Parking must be at or near the business premises of the employer
*Value is capped and indexed for inflation. So it changes year to year.
*Employee can choose transportation benefit or cash; cash is included in income,
benefit is not.
*So if your benefit exceeds the capped limit, the excess is subject to tax.
*If a fringe is within the definition of a qualified transportation fringe, it cant
qualify for a working condition fringe or de minimis fringe.
Qualified Moving Expense Reimbursement
-Benefits offered by the employer related to moving to employment are
excludable under certain circumstances.
132 (g)Certain individuals treated as employees for purposes of subsections (a)(1)
and (2).--For purposes of paragraphs (1) and (2) of subsection (a)-(1) Retired and disabled employees and surviving spouse of employee treated as
employee
(2) Spouse and dependent children
(3) Special rule for parents in the case of air transportation
132 (j) Special rules.-(1) Exclusions under subsection (a)(1) and (2) apply to highly compensated employees
only if no discrimination
(2) Special rule for leased sections of department stores
(3) Auto salesmen

Problem 1 pg. 87
Consider whether and to what extent the fringe benefits listed below may be
excluded from gross income and, where possible, support your conclusions with
statutory authority
a) Hotel Chain: Excluded under No-additional-cost service. Imposes no
substantial additional cost on the employer since there are other empty
rooms; AND is offered for sale in the ordinary course of business in which the
employee works.
b) Paying Guest Kicked Out: Doesnt qualify if you have foregone revenue.
Reg 132.2(a)(2) says that excess capacity services like hotels are treated as no
additional cost services. BUT here there is no excess capacity? BUT I say it is
still excluded assuming there is no substantial cost to the employer for kicking
the guest out.
c) Bill Rebated: Still excluded. Reg 1.132-2(a)(3) says the exclusion also applies
if the benefit is provided through a partial or total cash rebate of an amount
paid for the service.
d) Spouse and kids use room: Still excluded. Discount and no additional cost
fringes extend to spouses and children.
e) Reciprocal Agreement: Excluded. Two companies in the same line of business
can have written reciprocal agreements to allow employees of both companies
to use the services of the other taxfree as long as theres no substantial cost
in doing so.
f) Special Discount For Officers Not Excluded. Violates the non-discrimination
rule. Probably not since they are substantially the same terms.
g) Shipping Worker in Conglomerate: Not excluded. It violates the same line of
business rule. If he worked for both as an accountant or something it would
be excluded.
h) Controller of Conglomerate: Excluded since as a controller for both lines of
business, he performs substantial services that directly benefit both lines of
business.
(iv) Performance of services that directly benefit more than one line of
business (A) In general. An employee who performs substantial services that
directly benefit more than one line of business of an employer is treated as
performing substantial services in all such line of business.

i) Insurance Discount for Insurance Employee: Excluded. Code imposes 20%


ceiling on services.
j) Discounted Furniture for Store Salesman: 1,000,000-600,000/1,000,000=
40% is the gross profit percentage. The employee got a 50% discount, which
exceeds the gross profit percentage. Lowest she could have paid for the sofa
was 1,200 so shell have to pay taxes on the 200 dollar excess.
k) Employee goes to business convention and employer picks up the cost.
Excluded as a working condition fringe.
l) Happy Hour every week: Probably excluded as a de minimis fringe.
m) Case of Christmas Scotch. Not Excludable? Looks like a gift and employee
gift are rarely qualified as non-taxable gifts.
n) Officer of corp. parking is paid for. Non-officers pay their own fees: Looks
like its excluded under a qualified transportation fringe. **Not discriminatory
cause that rule only applies to discount and non-additional cost fringes.
o) Vouchers for $185 of parking: It would go over, so it would be the excess that
is taxable.
p) On-site gym. Excludable.

B. Exclusions for Meals and Lodging


119. Meals or lodging furnished for the convenience of the employer.
(a) Meals and lodging furnished to employee, his spouse, and his dependents,
pursuant to employment.There shall be excluded from gross income of an
employee the value of any meals or lodging furnished to him, his spouse, or any
of his dependents by or on behalf of his employer for the convenience of the
employer, but only if
(1) In the case of meals, the meals are furnished on the business premises of the
employer, or
(2) In the case of lodging, the employee is required to accept such lodging on
the business premises of his employer as a condition of his employment.
Reg 119-1
(a) Meals--(1) In general. The value of meals furnished to an employee by his employer
shall be excluded from the employee's gross income if two tests are met: (i) The meals
are furnished on the business premises of the employer, and (ii) the meals are furnished
for the convenience of the employer. The question of whether meals are furnished for the
convenience of the employer is one of fact to be determined by analysis of all the facts
and circumstances in each case. If the tests described in subdivisions (i) and (ii) of this

subparagraph are met, the exclusion shall apply irrespective of whether under an
employment contract or a statute fixing the terms of employment such meals are
furnished as compensation.
(2) Meals furnished without a charge. (i) Meals furnished by an employer without
charge to the employee will be regarded as furnished for the convenience of the employer
if such meals are furnished for a substantial noncompensatory business reason of the
employer
(b) Lodging. The value of lodging furnished to an employee by the employer shall be
excluded from the employee's gross income if three tests are met:
(1) The lodging is furnished on the business premises of the employer,
(2) The lodging is furnished for the convenience of the employer, and
(3) The employee is required to accept such lodging as a condition of his employment.
(c) Business premises of the employer--(1) In general. For purposes of this section, the
term business premises of the employer generally means the place of employment of
the employee. For example, meals and lodging furnished in the employer's home to a
domestic servant would constitute meals and lodging furnished on the business premises
of the employer. Similarly, meals furnished to cowhands while herding their employer's
cattle on leased land would be regarded as furnished on the business premises of the
employer

Herbert G. Hatt
-Hatt married the president and majority stockholder of a corporation that operated a
funeral home. Pursuant to an antenuptial agreement, he became the majority stockholder,
president, and general manager.
-Hatt moved into the apartment located in funeral home building where he would
answerer telephone calls when the office was closed and meet with customers after hours,
which was customary for funeral homes in the area.
-Government said that Hatt was taxable on the apartments fair rental value.
-Government argued that Hatt wasnt required to accept the lodging as a condition of his
employment since there were people already available 24 hours AND that the lodging
wasnt furnished for the convenience of the employer since he could control what was at
their convenience.
-Court said the market value of the rent was excludable and that the condition and
convenience tests were basically similar.
-Court noted that the funeral business requires someone to be available 24 hours a day
and every other funeral home had someone there 24 hours a day. The ambulance crews

shouldnt be considered a factor since they werent authorized to handle funereal


business.
-Also stated that the former president, his wife, lived in the apartment also.
CLASS NOTES:

107. Rental Value of Parsonages


In the case of a minister of the gospel, gross income does not include
(1) The rental value of a home furnished to him as part of his compensation; or
(2) The rental allowance paid to him as part of his compensation, to the extent used by
him to rent or provide a home and to the extent such allowances does not exceed
the fair rental value of the home, including furnishings and appurtenances such as a
garage, plus the cost of utilities.
Rule: A minister of the gospel may exclude from gross income either the rental value
of a home furnished to her or the housing allowance paid to her as part of her
compensation.

Problems pg. 91-92


1. Employer provides Employee and Spouse and Child a residence on
Employers business premises, having a rental value of $15,000 per year, but
charging Employee only $6,000.
a. What result if the nature of Employees work doesn't require
Employee to live on the premises as a condition of employment? Not
excluded. The employee is required to accept such lodging . . . as a
condition of his employment. All three conditions are not met. So the
9k excess will be considered income
b. What result if Employer and Employee simply agreed to a clause in
the employment contract requiring Employee to live in the residence?
Under 119(b)(1) the clause is not a determinative factor but probably
excluded. Have to prove why.
c. What result if Employees work and contract require Employee to live
on the premises and Employer furnishes Employee and family $6,000
worth of groceries each year? Not excluded. Groceries are too broad
and arent meals. The 6k would be taxable.
d. What result if Employer transferred the residence to Employee in fee
simple in the year that Employee accepted the position and

commenced work? Does the value of the residence constitute


excluded lodging? Not excluded. Cant give someone house because it
wouldnt be the employers premises anymore.
2. Planner incorporated her motel business and the corporation purchased a piece
of residential property adjacent to the motel. The corporation by contract
required Planner to use the residence and also furnished her meals. Planner
worked at the motel and was on call 24 hours a day. May Planner exclude the
value of the residence or the meals or both from her gross income? Probably
both are excluded. She minimized taxes by structuring her business that way.
3. State highway patrolman is required to be on duty from 8 a.m. to 5 p.m. At
noon he eats lunch at various privately owned restaurants, which are adjacent
to the state highway. At the end of each month the state reimburses him for
his luncheon expenses. Are such cash reimbursements included in his gross
income? The Supreme Court in that case said that Section 119 was only to
exclude meals in-kind and not cash reimbursements.

CHAPTER 5 AWARDS

A. PRIZES: With very limited exceptions, prizes and awards are includable in
gross income.
74. Prizes and Awards
(a) General Rule.Except as otherwise provided in this section or in section 117
(relating to qualified scholarships), gross income includes amounts received as
prizes and awards.
(b) Exception for certain prizes and awards transferred to charities.
Gross income does not include amounts received as prizes and awards made
primarily in recognition of religious, charitable, scientific, educational, artistic,
literary, or civic achievement, but only if
(1) The recipient was selected without any action on his part to enter the contest
or proceeding;
(2) The recipient is not required to render substantial future services as a
condition to receiving the prize or award; and
(3) The prize or award is transferred by the payor to a governmental unit or
organization described in paragraph (1) or (2) of section 170(c) pursuant to a
designation made by the recipient.
(c) Exception for certain employee achievement awards.

(1) In general.Gross income shall not include the value of an employee


achievement award (as defined in section 274(j)) received by the taxpayer if
the cost to the employer of the employee achievement award does not
exceed the amount allowable as a deduction to the employer for the cost of
the employee achievement award.
(2) Excess deduction award.If the cost to the employer of the employee
achievement award received by the taxpayer exceeds the amount allowable
as a deduction to the employer, then gross income includes the greater of
(A) An amount equal to the portion of the cost to the employer of the
award that is not allowable as a deduction to the employer (but not in
excess of the value of the award), or
(B) The amount by which the value of the award exceeds the amount
allowable as a deduction to the employer.
The remaining portion of the value of such award shall not be
included in the gross income of the recipient.
(3) Treatment of tax-exempt employers.In the case of an employer exempt
from taxation under this subtitle, any reference in this subsection to the
amount allowable as a deduction to the employer shall be treated as a
reference to the amount which would be allowable as a deduction to the
employer if the employer were not exempt from taxation under this subtitle.
Treasury Regulation 1.74-1 Prizes and awards.
(a) Inclusion in gross income.
(1) Section 74(a) requires the inclusion in gross income of all amounts received as
prizes and awards, unless such prizes or awards qualify as an exclusion from
gross income under subsection (b), or unless such prize or award is a
scholarship or fellowship grant excluded from gross income by section 117.
Prizes and awards which are includible in gross income include (but are not
limited to) amounts received from radio and television giveaway shows, door
prizes, and awards in contests of all types, as well as any prizes and awards
from an employer to an employee in recognition of some achievement in
connection with his employment.
(2) If the prize or award is not made in money but is made in goods or services,
the fair market value of the goods or services is the amount to be included in
income.
THE GENERAL RULE: The value of all prizes and awards received must be included
in gross income.

EXCEPTIONS:

Exception #1: Certain Awards Transferred to Charity


A taxpayer may exclude from gross income an award recognizing religious, charitable,
scientific, educational, artistic, literary, or civic achievement only if
1. The recipient was selected without any action on his part to enter the contest;
2. The award is not contingent on future services; AND
3. The award is transferred by the payor directly to a governmental unit or to a
Section 107(c) charity at the designation of the recipient.
Exception #2: Employee Achievement Awards
A taxpayer may exclude from gross income an employee achievement award relating to
term or service or safety. The award must be in the form of tangible personal property, be
awarded as part of a meaningful ceremony, and not be mere disguised compensation.
1. A length of service award does not qualify unless the employee has been in the
employers service for five years or more and has not received a length of service
award for the current year or any of the prior four years.
2. A safety achievement award only qualifies if made to a person other than a
manager, administrator, clerical employee or other professional employee and
only if 10 percent or less of an employers qualified employees receive such
awards during the year.
Allen J. McDonell, Tax Court (1967)- Required Travel Paid by Employer is Excluded
From Gross Income.
Here, McDonell was an administrative official at the Dairy Equipment Company. His
name was drawn in a lottery of administrative officials to supervise a company trip to
Hawaii. McDonell and his wife were required to attend to supervise the nonadministrative employees on their company-sponsored trip. The cost of the trip was
$1,200. The Tax Court held that it was not taxable income, it was really just a business
trip, which McDonell and his wife were required to go on. Furthermore, the Court noted
that the couple neither went shopping nor swimming while in Hawaii, two activities for
which Hawaii is famous. The trip served a business purpose. End of Story.
CLASS NOTES: This was a very fact sensitive inquiry to really figure out what was
going on.

Problems pg. 98
1. Each year national sportswriters get together and select the single most outstanding
amateur athlete in the country and award that person a check for $5,000. Michael, a
talented swimmer, has been selected for this years award. The award is giving with the
stipulation that the winner deliver a 15 minute acceptance speech at an awards banquet.

Michael, essentially delivering an acceptable rejection acceptance speech, designates


the Special Olympics, a charity under 170, to receive the $5,000 award. The
sportswriters send the check to the Special Olympics. Will Michael be able to exclude
the $5,000 from his gross income? Probably yes under the Certain awards transferred
to charity exception. But was it within the specific art of performance covered by the
code? Is the requirement of the speech substantial future services? A taxpayer may
exclude from gross income an award recognizing religious, charitable, scientific,
educational, artistic, literary, or civic achievement only if
1. The recipient was selected without any action on his part to enter the contest;
2. The award is not contingent on future services; AND
3. The award is transferred by the payer directly to a governmental unit or to a
Section 107(c) charity at the designation of the recipient.
*Even if didnt come under an exception, it would probably be deducted as a
charitable contribution.
2. Gusher Oil desired to make its employees feel more appreciated. To implement
this desire, Gusher creates an awards program whereby employees are given awards
for achieving certain lengths of service. In each case, determine the extent to which
employee, Cliff Hanger, is able to exclude the award from gross income.
a.) Cliff has been working for Gusher for 12 years. Announcing Cliffs retirement
at the Oil Barons Ball, Gusher Oil gives him a $300 gift certificate. Cash or cash
equivalent wont work. Considered additional compensation
b.) Cliff (who is continuing to work for Gusher) receives a gold watch worth $300
for his twelve years of service, presented at the Oil Barons Ball. Excluded. Its
tangible property, meaningful ceremony, not disguised compensation. Over five
years and under the 400.
c.) Same as (b) above, except that Cliff has worked for Gusher for only four years.
Not excluded, because not longer than five years.

B. SCHOLARSHIPS AND FELLOWSHIPS


117. Qualified Scholarships
(a) General Rule.Gross income does not include any amount received as a
qualified scholarship by an individual who is a candidate for a degree at an
educational organization described in section 170(b)(1)(A)(ii).
(b) Qualified Scholarship.For purposes of this section
(1) In general.The term qualified scholarship means any amount received by
an individual as a scholarship or fellowship grant to the extent the individual
establishes that, in accordance with the conditions of the grant, such amount
was used for qualified tuition and related expenses.
(2) Qualified tuition and related expenses.For purposes of paragraph (1), the
term qualified tuition and related expenses means

(A) Tuition and fees required for the enrollment or attendance of a student
at an educational organization described in section 170(b)(1)(A)(ii), and
(B) Fees, books, supplies, and equipment required for courses of instruction
at such an educational organization. **Room and board isnt included.
(c) Limitation.
(1) In general.Except as provided in paragraph (2), subsections (a) and (d)
shall not apply to that portion of any amount received which represents
payment for teaching, research, or other services by the student required as a
condition for receiving the qualified scholarship or qualified tuition reduction.
(2) Exceptions.Paragraph (1) shall not apply to any amount received by an
individual under
(A) The National Health Service Corps Scholarship Program under section
338A(g)(1)(A) of the Public Health Service Act, or
(B) The Armed Forces Health Professions Scholarship and Financial
Assistance program under subchapter I of chapter 105 of title 10,
United States Code.
(d) Qualified tuition reduction.
(1) In general.Gross income shall not include any qualified tuition reduction.
(2) Qualified tuition reduction.For purposes of this subsection, the term
qualified tuition reduction means the amount of any reduction in tuition
provided to an employee of an organization described in section 170(b)(1)(A)
(ii) for the education (below the graduate level) at such organization (or
another organization described in section 170(b)(1)(A)(ii)) of
(A) Such employee, or
(B) Any person treated as an employee (or whose use is treated as an
employee use) under the rules of section 132(h).
(3) Reduction must not discriminate in favor of highly compensated, etc.
Paragraph (1) shall apply with respect to any qualified tuition reduction
provided with respect to any highly compensated employee only if such
reduction is available on substantially the same terms to each member of a
group of employees which is defined under a reasonable classification set up
by the employer which does not discriminate in favor of highly compensated
employees (within the meaning of section 414(q)).
(4) Repealed
(5) Special rules for teaching and research assistants.In the case of the
education of an individual who is a graduate student at an educational
organization described in section 170(b)(1)(A)(ii) and who is engaged in
teaching or research activities for such organization, paragraph (2) shall be
applied as if it did not contain the phrase (below the graduate level).
127
(a) Exclusion from gross income.--

(1) In general.--Gross income of an employee does not include amounts paid or expenses
incurred by the employer for educational assistance to the employee if the assistance is
furnished pursuant to a program which is described in subsection (b).
(2) $5,250 maximum exclusion.--If, but for this paragraph, this section would exclude
from gross income more than $5,250 of educational assistance furnished to an individual
during a calendar year, this section shall apply only to the first $5,250 of such assistance
so furnished.
(b) Educational assistance program.-(1) In general.--For purposes of this section an educational assistance program is a
separate written plan of an employer for the exclusive benefit of his employees to provide
such employees with educational assistance. The program must meet the requirements of
paragraphs (2) through (6) of this subsection

Section 117(a) excludes from gross income amounts received as a qualified scholarship
by a degree candidate at an educational organization. The principal requirements of the
exclusion are found in the definition of a qualified scholarship, which is defined as any
amount received as a scholarship or fellowship grant that in accordance with the grant is
used for qualified tuition and related expenses. Those expenses encompass tuition and
enrollment fees at the educational organization as well as fees, books, supplies and
equipment required for courses of instruction. There is no exclusion for amounts which
cover personal living expenses, such as meals and lodging, or for travel and research.

Under Section 117(c), a portion of otherwise excluded scholarship or fellowship is


generally required to be included in the recipients gross income to the extent that the
portion represents a payment for teaching, research or other services by the student
required as a condition fro receiving the otherwise excludable amount.

Athletic Scholarshipsthe Service allows an exclusion for a university athletic


scholarship if the university expects but does not require the student to participate in a
particular sport, requires no particular activity in lieu of participation, and cannot
terminate the scholarship if the student cannot participate.

Section 117(d) allows a qualified tuition reduction to be excluded from gross income in
the case of education below the graduate level or at the graduate level if the graduate
student is engaged in teaching or research activities. There is a non-discrimination
requirement.
Note: Although a qualified tuition reduction is eligible for an exclusion from grossincome, to the extent that the reduction represents payment for research, teaching, or

other services required of the student to receive the reduction, the qualified tuition
reduction is required to be included in gross income.

Section 127 permits an employee to exclude up to $5,250 from gross income for amounts
paid by the employer for educational assistance at the undergraduate or graduate level of
education, provided the education assistance programs meets certain requirements related
primarily to non-discrimination in favor of highly compensated employees. Education
assistance under Section 127 includes tuition, books, supplies and an employer provided
educational course, but it does not include assistance for courses involving sports, games,
or hobbies.
Problems pg. 100-101
1. Student working toward an A.B. degree is awarded a scholarship of $15,000 for full
tuition and for room and board during the academic year. The tuition, including the cost
of books, is $10,000, and the room and board costs $5,000. As a scholarship recipient,
Student is required to do about 300 hours of research for the professor to whom he is
assigned. Non-scholarship students, if hired, receive $10.00 per hour for such work.
a.) What tax consequences to Student? 10k could be excluded. The other 5k is taxable.
What about the 3k excluded?
b.) What tax consequences to Student if all students are required to do 300 hours or
research for faculty? Same?
c.) What result if Student is not required to do any research but receives the $15,000 as
an athletic scholarship? Excluded. 10k is excludable, 5k for room and board is included.
d.) ) What tax consequences to Student if Student receives only a tuition scholarship
worth $9,000 (no books) because Students spouse is an employee at a neighboring
educations institution and the tuition scholarship is part of a nondiscriminatory plan
between several institutions applicable to all employees of such institutions? Excluded
under 117(d).
2. Secretary, in a large tax law firm, receives a $10,000 stipend from her firm to assist her
while on a leave of absence to obtain a college degree. The stipend is part of a firm plan
under which all recipients are required to return to the firm following their educational
leave.
a.) What tax consequences to Secretary? Exclude up to 5,250 even though its still
considered compensation for future services. The rest is included in gross income since
its basically compensation for services.
b.) ) What tax consequences to Secretary if she is not required to return to the firm
after competing her degree? 5,250 still excluded, the excess is still probably considered
compensation so that would be included.

c.) What are the tax consequences to Secretary if she is not an employee, but instead
receives the stipend as a prize in an essay contest? As long as it is tied to tuition and
related fees, its excluded.

CHAPTER 6 GAINS FROM DEALINGS IN PROPERTY


**Understand how the code is organized and its importance for the midterm. This is all
the way in subchapter in O, tells you its a whole new type of transfer.
Dirusso Breakdown: Amount realized Basis = Gain
*Basis is what you put into it. Most common is cost (how much you paid for it).
*Amount realized is how much you go for it.
1001. Determination of amount of and recognition of gain or loss
(a) Computation of gain or loss.The gain from the sale or other disposition of
property shall be the excess of the amount realized there from over the adjusted
basis provided in section 1011 for determining gain, and the loss shall be the
excess of the adjusted basis provided in such section for determining loss over the
amount realized.
(b) Amount realized.The amount realized from the sale or other disposition of
property shall be the sum of any mon3ey received plus the fair market value of
the property (other than money) received.
(c) Recognition of gain or loss.Except as otherwise provided in this subtitle, the
entire amount of the gain or loss, determined under this section, on the sale or
exchange of property shall be recognized. *Recognition doesnt occur until you
dispose the property.
1011. Adjusted basis for determining gain or loss
(a) General rule.The adjusted basis for determining the gain or loss from the sale
or other disposition of property, whenever acquired, shall be the basis (determined
under section 1012 or other applicable sections of this subchapter and subchapters
C (relating to corporate distributions and adjustments), K (relating to partners and
partnerships), and P (relating to capital gains and losses)), adjusted as provided in
subsection 1016.
1012. Basis of propertycost
(a) In general.The basis of property shall be the cost of such property, except as
otherwise provided in this subchapter and subchapter C, K and P.

Treasury Regulation 1.1001-1 Computation of gain or loss


(a) General rule. Except as otherwise provided in subtitle A of the Code, the gain or
loss realized from the conversion of property into cash, or from the exchange of
property for other property differing materially either in kind or in extent, is
treated as income or as loss sustained.
The amount realized from a sale or other disposition of property is the sum of any
money received plus the fair market value of any property (other than money)
received. The fair market value of property is a question of fact, but only in rare
and extraordinary cases will property be considered to have no fair market value.
The general method of computing such gain or loss is prescribed by section
1001(a) through (d) which contemplates that from the amount realized upon the
sale or exchange there shall be withdrawn a sum sufficient to restore the adjusted
basis prescribed by section 1011 and the regulations thereunder (i.e., the cost or
other basis adjusted for receipts, expenditures, losses, allowances, and other items
chargeable against and applicable to such cost or other basis).
The amount which remains after the adjusted basis has been restored to the
taxpayer constitutes the realized gain. If the amount realized upon the sale or
exchange is insufficient to restore to the taxpayer the adjusted basis of the
property, a loss is sustained to the extent of the difference between such adjusted
basis and the amount realized.. . . **Take out what you put into it, like spending
money to fix something.

A. DETERMINATION OF BASIS
-Cost basis: Most common and the default. Its just what you paid for it.
-Transfer basis- When your basis is derived from someone elses basis.
Gift: Donees basis is donors basis. Except for measuring loss, it becomes
the lower between fmv and the donors basis.
Transfer between Spouses:
-Date of Death Basis (Decedent)
-Basis may also be adjusted.
1. COST AS BASIS
*Unless another rule applies, the initial basis of property is its cost to the taxpayer.

1009. Improvements by lessee on lessors property

Gross income does not include income (other than rent) derived by a lessor of real
property on the termination of a lease, representing the value of such property attributable
to buildings erected or other improvements made by the lessee.

1011. Adjusted basis for determining gain or loss

(a) General rule.The adjusted basis for determining the gain or loss from the sale
or other disposition of property, whenever acquired, shall be the basis . . . adjusted
as provided in section 1016.
1012. Basis of propertycost

(a) In general.The basis of property shall be the cost of such property, except as
otherwise provided . . . .(What you paid for it)
Treasury Regulation 1.1012-1 Basis of Property
(a) General Rule. In general, the basis of property is the cost thereof. The cost is the
amount paid for such property in cash or other property. This general rule is
subject to exceptions stated in subchapter O, C, K and P.
1016. Adjustments to basis
(a)General rule.Proper adjustments in respect of the property shall in all cases be
made
(1) for expenditures, receipts, losses, or other items, properly chargeable to capital
account, but no such adjustment shall be made
(A) for taxes or other carrying charges described in section 266, or
(B) for expenditures described in section 173(relating to circulation expenditures),
For which deduction has been taken by the taxpayer in determining taxable income for
the taxable year or prior taxable years.

Gain = Amount realized - basis


Philadelphia Amusement Park Co. v. United States, Court of Claims of the U.S. (1954)In 1899, Taxpayer was granted a 50-year franchise to operate a passenger railway in
Philly. Taxpayer built a bridge at a cost of $381,000 for use by its streetcars. In 1946,
taxpayer deeded the bridge to the city in exchange for an extension of its franchise.
Taxpayer abandoned the extended franchise with three years left. Taxpayer asserted

depreciation deductions based on the cost of the extension and a loss upon abandonment
of the franchise.
*Real issue is basically how the cost basis of property is determined in a taxable
exchange. Court said that the cost basis is the fair market value of property received.
Issue: May the taxpayer use the fair market value of the franchise extension (as of the
date of the exchange) as the cost basis?
Held: The taxpayer was entitled to fair market value of the franchise extension (on the
date of the exchange for the bridge) as the cost basis. The Court remanded the case for
further consideration of what that fair market value actually was.
*The fair market value of what you received determines how much gain you should be
taxed on.
CLASS NOTES:
*If exchanging property, presumed that its equal value. So if something is hard to value
and you atleast know the value of the other, you can determine it.
Problem 2 on page 107
2. In an arms-length exchange, Sharp exchanges some land with a cost basis of
$6,000 and a value of $9,000 with Dull for some non-publicly traded stock which
Dull owns and in which Dull has a basis of $8,000 and is worth $10,000 at the time of
the exchange.
a.) Consider Sharp and Dulls gains on the exchange and their respective cost bases
in the assets that they receive. Sharps amount realized 10k- sharps basis of 6k = has
a gain of 4k. Dull has amount realized of 9k- Dulls basis of 8k =gain of 1k. Sharp
cost basis is 10k and Dulls cost basis is 9k? OR
b.) What results in (a), above, if the value of Dulls stock cannot be determined
with any reasonable certainty? Assume that the fair market values are equal. So
Sharp would now have a 3k gain. Amount realized of 9k Basis of 6k= 3k.

1. PROPERTY ACQUIRED BY GIFT


CLASS NOTES: Basis for gifts is a modified transfer basis rule. There is pure
transfer basis (donees basis = donors basis) except if FMV< basis on the date of gift,
FMV at date of gift is used as the basis for purposes of calculating loss. Why?
Because you dont want to shift losses among taxpayers. Only use the lower FMV.

1015. Basis of property acquired by gifts and transfer in trust

(a) Gifts after December 31, 1920.If the property was acquired by gift after
December 31, 1920, the basis shall be the same as it would be in the hands of the
donor or the last preceding owner by whom it was not acquired by gift, except
that if such basis (adjusted for the period before the date of the gift as provided in
section 1016) is greater than the fair market value of the property at the time of
the gift, then for the purpose of determining gain or loss the basis shall be such
fair market value. If the facts necessary to determine the basis in the hands of the
donor or the last preceding owner are unknown to the donee, the Secretary shall,
if possible, obtain such facts from such donor or last preceding owner, or any
other person cognizant thereof. If the Secretary finds it impossible to obtain such
facts, the basis in the hands of such donor or last preceding owner shall be the fair
market value of such property as found by the Secretary as of the date or
approximate date at which, according to the best information that the Secretary is
able to obtain, such property was acquired by such donor or last preceding owner.
Treasury Reg. 1.1015-1 Basis for property acquired by gift after December 31,
1920.
(a) General Rule.
(1) In the case of property acquired by gift after December 31, 1920 (whether by
a transfer in trust or otherwise), the basis of the property for the purpose of
determining gain is the same as it would be in the hands of the donor or the
last preceding owner by whom it was not acquired by gift. The same rule
applies in determining loss unless the basis (adjusted for the period prior to
the date of gift in accordance with sections 1016 and 1017) is greater than the
fair market value of the property at the time of the gift. In such cases, the basis
for determining loss is the fair market value at het time of the gift.
(2) The provisions in subparagraph (1) of this paragraph may be illustrated by the
following example.
Example: A acquires by gift income-producing property, which has an adjusted
basis of $100,000 at the date of gift. The fair market value of the property at the
date of gift is $90,000. A later sells the property for $95,000. In such cases, there
is neither gain nor loss. The basis for determining loss is $90,000; therefore, there
is no loss. Furthermore, there is no gain, since the basis for determining gain in is
$100,000.
Section 1015(a), Property Acquired by Gift
i. Generally, for property acquired by gift, basis is the same as it would be in the
hands of the donor.
ii.
If the property has a fair market value less than basis (its taken a loss), basis is
the fair market value.
Taft v. Bowers, U.S. Supreme Court (1929)

Father gave shares of stock to daughter in 1921 and 1922. She sold them in 1923 for
more than their fair market value at the time the gift was made. The I.R.S. demanded
income tax on the difference between the cost to the donor and the price received by the
daughter. She paid the tax and then demanded a refund.
*She thought the cost basis should be determined by the fair market value of the stock
when it was given to her, and not when he originally bought it.
Held: The Court Affirmed and held that she should be taxed on the gain based on the
value of the stock when acquired by the donor.
*When she accepted the gift, she assumed the position of the owner.
The Court noted that Taft took the gift knowing the law and she benefited from the
increase in value.
Further, when the stock was sold she did not receive only gain in value, but it included
the original amount as paid by her donor father.

FaridEsSultaneh v. Commissioner,
Man and woman had an agreement that she would promise to marry him in exchange for
shares of stock.
Sulataneh and Kresge were eventually divorced. Sultaneh then sold 12,000 shares of the
S.S. Kresge stock for $230,802.36.
-The Commissioner of Internal Revenue characterized the stock as a gift and thereby
counted the donors basis for determining the gains.
-Wife contended that the stock was not a gift; it was consideration in a contract.
Therefore, the basis should be calculated from the date of her acquisition. The Tax Court
found for the Commr.
HOLDING: The stock was not a gift and thus, the basis at the time acquired by Miss
Sultaneh should be used.
*She gave consideration for the stock, a promise to marry so no gift. Costs basis is the
date she acquired the stock.
*A transfer that is designated as a gift under gift tax law doesnt necessarily mean it is to
be treated as a gift income-tax wise.
CLASS NOTES:
Sale = Cost Basis Rule
Gift= Transfer Basis Rule

***Problem 1 pg. 113 ****WILLL BE ON MIDTERM******** WILL BE


TRICKIER
1. A.) Donor gave Donee property under circumstances that required no payment of gift
tax. What gain or loss to Donee on the subsequent sale of the property if:
1. 35,000? 15k gain. Use carryover basis.
2.) 15,000? 5k loss. Because the basis isnt greater than the FMV.
3.) 25,000? 5k gain.
B.) The property had cost Donor $30,000 and had a $20,000 fair market value at the time
of the gift, and Donee sold it for:
1.) $35,000? 5k gain. Carryover basis
2.) 15,00? 5k loss.
3.) 24,000? No gain an no loss since its between the two bases.

1. PROPERTY ACQUIRED BETWEEN SPOUSES OR INCIDENT TO


DIVORCE
CLASS NOTESL: Generally, when property is transferred between spouses or incident to
divorce, we use a pure transfer basis rule and do not recognize gain or loss on the
transfer.

1041. Transfers of property between spouses or incident to divorce

(a) General rule.No gain or loss shall be recognized on a transfer of property from
an individual to (or in trust for the benefit of)
(1) A spouse, or
(2) A former spouse, but only if the transfer is incident to the divorce.
(b) Transfer treated as a gift; transferee has transferors basis.In the case of
any transfer of property described in subsection (a)

(1) For purposes of this subtitle, the property shall be treated as acquired by the
transferee by gift, and
(2) The basis of the transferee in the property shall be the adjusted basis of the
transferor.
(c) Incident to divorce.For purposes of subsection (a)(2), a transfer of property is
incident to the divorce if such transfer
(1) Occurs within 1 year after the date on which the marriage ceases, or
(2) Is related to the cessation of the marriage.

Property Acquired between Spouses or Incident to Divorce:


Section 1041:

1. 1041(a): no gain or loss shall be recognized on a transfer of property from an


individual to (or in trust for the benefit of)
a. A spouse; or
b. A former spouse, so long as the transfer is incident to a divorce.
2. 1041(c): Incident to Divorcea transfer of property is incident to a divorce if the
transfer:
a. Occurs within one year of the cessation of marriage; or
b. Is otherwise related to the cessation of marriage.
3. Like a Giftthis code section treats transfers between spouses/former spouses as
gifts; the basis of the transferee is the basis of the transferor.
4. No Loss Exceptionunlike the gift rules, there is no loss exception permitting
use of the fair market value as the basis. The basis of the transferor is the basis,
flat out.
Problem 1 pg. 115
1. Andre purchased some land ten years ago for $40,000 cash. The property appreciated
to $70,000 at which time Andre sold it to his wife Steffi for $70,000 cash, its fair market
value.
a.) What are the income tax consequences to Andre? No tax consequences: do
not recognize gain or loss in transfer between spouses.
b.) What is Steffis basis in the property? 40k. Pure Carryover basis.
c.) ) What gain to Steffi if she immediately resells the property? If she sells it
for the fair market value, it would be 30k.

d.) ) What results in (a)-(c) above, if the property had declined in value to
$30,000 and Andree sold it to Steffi for $30,000? Non-recognition of gain or
loss.
e.) ) What results (gains, losses, and bases) to Andre and Steffi if Steffi transfers
other property with a basis of $50,000 and value of $70,000 (other than cash)
to Andre in return for his property? No gains or losses. Andres basis would be
50k in the property.

2. PROPERTY ACQUIRED FROM A DECEDENT


**Rule: Date of Death Basis is the default rule. Basis is the fair market value on the date
of death.
a. I.R.C. 1014(a), Property Acquired form a Decedentexcept as
otherwise provided, the basis of property acquired from a decedent is the
fair market value at the time of the decedents death.
i. Because this often results in a tax-free gain, its often called the
step-up basis rule.
ii. When property depreciates before devise, however, it can result in
a stepped-down basis, which cannot be deducted by anyone.
(b) Property acquired from the decedent.--For purposes of subsection (a), the
following property shall be considered to have been acquired from or to have
passed from the decedent:
(1) Property acquired by bequest, devise, or inheritance, or by the decedent's
estate from the decedent;
(2) Property transferred by the decedent during his lifetime in trust to pay the
income for life to or on the order or direction of the decedent, with the right
reserved to the decedent at all times before his death to revoke the trust;
(3) In the case of decedents dying after December 31, 1951, property transferred
by the decedent during his lifetime in trust to pay the income for life to or on the
order or direction of the decedent with the right reserved to the decedent at all
times before his death to make any change in the enjoyment thereof through the
exercise of a power to alter, amend, or terminate the trust;
(4) Property passing without full and adequate consideration under a general
power of appointment exercised by the decedent by will.
ANIT MONKEY BUSINESS RULE (Drafted to prevent people from abusing
the system): I.R.C. 1014(e), Property Acquired within 1 year of Deathif
appreciated property is acquired by a decedent within the one-year period ending

on the decedents death AND if the property (or proceeds therefrom) passes from
the decedent back to the donor or the donors spouse, the basis in the property is
the adjusted basis of the property in the hands of the decedent immediately before
the death.
The Section 1014 basis rule is an important element in estate planning. It means that,
although appreciated property is fully subjected to the estate tax, the appreciation itself
entirely escapes the income tax. Thus, elderly people with substantially appreciated
property often choose not to sell such property in order to avoid income taxation and are
said to be locked-in to their positions.

Problem 1 pg. 118


In the current year, Giver holds two blocks of identical stock, both worth $1,000,000.
Giver purchased the first block years ago for $50,000 and the second block more recently
for $950,000. Giver plans to make an inter vivos gift of one block and retain the second
until death. Which block of stock should Giver transfer inter vivos and why?
He should gift the most recent stock that he purchased for 950,000 because the basis for
the older stock would be one million instead of 50k if acquired from a decedent. Gift
requires transfer basis, and acquired through deaths requires date of death business.
C. THE AMOUNT REALIZED
Gain = Amount Realized Adjusted Basis
*We include indebtedness in your debt and amount realized.
I.R.C. 1001(b), Amount Realizedthe amount realized from the sale of other
disposition of property is the sum of any money received + the fair market value of the
property (other than money) received.
IRC 1016 Adjustment Basis
(a) General rule.--Proper adjustment in respect of the property shall in all cases be
made-(1) for expenditures, receipts, losses, or other items, properly chargeable to capital
account
(2) in respect of any period since February 28, 1913, for exhaustion, wear and tear,
obsolescence, amortization, and depletion, to the extent of the amount-(A) allowed as deductions in computing taxable income under this subtitle or prior
income tax laws, and
(B) resulting (by reason of the deductions so allowed) in a reduction for any taxable year
of the taxpayer's taxes under this subtitle

*Relief of indebtedness is included in your amount realized and allow you to take a
depreciation deduction.
International Freighting Corp. v. Commissioner
-The taxpayer in this case is a corporation. The corporation had a bonus plan in which it
would pay their employees shares of common stock of their parent company.
-In 1936, the corporation gave out bonuses in shares of stock, which cost the corporation
about 16k but had a market value of 24k.
-Corporation took a 24k deduction on account of the bonuses paid but the Commissioner
reduced the deduction to 16k arguing that the basis for determining the amount is the cost
of the property and not the fair market value.
- Commissioner argued that if Taxpayer were entitled to a deduction in the amount of
$24k, then Taxpayer realized a gain that should be reported as taxable profit. The Tax
Court held that the Taxpayer was entitled to the full deduction and that the profit should
count as gross income.
-**Court of appeals affirmed the tax courts ruling, holding that the market value was the
stock was properly deductible and that the transfer to the employees resulted in a taxable
gain.
-***There was a taxable gain on the difference between the market value of the shares
and the actual cost of the shares.
-**The delivery of shares was a disposition for valid consideration, but resulted in a
taxable gain.
((Amount realized was the fmv of the stock.

Crane v. Commissioner, U.S. Supreme Court (1947)


Mrs. Crane was the sole beneficiary of her husbands will and inherited an apartment
building and lot, which was valued at $255k, and subject to a mortgage of the same
amount. Crane entered into an agreement with the mortgage holder, which stipulated she
would continue to operate the property and give profits to mortgage holder.
-Seven years later Crane sold the property to a third party for $3k in cash, subject to the
mortgage and paid $500 in sale expenses.
She reported a taxable gain of $1,250, on the theory that the property she sold was the
equity in the building (i.e., the value of the building in excess of the mortgage).
Because she never assumed the mortgage, she reported the net cash received, $2,500, half
of which ($1,250) was taxable as a capital asset.
*She argued that what was sold was the equity in the property with a 0 basis. IRS said the
property sold wasnt equity but the physical property.

-The Commissioner of Internal Revenue determined that Crane realized a net taxable gain
of $23,767.03. The Commissioners theory was that the property sold was not equity in
the building; it was the physical property. Per the Commissioner, the basis reduced based
on the deductions she took, and as a result, the basis was less than the mortgage
liability. The difference between the liability and the basis$23,767.03constituted a
gain when she sold the property and the liability
HOLDING: -Supreme Court agreed with the Commissioner held that the proper basis
was the value of the property, adjusted by deductions, but undiminished by the mortgage.
The Court looked at the meaning of property and concluded that it included the physical
thing that is subject to ownership or the aggregate of the owners rights to control and
dispose of it.
Seller received benefit by relief of mortgage indebtedness
Rule: A person who sells property with by a nonrecourse mortgage, must include the
unpaid balance of the mortgage in when computing the amount realized.
This case stands for the idea that non-recourse liability will occupy the same posture for
tax purposes as full recourse liability does.
Basically, a taxpayer is treated as having realized proceeds from the elimination of a debt
even if that taxpayer had no personal liability for the debt.
**RELIEF of indebtness is

*** Add SECTION 1016 Adjustment to Basis

Commissioner v. Tufts, U.S. Supreme Court (1983)


Taxpayers had a partnership, which owned property, which was secured by a nonrecourse mortgage, meaning that neither the partnership nor any of its partners had any
personal liability for the loan (the Bank would just take the property on default).
-The mortgage was for 1.85 million. When the taxpayers sold the property, the fair
market value was less than 1,400,000. The adjusted basis at the time of the sale was
1,450,000 million. Each partner sold his partnership interest to an unrelated third party,
As consideration, third party assumed the nonrecourse mortgage and associated fees.
-Each partner reported a partnership loss of 55k.
-The Commissioner of Internal Revenue determined that the sale of the partnership
resulted in a capital gain of approximately $400,000 arguing that the partnership
benefited from the third party assuming the nonrecourse mortgage.

**This case basically asks whether the Crane ruling applies when the unpaid amount of a
nonrecourse mortgage exceeds the fair market value of the property sold. Court said it
did.
Held: Supreme Court rule in favor of IR.
*When a taxpayer sells or disposes of property, he is required to include the outstanding
amount of the obligation as part of the amount realized.
It isnt relevant that this was a non-recourse loan or that the loan was in excess of the fair
market value of the property at the time.
-Its just as if the buyer gave them the full 1.8 to fulfill their obligation.
Problems 1,2 pg. 135-136
1.) Mortgagor purchases a parcel of land from Seller for $100,000. Mortgagor borrows
$80,000 from Bank and pays that amount and an additional $20,000 of cash to seller
giving Bank a nonrecourse mortgage on the land. The land is the security for the
mortgage which bears an adequate interest rate.
a.) What is Mortgagors cost basis in the land? 100k
b.) Does Mortgagor have income when she takes out a second nonrecourse
mortgage after the property has appreciated to $300,000? No. Loans arent
considered income. Still has an obligation to repay.
c.) What is Mortgagors basis in the land if the $100,000 or mortgage proceeds
are used to improve the land? 200k. Example of how you can adjust basis.
d.) What is Mortgagors basis in the land if the $100,000 or mortgage proceeds
are used to purchase stock and bonds worth $100,000? 100k basis in the land.
Other 100k not used to improve the land.
e.) What result under the facts of (d) above if when the principal amount of the
two mortgages is still $180,000 and the land is still worth $300,000, Mortgagor
sells the property subject to both mortgages to Purchaser for $120,000 of cash?
What is Purchasers cost basis in the land? The amount realized will be 300k for
the mortgagor with a 200 gain. Purchasers basis would only be 300k (120 cash,
and 180 in mortgages is what he put into it).
2.) Investor purchased three acres of land, each acre worth $100,000 for $300,000.
Investor sold one of the acres in year one for $140,000 and a second in year two for
$160,000. The total amount realized by Investor was $300,000 which is not in excess of
her total purchase price. Does Investor have any gain or loss on the sales? It would be a
gain. Basis in each parcel would be 100k. 40k gain on the first, and 60k gain on the
second.

Ch. 7 LIFE INSURANCE PROCEEDS AND


ANNUITIES
A. LIFE INSURANCE PROCEEDS
Generally, life insurance proceeds paid by reason of the death of the insured are excluded
from gross income. Similar to gifts.

101. Certain death benefits.


I.R.C. 101(a)(1), General Rulea taxpayer may exclude from gross income the
proceeds of life insurance policies, paid upon the death of the insured. [This is solely a
judgment of public policy].

I.R.C. 101(a)(2), Transfer for valuable consideration.--In the case of a transfer for a
valuable consideration, by assignment or otherwise, of a life insurance contract or any
interest therein, the amount excluded from gross income by paragraph (1) shall not
exceed an amount equal to the sum of the actual value of such consideration and the
premiums and other amounts subsequently paid by the transferee. The preceding sentence
shall not apply in the case of such a transfer-(A) if such contract or interest therein has a basis for determining gain or loss in
the hands of a transferee determined in whole or in part by reference to such basis
of such contract or interest therein in the hands of the transferor, or
(B) if such transfer is to the insured, to a partner of the insured, to a partnership in
which the insured is a partner, or to a corporation in which the insured is a
shareholder or officer.
The term other amounts in the first sentence of this paragraph includes interest paid or
accrued by the transferee on indebtedness with respect to such contract or any interest
therein if such interest paid or accrued is not allowable as a deduction by reason of
section 264(a)(4).
CLASS NOTES: Most of the time insurance proceeds of gratuitous transfer are not
taxable, but if you sold it or treated it like an investment, then it is taxable.
But, often times it becomes necessary to tap the life insurance before you die. If you are
chronically or terminally ill, you may be unable to work, but still have substantial
expenses. To deal with this, Congress has put special rules into the Code for accelerated
death benefits.

101(c): Interest is included in taxable income.

(d) Payment of life insurance proceeds at a date later than death.-(1) General rule.--The amounts held by an insurer with respect to any beneficiary
shall be prorated over the period or periods with respect to which such
payments are to be made. There shall be excluded from the gross income of such
beneficiary in the taxable year received any amount determined by such proration.
Gross income includes, to the extent not excluded by the preceding sentence,
amounts received under agreements to which this subsection applies.
(2) Amount held by an insurer.--An amount held by an insurer with respect to
any beneficiary shall mean an amount to which subsection (a) applies which is-(A) held by any insurer under an agreement provided for in the life
insurance contract, whether as an option or otherwise, to pay such amount
on a date or dates later than the death of the insured, and
(B) equal to the value of such agreement to such beneficiary
(i) as of the date of death of the insured (as if any option exercised
under the life insurance contract were exercised at such time), and
(ii) as discounted on the basis of the interest rate used by the
insurer in calculating payments under the agreement and mortality
tables prescribed by the Secretary.
(3) Application of subsection.--This subsection shall not apply to any amount to
which subsection (c) is applicable.
I.R.C. 101(g), Accelerated Death Benefitsbenefits received from a life insurance
policy on the life of a terminally ill or chronically ill insured person are treated as paid
by reason of the death of the insured under 101(a)(1), and as a consequence are
excluded from gross income.
I.
II.

Designation as Terminally Ill requires certification by a physician that the


insured is reasonably expected to die within 24 months. There is no amount
ceiling applicable to the terminally ill exception.
Designation as Chronically Ill requires certification by a physician within the
preceding 12-month period that the insured is unable to perform at least two
activities of daily living for a period of at least 90 days due to a loss of functional
capacity. The chronically ill exception is subject to an amount ceiling of $175/day
less medical insurance reimbursements.

Another issue to consider is that of insureds who take money before death, who receive
amounts in excess of the policy value.

i.
ii.

If one takes a cash surrender during life, the insured will realize an amount in
excess of the basis, which is taxable gain unprotected by these exclusions.
If a beneficiary takes fixed monthly payments for life, these may also cause the
realization of an amount in excess of basis. If this is the case, one must turn to the
mortality tables to calculate the taxable amount.

Viatical Settlements
I.R.C. 101(g)(2), Viatical SettlementsAny amount one receives from a viatical
settlement is considered to be by reason of death, and therefore not taxable.
DIRUSSO KEY POINTS
Generally, life insurance proceeds paid by reason of the death of the insured are excluded
from gross income.
Transfers during life of an insurance policy do not benefit from this exclusion and may
trigger taxable gain, except in the case of Viatical settlements for terminally ill or
chronically ill people.
Interest payments on insurance proceeds remain taxable.
Insurance proceeds paid out over time are excluded only.
Problems pg. 140
1. Insured died in the current year owning a policy of insurance that would pay
Beneficiary $100,000 but under which several alternatives were available to beneficiary.
a. What result if Beneficiary simply accepts the $100,000 in cash? Excludable.
Treat just like a gift.
b. What result in (a), above, if Beneficiary instead leaves all the proceeds with
the company and they pay her $6,000 interest in the current year? 6k interest is
not excludable.
c. What result if Insureds daughter is beneficiary of the policy and in accordance
with an option that she elects, the company pays her $12,000 in the current year?
Assume that such payments will be made annually for her life and hat she has a
25-year life expectancy. Use a ratio for policies paid overtime. 4k each year is tax
free. (100k/25). The rest is subject to tax.
d. What result in (c) above if Insureds daughter lives beyond her 25-year life
expectancy and receives $12,000 in the twenty-sixth year? Still going to be taxed
on the 8k and not the 4k.
3.

Insured purchases a single-premium $100,000 life insurance policy on her life for
a cost of $40,000. Consider the income tax consequences to Insured and the
purchaser of the policy in each of the following alternative situations

a. Insured sells the policy to her Child for its $60,000 fair market value and on
Insureds death the $100,000 of proceeds are paid to Child. Taxable on the 40k.
100k-60k. Not a gift but a transfer. Insurer would pay tax on 20k. (60-40)
b. Insured sells the policy to her Spouse for its $60,000 fair market value and on
Insureds death the $100,000 of proceeds are paid to Spouse. Spouse gets a
transfer basis of 40k. No gain recognized.
c. Insured is certified by her physician as terminally ill and she sells the policy for
its $80,000 fair market value to Viatical Settlement Provider who collects the
$100,000 of proceeds on Insureds death? Excludable because shes terminally
ill. Viatical settlement provider basis would be 80k.

B. ANNUITY PAYMENTS
-Generally, only the portion of an annuity payment that reflects the return of the initial
capital investment is excluded from gross income. The portion that reflects earnings
from that investment is included.
-Once you get your entire investment back, everything else is subject to tax. : If you die
before you get all your basis back, you may deduct what remains of your basis in that
year.
-Different from life insurance.
Exclusion Ratio: Same as life insurance
Problem 1 pg. 145
1. In the current year, T purchases a single life annuity with no refund feature for
$48,000. Under the contract T is to receive $3,000 per year for life. T has a 24-year life
expectancy.
a. To what extent, if at all, is T taxable on the $3,000 received in the first
year? Exclusion Ratio: 3k*24= 72k is expected return. We put 48k in. So
what is 48/72 reduces to 2/3. So two thirds is excluded and only taxed on
one third, so he will be taxed on 1k per year, other 2k is excluded.
b.

If the law remains the same and T is still alive, how will T be taxed on the
$3,000 received in the thirtieth year of the annuity payments? Will be
taxed on all of it (different from life insurance).

c. If T dies after nine years of payments will T or Ts estate be allowed an


income tax deduction? How much? Yes (different from life insurance).
Deduction is the unrecovered capital, so a deduction of 30k.

d. To what extent are T and Ts spouse taxable on the $3,000 received in the
current year if at a cost of $76,500 they purchase a joint and survivorship
annuity to pay $3,000 per year as long as either lives and they have a joint
life expectancy of 34 years? 3k*34: 102k. Ends up giving you a
exclusion ratio. 2,250 is excluded. So 750 is taxable.

CH. 8 DISCHARGE OF INDEBTEDNESS


-Unless it falls within en exception, discharge of indebtedness reflects taxable income.
61(a)(12), Gross Incomegross income, generally, includes income from the
discharge of indebtedness.
108(a), Exclusion [of Certain Discharge of Indebtedness] From Gross Incomea
taxpayer may exclude from gross income debt discharge if
A.
B.
C.
D.

The discharge occurs in a title 11 case, (Title 11 Bankruptcy)


The discharge occurs when the taxpayer is insolvent,
The debt discharged is qualified farm indebtedness,
The debt is qualified real property business debt (and the taxpayer is not a C
Corp), or
E. The debit is qualified principle residence debt (discharged before Jan. 1, 2014).
U.S. v. Kirby Lumber Co., United States Supreme Court (1931)
Kirby Lumber issued corporate bonds. Later in the year, Kirby was able to repurchase
those bonds at below par value, thereby cancelling the obligation to pay interest on those
bonds, equaling $137,500 of debt discharged. The issue in this case was whether that
discharge of that debt was income?
Held: Yes, it is income.
The Court: [I]f the corporation purchases and retires any bonds at a price less than
the issuing price or face value, the excess of the issuing price or face value over the
purchase price is gain or income for the taxable year.
CLAS NOITES: Bonds are basically loans. There is relief from indebtedness and is
taxable ncome.
Zarin v. Commissioner, 3rd Circuit (1990)
Zarin was a compulsive gambler, he liked to go to the boats a little too much, and when
he went, he had a habit of losing. In fact, he lost a lot of money (millions). He had been

given credit at the casino. So, he was losing on credit. The casino brought an action
against Zarin to recover the debt. Zarin protested, claiming that under New Jersey law, he
did not owe the debt. They settled for $500,000. The I.R.S. attempted to collect the
balance of the debt as income from the forgiveness of debt.
Held: No income. The settlement is governed by the contested liability doctrine. Under
which a taxpayer, who in good faith, disputes the amount of the debt, may treat the
settlement value as the amount of debt for tax purposes. Since Zarin was deemed to have
owed $500,000, and because he paid the $500,000 debt, there were no adverse tax
consequences to Zarin.
CLASS NOTES: Did not find a financial gain because theres no relief from indebtedness
since it was not a legally enforceable debt.
**Special Rule applies to bankruptcy. In certain proceedings you wont recognize the
relief of indebtedness as taxable income.

(f) Student loans.-(1) In general.--In the case of an individual, gross income does not include any amount
which (but for this subsection) would be includible in gross income by reason of the
discharge (in whole or in part) of any student loan if such discharge was pursuant to a
provision of such loan under which all or part of the indebtedness of the individual would
be discharged if the individual worked for a certain period of time in certain professions
for any of a broad class of employers.
(2) Student loan.--For purposes of this subsection, the term student loan means any
loan to an individual to assist the individual in attending an educational organization
described in section 170(b)(1)(A)(ii) made by
(3) Exception for discharges on account of services performed for certain lenders.
(4) Payments under National Health Service Corps Loan Repayment Program and
certain State loan repayment programs
Revenue Ruling 2008-34
Facts: C, attended law school and has student loan debt. The loan documents do not
specify whether any of the indebtedness would be forgiven if C worked in a particular
profession for a specified period of time. Cs law school offers a Loan Repayment
Assistance Program (LRAP) to help reduce the student loan debt of graduates who
engage in public service. To qualify a graduate must work in a law-related public service
position for a tax-exempt charitable organization or a governmental unit.
Law:

Under Section 61(a)(1) gross income includes income from the discharge of
indebtedness.

Section 108(f)(1) provides that in the case of an individual, gross income does not
include any amount from the discharge of any student loan if the discharge was
pursuant to a provision of such loan under which all or part of the indebtedness
would be discharge if the individual worked for a certain period of time in a
certain profession for any of a broad class of employers.
Section 108(f)(2) defines student loan to include any loan to an individual to
assist the individual in attending an educational organization. This includes loans
made by the educational institution which are designed to encourage its students
to serve in occupations with unmet needs or in areas with unmet needs, where the
services provided by the student are for or under the direction of a non-profit or
governmental organization. A public service requirement is a part of this type of
loan.
Analysis: The terms of Cs LARP provides for loan forgiveness only if C works for a
certain minimum period of time in a qualifying law-related public service position. This
comports with the requirements of 108(f)(1). Additionally, the law schools LARP is
designed to encourage its students to engage in public service in occupations and areas of
unmet needs, meeting the definition of student loan in 108(f)(2).
Holding: The terms of the loan made under the LARP satisfy the requirements of
108(f)(1), and the LARP loan is a student loan within the meaning of 108(f)(2).

Problems pg. 162


1. Poor borrowed $10,000 from Rich several years ago. What tax consequences to Poor
if Poor pays of the so far undiminished debt with:
(a) A settlement of $7,000 of cash? Taxable Income of 3k. Example of the general rule. It
is legally enforceable.
(b) A painting with a basis and fair market value of $8,000? 2k worth of taxable income.
(c) A painting with a value of $8,000 and a basis of $5,000? 5k worth of taxable income.
(d) Services, in the form or remodeling Richs office, which are worth $10,000? 10k
worth of taxable income. Compensation for services.
(e) Services that are worth $8,000? 10k worth of taxable income.
(f) Same as (a), above, except that Poors employer makes the $7,000 payment to Rich,
renouncing any claim to repayment by Poor. 10k worth of taxable income. 3k relief and
7k of indirect compensation.
4. Decedent owed Friend $5,000 and Nephew owed Decedent $10,000.
(a) At Decedents death, Friend neglected to file a claim against Decedents estate in the
time allowed by state law and Friends claim was barred by the statute of limitations.

(Lets defer our concern for Nephew.) What result to Decedents estate? Debt is
discharged so 5k of taxable income to the estate because the SOL had run.
(b) What result to the estate in (a) above (with Decedent still in cold storage) if instead
Friend simply permitted the statute to run stating that she felt sorry for Decedents
widow, the residuary beneficiary of his estate? No relief of indebtedness comes off as a
gift. Looks like discharge but it is a gift.
(c) Now, what result to Nephew if Decedents will provided that his estate not collect
Nephews debt to the estate? Not taxable because its a bequest.
**Gratuitous transfers for relief of indebtedness are not taxable income.**

Ch. 9 Damages and Related Receipts


A. Intro: Whether damages are included in taxable income largely depends upon what
loss the damages represent.
B. Damages in General
Not only damages that reflect lost profits, but other damages that substitute for taxable
income, are includable in gross income.
Raytheon Production Corp. v. Commissioner, 1st Cir. (1944)
Raytheon recovered damages in an anti-trust settlement against RCA. In the suit,
Raytheon claimed that RCA completely destroyed the profitable business of Raytheon,
wholly destroying the business and goodwill of the enterprise. Are the damages taxable as
gross income?
Holding: To the extent that the damages are reimbursements for the loss of good will,
they are not taxable. However, any portion of the damage award that is over the value of
the good will are income.
Rule of the Case: Recoveries for reimbursement of lost profits are income, but
recoveries for injury to good will are return of capital. The critical question is: In lieu of
what were the damages awarded? As a rule, if the damages are awarded in lieu of
profits, the damages are taxable; if the damages are awarded in lieu of injury, then the
damages are not taxable.
Problems pg. 168
Plaintiff brought suit and unless otherwise indicated successfully recovered. Discuss the
tax consequences in the following alternative situations.
(a) Plaintiffs suit was based on a recovery of an $8,000 loan made to Debtor. Plaintiff
recovered $8,500 cash, $8,000 for the loan plus $500 of interest. 500 of interest is
taxable. Amount over return of capital is subject to tax.

(b) What result to Debtor under the facts of (a) above if instead Debtor transferred some
land worth $8,500 with a basis of $2,000 to Plaintiff to satisfy the obligation? What is
Plaintiffs basis in the land? 6,500 of taxable income. Ps basis is 8,500.
(c) Plaintiffs suit was based on a breach of a business contract and Plaintiff recovered
$8,000 for lost profits and also recovered $16,000 of punitive damages. 24k is taxable.
Punitive damages are always taxable.
(d) Plaintiffs suit was based on a claim of injury to the goodwill of Plaintiffs business
arising from a breach of a business contract. Plaintiff had a $4,000 basis for the goodwill.
The goodwill was worth $10,000 at the time of the breach of contract.
(1) What result to Plaintiff if the suit is settled for $10,000 in a situation where the
goodwill was totally destroyed? 6k taxable and 4k of return of capital.
(2) What result if Plaintiff recovered $4,000 because the goodwill was partially
destroyed and worth only $6,000 after the breach of contract? Just return of capital for
the first 4k.
(3) What result if Plaintiff recovers only $3,000 because the goodwill was worth $7,000
after the breach of contract? 3k is
C. Damages and other Recoveries for Personal Injuries
- Certain damages that reflect payment for physical injuries or physical sickness may be
excluded from taxable income.
I.R.C. 104(a): ) In general.--Except in the case of amounts attributable to (and not in
excess of) deductions allowed under section 213 (relating to medical, etc., expenses) for
any prior taxable year, gross income does not include
I.R.C. 104(a)(1), Workers Compensation Excludesbenefits paid to an employees
survivors under state WC acts and similar statutes in the case of jobrelated death.
Does Not Excludebenefits paid to the employee for jobrelated injuries; non
occupational benefits, such as amounts paid for disability during employment but not
caused by injury or sickness.
104(a)(2), Tort Damages Excludesdamages incurred on account of personal physical
injuries or physical sickness; damages incurred emotional distress to the extent those
damages required medical care; does not exclude other ED damages.
Does Not Excludedamages for nonphysical injuries, such as defamation, First
Amendment rights, and sex and age discrimination.
Settlement AgreementsIf the parties settle, the Tax Court will follow the express
allocation of compensatory and punitive damages in the settlement agreement for tax
purposes if the agreement is reached in an adversarial context, at arms length, and in
good faith. Trial Judgmentthe court will use the values demanded in the complaint to
establish the ration of nontaxable compensatory damages to taxable punitive damages.

104(a)(3), Personal Pay Health Insurance Excludesamounts received from accident


and health insurance policies for personal injuries or sickness. This is limited to proceeds
of policies paid for by the individual.
Does Not Excludeemployeepaid or employeesubsidized policies.
104(a)(4), (5), Disabilities Excludesdisability pensions of armed forces and other
federal government units. Also Excludesdisability income attributable to injures
sustained in terrorist or military attacks.

EMOTIONAL DISTRESS UNDER 104(a)(2):


The taxation of recoveries for emotional distress depend on the nature of the underlying
action:
Damages recovered for emotional distress incurred on account of physical injury
are excludable.
However, emotional distress is not a physical injury, despite the manifestation of
physical symptoms, and recoveries arising out of emotional distress are included in
gross income except to the extent that damages are received for amounts paid for medical
care which is attributable to the emotional distress

.Emotional distress is not considered a physical injury or physical sickness. However,


damages for emotional distress attributable to a physical injury or physical sickness are
excluded from income under section 104(a)(2). Section 104(a)(2) also excludes damages
not in excess of the amount paid for medical care (described in section 213(d)(1)(A) or
(B)) for emotional distress
SECTION 105 EMPLOYER PAID HEALTH INSURANCE
Section 105(a)Includes in gross income amounts that an employee receives through
accident or health insurance IF:
1. Attributable to an employers contributions to a plan which were not taxed to the
employee under Section 106(a); OR
2. Paid directly by the employer.
Section 105(b)If an employer directly or indirectly reimburses an employee for
expenses of medical care for the employee or employees spouse or dependents, the
amount received is excluded from gross income under Section 105(b). Note: It is the
amount of medical care actually paid for which determines the exclusion.
Section 105(c)Provides that, if an employee receives payments through health or
accident insurance provided by an employer without tax cost to the employee for loss of
a member or function of the body or for disfigurement of the employee or the
employees spouse, or dependent, and if the amount is computed only with regard to

the nature of the injury and not to the period the employee is absent from work, the
amount is excludable from gross income.
**Only briefly discussed** Section 106(a)Section 106(a) excludes from an
employees gross income an employers contributions to accident and health plans set up
to pay compensation to employees for injuries or sickness. The exclusion under Section
106(a) relates, not to amounts paid to employees who are sick or injured, but to amounts
paid by employers for insurance premiums or into funded plans to set up benefits for
employees in case of future sickness or injuries. Fringe-BenefitSection 106(a)
provides a tax-free fringe benefit.
Tax Policy: Provides an economic incentive for employers to provide health insurance
for employees.

Problem 1 pg. 175


1. Plaintiff brought suit and successfully recovered in the following situations. Discuss
the tax consequences to Plaintiff.
(a) Plaintiff, a professional gymnast, lost the use of her leg after a psychotic fan assaulted
her with a tire iron. Plaintiff was awarded damages of $100,000. Not taxable.
(b) $50,000 of the recovery in (a) above is specifically allocated as compensation for
scheduled performances Plaintiff failed to make as a result of the injured leg. Personal
injury so not taxable.
(c) The jury also awards Plaintiff $200,000 in punitive damages. Taxable.
(d) The jury also awards Plaintiff damages of $200,000 to compensate for Plaintiffs
suicidal tendencies resulting from the loss of the use of her leg. Tied to the physical
injury so its excludable and tax free.
(e) Plaintiff in a separate suit recovered $100,000 of damages from a fan who
mercilessly taunted Plaintiff about her unnaturally high, squeaky voice, causing Plaintiff
extreme anxiety and distress. Taxable. Not a persona physical injury.
(f) Plaintiff recovered $200,000 in a suit of sexual harassment against her former coach.
Not taxable. Not a physical injury.
(g) Plaintiff dies as a result of the leg injury, and Plaintiffs parents recover $1,000,000
of punitive damages awarded in a wrongful death action under long-standing State
statute? Since it wrongful death and its the sole damages, they are tax-free. Its labeled
punitive damages but its actually a statutory recovery.

CH. 10 Separation and Divorce

A. ALIMONY AND SEPARATE MAINTENANCE PAYMENTS


-As a general rule: payments constituting alimony are income to the payee and
deductible by the payor. Can choose who the taxpayer is by designating something as
alimony.
Want the person who is in higher tax bracket to get the deduction and get the person in
the lower bracket to be taxed.
1. DIRECT PAYMENTS
In general, Section 71 applies to alimony and separate maintenance payments made
pursuant to a divorce or separate maintenance instrument. The instrument may be
a decree of divorce or of legal separation, a written instrument incident to such a decree, a
written separation agreement, or a decree for support.
Payments that qualify as alimony or separate maintenance are gross income to the
payee spouse under Section 71(a) and deductible by the payor spouse under Section
215(a).
Alimony = Paid After A Divorce, for support.
Separate Maintenance Payment = Paid during separation.
WHAT QUALIFIES AS ALIMONY? THE ELEMENTAL REQUIREMENTS
Payments made in cash qualify as alimony (or separate maintenance) so long as:
(1) The payment is received by or on behalf of the spouse under a divorce or separation
agreement.
(2) The divorce or separation agreement does not designate the payment as a nonalimony payment. *Means you can designate the tax treatment.
(3) In the case of a decree of legal separation of divorce, the parties are not members of
the same household at the time the payment is made.
(4) There is no liability to make any payment in cash or property, after the death of the
payee spouse.
(5) The payment is not child support.
Generally:
Section 71, Alimony Received Is IncomeAlimony or separate maintenance is gross
income to the payee spouse.

Section 215, Deduction for Alimony PaidAlimony or separate maintenance is


deductible by the payor spouse.
Example: Payor spouse earns $100,000. He is obligated to remit to Payee spouse $25,000
in alimony. Payor may deduct the $25,000 alimony payment in determining his Adjusted
Gross Income. The $25,000 alimony payment is income to the payee spouse.

Alimony Recapture
***WONTBE ASKED TO DO ON MIDTERM
When front-loaded alimony payments reflect a disguised property settlement, alimony
recapture functions to reverse the effect of payor deduction and payee inclusion by
assigning the payee a deduction and the payor inclusion.
The amount recaptured, which occurs in the third taxable year of the payments is the sum
of the YEAR 2 RECAPTURE and the YEAR 1 RECAPTURE, where:
YEAR 2 RECAPTURE = YEAR 2 PAYMENTS YEAR 3 PAYMENTS - $15,000
YEAR 1 RECAPTURE = [(YEAR 2 PAYMENTS YEAR 2 RECAPTURE) + YEAR 1
PAYMENTS]/2 - $15,000.

Problem 1 pg 184-185
1. Determine whether the following payments are accorded alimony or separate
maintenance status and therefore are includible in the recipients gross income under
71(a) and deductible by the payor under 215(a). Unless otherwise stated, Madonna and
Guy are divorced and payments are called for by the divorce decree.
(a) The divorce decree directs Madonna to make payments of $10,000 per year to Guy
for his life or until he remarries. Madonna makes a $10,000 cash payment to Guy in the
current year. Gets the alimony status.
(b) Same as (a) above except that Madonna, finding herself short on cash during the
year, transfers her $10,000 promissory note to Guy. Not cash so doesnt get the status.
(c) Same as (b) above except that instead of transferring her promissory note to Guy,
Madonna transfers a piece of art work, having a fair market value of $10,000. Not cash
so doesnt get the alimony status.
(d) Same as (a) above except that in addition the decree provide that the payments are
nondeductible by Madonna and are excludible from Guys gross income. No alimony
status because they chose non-alimony treated,
(e) Would it make any difference in (d) above if you learned that Madonna anticipated
that she would have little or no taxable income in the immediate future, making the 215

deduction practically worthless to her, and as a consequence of this agreed to the


nondeductibility provision in order to enable Guy to avoid the imposition of federal
income taxes on the payments? Not alimony status because it gives you the opportunity
to do pick your tax treatment.
(f) What result in (a) above if the divorce decree directs Madonna to pay $10,000 cash
each year to Guy for a period of 10 years? Doesnt qualify because it is possible that
there are rights to payment after death.
(g) Same as (f) above except that under local law Madonna is not required to make any
post-death payments. Qualifies because youre not making payments after death.
(h) Same as (a) above except the divorce decree directs Madonna to pay $10,000 cash
each year to Guy for a period of 10 years or his life, whichever ends sooner. Additionally,
the decree requires Madonna to pay $15,000 cash each year to Guy or his estate for a
period of 10 years. Madonna makes a $25,000 cash payment to Guy in the year.
-Only the 10k qualifies as alimony. The 15k doesnt qualify.
(i) Same as (a) above except that at the time of the payment, Madonna and Guy are
living in the same house. Doesnt qualify because they are still living together.
(j) Same as (i) above except that Madonna and Guy are not divorced or legally separated
and the payments are made pursuant to a written separation agreement instead of a
divorce decree. Qualifies as alimony. Living together only applies for a legal divorce or
separation agreement.
2. INDIRECT PAYMENTS
Payments for the benefit of the surviving spouse may qualify as alimony

I.T 4001
-Life insurance premiums are includable by wife and deductible by husdband. Example
of indirect payment that is still treated as alimony.

Problem 1 pg. 188


1. Tom and Nicole are divorced. Pursuant to their written separation agreement
incorporated in the divorce decree, Tom is required to make the following alternative
payments, which satisfy the 71(b) requirements. Discuss the tax consequences to both
Tom and Nicole.
(a) Rental payments of $1,000 per month to Nicoles landlord. Qualifies, indirect
payment of her lodging expenses. Paying rent, tuition, grocery bill, all qualify.

(b) Mortgage payments of $1,000 per month on their family home which is transferred
outright to Nicole in the divorce proceedings. Qualifies because it is made for her
benefit.
(c) Mortgage payments of $1,000 per month as well as real estate taxes and upkeep
expenses on the house where Nicole is living which is owned by Tom. Doesnt qualify
as alimony. Not just for the benefit of the spouse, also for the benefit of himself.

B. Property Settlements-Property settlements, being transfers between spouses in connection with a divorce,
should be tax-neutral.
Section 1041, Transfers Incident to Divorce (or between Spouses)with respect to
any transfer of property between married persons or once-married persons, any gain is
not recognized. Basis carries over from one party to the other party.
Section 1041 provides a clear non-recognition rule for gains and losses with respect to
any transfer of property between married persons and also between formerly married
persons.
Caveat: However, in regards to transactions between formerly married persons, 1041
only applies when the transfer is incident to divorce.
Section 1041(c), Transfers Incident to DivorceA transfer of property is incident to
divorce if the transfer
(1) Occurs within 1 year after the date on which the marriage ceases, or
(2) Is related to the cessation of the marriage.
Note: If the transfer occurs within 1 year after the marriage ceases, it is automatically
assumed to be incident to divorce, if it occurs after, then it must be related to the
cessation of the marriage.
Young v. Commissioner, 4th Cir. (2001)
Mr. & Mrs. Young were divorced in 1988. They entered into a Settlement Agreement in
1989 which provided that Mrs. Young was to be delivered a promissory note secured by a
deed of trust on 71 acres of property that belonged to Mr. Young under the terms of the
agreement. In 1990, Mr. Young defaulted on his obligation on the note. Mrs. Young
brought a state court action to collect. In 1991, Mrs. Young was awarded a judgment. Mr.
Young only paid a portion of the judgment, so Mrs. Young commenced steps to execute
the judgment. However, before execution of the judgment Mr. and Mrs. Young entered
into a Settlement Agreement under which Mr. Young would transfer 59 acres to Mrs.
Young. Mr. Young retained an option to repurchase. Mr. Young assigned the option to a
third party, who exercised it and bought the property for $2.2 million.
The Issue: What are the tax consequences.
1. Of the transfer between Mr. Young and Mrs. Young?

Even though it is beyond the one year safe harbor, this is clearly a transaction
incident to divorce. The sole reason for the 1992 Settlement was to resolve
disputes that arose from the divorce and subsequent property settlement, therefore
it was related to the cessation of the marriage. This was a 1041 tax neutral
transaction.
2. Of the transaction between Mrs. Young & the Third Party?
Taxable event, income to Mrs. Young.

Problem 1 pg. 197


1. Brad and Jens divorce decree becomes final on January 1 of year one. Discuss the tax
consequences of the following transactions to both Brad and Jen:
(a) Pursuant to their divorce decree, Brad transfers to Jen in March of year one a parcel
of unimproved land he purchased 10 years ago. The land has a basis of $100,000 and a
fair market value of $500,000. Jen sells the land in April of year one for $600,000. No
gain or loss for Brad due to non-recognition rule. Jen has a gain of 500k, basis of 100k
and 600k realized.
(b) Same as (a) above except that the land is transferred to satisfy a debt that Brad owes
Jen. The land has a basis of $500,000 and a fair market value of $400,000 at the time of
the transfer. Jen sells the land for $350,000. Brad has no gain or loss due to nonrecognition rule. Jen has a basis of 500k, has a 150k loss.
(c) What result if pursuant to the divorce decree, Brad transfers the land in (a) above to
Jen in March of year four. Same as above as long as incident to the divorce, still look at
the facts and circumstances.
(d) Same as (c) above except that the transfer is required by a written instrument incident
to the divorce decree. Same.
(e) Same as (c) above except the transfer is made in March of year seven. No change,
so probably the same. You start to question more as the years go on. Look at the facts.

C. Other Tax Aspects of Divorce


1. Child Support
-Child support is distinguishable from alimony and has tax-neutral treatment (not
includible/deductible).
Section 71(c), Child SupportGross income does not include any part of any payment
which the terms of the divorce or separation instrument fix as a sum which is payable fo
the support of children of the payor spouse.

Notes:
There is no income involved, not to the payee spouse, not to the child. There is no
deduction for the payor spouse.
If any amount specified in a divorce settlement agreement will be reduced on a
child attaining a specific age, marrying, dying, etc., the amount of the reduction
will be treated as an amount fixed for the support of a child, and the balance will
be treated as alimony.
Problems 1. Pg. 198
1. Kris and Kim enter into a written support agreement, which is incorporated into their
divorce decree at the time of their divorce. They have one child who is in Kims custody.
Discuss the tax consequences in the following alternative situations:
(a) The agreement requires Kris to pay Kim $10,000 per year and it provides that $4,000
of the $10,000 is for the support of their child. -6k is treated as alimony. The 4k is tax
neutral.
(b) The agreement requires Kris to pay Kim $10,000 per year, but when their child
reaches age twenty-one, dies, or marries prior to reaching twenty-one, the amount is to be
reduced to $6,000 per year. -6k is alimony, 4k is tax neutral.
(c) The agreement requires Kris to pay Kim $10,000 per year but that the payments will
be reduced to $8,000 per year on January 1, 2008 and to $6,000 per year on January 1,
2012. Kris and Kim have two children: Daughter (born June 17, 1990) and Son (born
March 5, 1993). The amount being reduced is child support, so 4k. The base amount of
6k is alimony.
(d) What result in (a) above if Kris pays Kim only $5,000 of the $10,000 obligation in
the current year? -4k is child support, the balance to alimony, the other 5k is unpaid
alimony.
2. ALIMONY PAYMENTS MADE BY A THIRD PARTY
Annuity payments
Payments from an annuity, even an annuity purchased in connection with a divorce, are
taxable as discussed earlier.
Alimony Trusts
Income from alimony trusts is includable in the income of the payee. No need to deduct
from the income of the payor, because the income was excluded to begin with.
3. DIVORCE
Rules relating to inclusion and deductibility of alimony payments apply only
when there is a divorce. State law generally determines the legal status of parties
as married, separated, or divorced

Ch. 11 Other Exclusions from Gross Income


A. Gain From the Sale of a Principle Residence
As a general rule, each individual taxpayer may exclude gain up to $250,000
realized on the sale of a primary residence, once every two years.
Section 121, Exclusion of Gain From Sale of Principal ResidenceSection 121
permits exclusion of up to $250k of gain on the sale or exchange of a taxpayers principal
residence. $500k for married couples.
Frequency LimitationCannot be exercised in whole more frequently than once every
two years.
Eligibility RequirementsTo be eligible for the exclusion, a taxpayer must have owned
the residence and occupied it as a principal residence for at least two of the five years
prior to the sale or exchange.
Change of Circumstances ExceptionTaxpayers who fail to meet these requirements
by reason of a change of place of employment, health, or unforeseen circumstances is
able to exclude the fraction of the $250k (or 500k, if married) equal to the fraction of two
years that these requirements are met.
Term Residence Defined BroadlyThe term residence is defined broadly to include
not only a house, but a house trailer, a house boat, stock in a cooperative housing unit,
and any other dwelling place. In fact, a residence can include surrounding vast acreage,
so long as it is not used for business or profit.
Special Rules apply to:
a. Nonqualifying uses (i.e. rental properties, vacation homes, etc.)a taxpayer is
prevented from excluding gain allocated to periods of nonqualified use.
b. Deceased and Divorced SpousesIn the case of a divorce, the gain and the
exemption are split between the spouses. If the gain exceeds the exclusion, then
the taxable value is divided between the spouses, unless otherwise agreed to in a
settlement.
c. Certain Military Personnel and Govt workers.
CLASS NOTES:
KEY Requirements for Exclusion of Gain on Sale of Principle Residence
The term residence is broad.
Exclusion applies only to principal residence.
Ownership and use tests must be met; does not apply to periods of nonqualified use.

Applies only once every two years, unless an exception applies allowing fractional
exclusion.
Problems pg. 210
1. Determine the amount of gain that Taxpayers (a married couple filing a joint return)
must include in gross income in the following situations:
(a) Taxpayers sold their principal residence for $600,000. They had purchased the
residence several years ago for $200,000 and living in it over those years. -They have a
gain of 400k and there is a 500k exclusion since they are married. So they arent taxed on
it.
(b) Taxpayers in (a) above purchased another principal residence for $600,000 and sold it
2 years later for $1 million. Same (a). They met the two-year requirement.
(c) What result in (b) above if the second sale occurred 1 years later? 400k gain and is
taxable because they havent lived there long enough, and no exception applies.
(d) [Skip]
(e) What result in (a) above if the residence was Taxpayers summer home, which they
used three months of the year? 400k gain and is taxable. Only applies to the primary
residence.
(f) What result if Taxpayer who met the ownership and use requirements is a single
taxpayer who just sold a principal residence for $400,000 and it had an adjusted basis of
$190,000 after Taxpayer validly took $10,000 of post-1997 depreciation deductions on
the residence, which served as an office in Taxpayers home? -10k is income, but can
exclude the 200k. **Check the answer

B. Income Earned Abroad


Subject to certain limitations, income earned while living in a foreign country, in addition
to a limited housing allowance, may be excluded from gross income.
Primary reason for exclusion is expectation that the hosting country will be assessing tax.
Section 911, Income Earned by Citizens/Residents of the United States Living
AbroadPermits an $80,000 exclusion, indexed for inflation, for foreign income earned
for the performance of services.
Not Excluded: Pensions, Foreign Trusts, Payments by the U.S. Government or its
agencies, or amounts beyond a taxable year for services rendered in said taxable year.
Requirements for Eligibility:
o American Citizen is a Bona Fide resident of a foreign country or countries for an
uninterrupted period that includes an entire taxable year OR American Citizen or

Resident is present in a foreign country or countries for at least 330 days during a
period of twelve consecutive months.
Foreign Housing Costs: Section 911 provides an exclusion for amounts paid as
reimbursement of foreign housing expenses if the expenses are paid by the taxpayers
employer. Alternatively, if a taxpayers housing costs are not paid by their employer, they
may take a deduction for a certain limited amount of those expenses.
-Housing expenses include reasonable amounts paid for housing, including utilities.
C. Exclusions And Other Tax Benefits Related To Costs of Higher Education
The code includes a collection of tax benefits for costs of higher education, including:
117 scholarship exclusion
108(f) student loan forgiveness exclusion
162 limited educational expense deduction
132 education as working condition fringe
221 interest deduction for education loans
222 deduction for certain tuition expenses
25A Hope Scholarship and Lifetime Learning Credits
135 exclusion of bond income used for education
529 tax-advantaged college savings plans *Wont have to know for midterm
530 Coverdell educational savings accounts *Wont have to know for midterm.

25A: Hope and Lifetime Learning Credits


Credits directly reduce taxable liability, not just the amount subject to tax like deductions.
Required for both Hope and Lifetime Learning Credits:
-For qualified tuition and related fees only.
-Must be at an educational institution.
Hope:
-Per student credit of up to $2500 (the first $2000 plus 25% of the amount
between $2000 and $4000). Unless you spend less the $2500, then its just what
you spent.
-Only allowed for the first four years of post-secondary education.

Lifetime Learning: (alternative to the hope)


-Per taxpayer credit of up to $2000 (20% of first $10000 of expenses). If you
spend less than $2000, then its what you spend.
-Allowed for any year of post-secondary education.
-Taxpayer cannot take both Hope Credit and Lifetime Learning Credit (so if eligible for
both, taxpayer takes Hope.
Section 135: Savings Bond Income Used to Pay Higher Education and Fees:
General rule--In the case of an individual who pays qualified higher education
expenses during the taxable year, no amount shall be includible in gross income
by reason of the redemption during such year of any qualified United States
savings bond.
(b) Limitations.-(1) Limitation where redemption proceeds exceed higher education expenses
(2) Limitation based on modified adjusted gross income
**DONT have to know for exam*** Section 529, Qualified Tuition Programs
provides that qualifying programs are tax-exempt.
The Requirements:
The plans must provide funds for qualified higher education expenses. These
expenses include tuition, fees, books, supplies, equipment, room, and board; in
connection with enrollment or attendance at an educational institution.
o Note: Here living expenses (room and board) are excludable, whereas
under a qualified scholarship, room and board are not excludable from
income.
o But, room and board are qualified expenses only if the student is more
than half-time.
Only cash contributions may be made to such plan.
The Tax Consequences:

Earnings of the plan are not taxed as they are earned.


Distributions are not taxable to the extent that they are used to fund qualified
higher educational expenses of the beneficiary.
Excess distributions are taxable to the beneficiary using Section 72 annuity
principles, excluding prior contributions and taxing the income element

Section 530 Coverdell Education Savings Accounts **dont have to know for
exam**

Problems pg. 219


Law Student and Spouse are self-supporting and Spouse works while Student
attends Law School. Consider the amount of any Section
25A
credit they may elect in the following circumstances assuming
they
file a joint return:
(a) Student pays $10,000 in tuition for the current year.
Spouse works and Spouses have a modified adjusted gross
income of $30,000 for the year.
Since Student is in law school and the year is not one of Student's first two years of
higher education, no Hope Scholarship credit is allowed. 25A(b)(2)(C). And since
Student is beyond four years of college, no American Opportunity Tax credit is available.
25A(i).
However, a nonrefundable Lifetime Learning credit of $2,000 would be allowed (20% of
expenses up to $10,000). 25A(c)(1). There would be no phase-out of the credit. See
25A(d).
Taking the problem a step further, if Spouses had $3,000 of 31 withholding on wages
and a $2,500 tax liability, their income tax refund would be increased from $500 to
$2,500 as a result of the credit.
(b) Same as (a) above except the tuition is paid from a student loan.
The fact that the tuition is paid from a student loan does not disallow the credit. Cf.
25A(g)(2). One also might want to mention the 221 interest deduction considered in
Chapter 16B.
(c) Same as (a) above except that Student is granted a $7,000 scholarship excluded
by 117 that reduces the tuition to $3,000.
The qualifying expenses are reduced by any 117 scholarship. 25A(g)(2)(A). Thus,
only $3,000 of expenses qualify and a $600 credit would be allowed (20% of $3,000).
(d) Same as (a) above except that Students Parents pay $8,000 of the tuition and
Student pays $2,000, although Student is not a dependent of Parents.
Parents' payment is a 102 gift and it does not reduce the amount of Student's qualifying
expenses. 25A(g)(2)(C) parenthetical. Thus, a $2,000 credit is still allowed.

(e) Same as (a) above except that Spouses have a modified adjusted gross income of
$100,000 for the year.
If Spouses have a modified adjusted gross income of $100,000, the credit is fully phased
out. 25A(d)(1) and (2). If the modified adjusted gross income had been $92,000, the
$2,000 potential credit would be reduced to $800, because $12,000/$20,000 or 60% of
the
credit
would
be
disallowed
under

25A(d)(1)
and
(2).
(f) What result in (a) above if, before credits, the Spouses has $3,000 of tax liability
and $3,000 of withholding from wages which qualifies for a potentially refundable
credit under 31?
As in problem 1(a), above, Spouses would qualify for a $2,000 25A credit. The credit
would reduce tax liability to $1,000 and result in a $2,000 refund under refundable 31.
See footnote 11 on page 213 of the Text.
(g) Same as (a) above, except that Spouse is also in college. Spouse is in the third
year of college and pays $5,000 in tuition and $500 for books.
Spouse would qualify for a $2,500 (American Opportunity Tax) credit $2,000 plus 25%
of $2,000. 25A(i)(1). Spouse would potentially qualify for the Lifetime Learning
credit but it would be only $2,000 and so the $2,500 credit would be preferable. If
25A(i) did not apply, there would only be a $2,000 credit.
The Lifetime Learning credit is a per taxpayer (not a per student) credit. ( 25A(c)(1))
and since both spouses are "the taxpayer" on a joint return, the results should be the same
as in (a), above, a $2,000 credit. Cf. 25A(d)(3) where "the taxpayer" would be the two
spouses filing a joint return; however, compare Reg. 1.151-1(b) where two personal
exemptions are allowed for the spouses filing a joint return.
(h) Same as (a) above, except that after Law Student graduates, Spouse, who has
not previously attended college, quits work and enters a vocational school as defined
in 25A(f)(2). Spouse pays $10,000 in tuition and Lawyer earns $75,000.
Spouses tuition would qualify for a $2,500 (American Opportunity Tax) credit.
25A(i).
If 25A(i) did not apply, Spouse's tuition would still qualify for a Hope Scholarship
credit since the costs are incurred within the first two years of Spouse's education.
25A(b)(2)(C). $1,500 of Spouse's first $2,000 will qualify for a Hope Scholarship credit.
25A(b)(1).
Federal Taxes and State Activities
The doctrine of intergovernmental immunity has bearing on the history and structure of
taxes imposed by the states on activities of the federal government and, to a more limited
extent, on taxes imposed by the federal government on activities by states.

Interest paid on state-issued (or municipality-issued) bonds is exempt from federal taxes
under certain circumstances.

*After midterm
Ch. 14 Business Deductions
A. INTRODUCTION
-Rather than assessing a tax against gross income, taxpayers are permitted to
deduct certain costs.
Taxpayers may report qualifying business deductions and thereby reduce the
amount of income that is subject to tax.

A taxpayer must show that they are entitled to a deduction.


Business deductions under 16d are Above-The-Line deductions because Schedule
C allows the deductions of costs before the net amount is included in gross
income.
Above the line is deduction is better because it makes your AGI lower.

B. ANATOMY OF THE BUSINESS DEDUCTION: SECTION 162


I.R.C. 162(a), Deduction for Trade or Business ExpensesThere shall be allowed as
a deduction all the ordinary and necessary expenses paid or incurred during the taxable
year in carrying on any trade or business, including
1. A reasonable allowance for salaries or other compensation for personal services
actually rendered;
2. Traveling expenses (including amounts expended for meals and lodging other
than amounts which are lavish or extravagant under the circumstances) while
away from home in the pursuit of a trade or business; and
3. Rentals or other payments required to be made as a condition to the continued
use or possession, for purposes of the trade of business, of property to which the
taxpayer has not taken or is not taking title or in which he has no equity.
*So the expense must be:

1. Ordinary.
2. Necessary.
3. A current expense (and not an investment that must be capitalized).
4. Paid or incurred in the taxable year.
5. For carrying on the trade or business.
[P.S. this is only the beginning the expense might otherwise be disallowed or
limited.]

IRC Reg. 1.162-Business Expenses


-Can get a business deductions that exceeds your income. Ex.)First year lawyer out on his
own.
Section 274: Disallowance of certain entertainment expenses.
1. ORDINARY AND NECESSARY
-To be deductible, a business expense must be both ordinary and necessary.
-To be ordinary, an expense must relate to a transaction that is common or frequent in
the type of business to which the payment relates.
-To be necessary, an expense must be appropriate and helpful for the development of
the business.
Welch v. Helvering, United States Supreme Court (1933)
Welch was the Secretary of the Welch Company, a firm that participated in the grain
industry. Welch filed for bankruptcy and the court discharged the firms debts. Prior to the
bankruptcy, Welch entered into a contract with the Kellogg Company to purchase grain
on commission. Despite the fact that the corporate debt had been discharged in
bankruptcy, Welch decided to pay the debts owed to Kellogg in order to repair his credit
and standing resulting from the bankruptcy. Welch attempted to deduct the expenses as
ordinary and necessary business expenses.
Held: The Supreme Court held that the expenses were not deductible as ordinary and
necessary business expenses.
NECESSARY: Necessary means appropriate and helpful.
ORDINARY: Ordinary, a strain of constancy, requiring common and accepted means.
Ordinary does not necessarily mean routine. But, it has to be the ordinary response in the
given situation. For instance, a company may only be sued once, but it is ordinary, when
that suit does come, to hire an attorney and expend resources in defense of the suit. A key
point here is differentiating ordinary expenses from capital expenditures. Ordinary relates

to the current costs of operation; whereas a capital expense benefits many years into the
future. Capital assets cannot be deducted, rather they are depreciated. So, when you
purchase a building, it is not an ordinary expense, it is a capital expenditure, as it benefits
the corporation many years into the future. Therefore, the initial purchase is not an
ordinary expense, however the amortization in subsequent years can be deducted
(depreciation.).
The Big Take Away from Welch Necessary = Appropriate & Helpful.
Ordinary = Current Cost of Operation.

2. EXPENSES
-To be deductible, the cost must be a current expense as opposed to an investment that
must be capitalized.
-When a cost is capitalized, the cost may be recoverable upon sale or disposition of the
asset (by increasing basis and lowering gain) or through depreciation deductions.
-From a taxpayers perspective, a deduction is preferable to capitalizing, because now is
better than later. Expense is now and capitalization is later.
Section 263(a), Capital ExpendituresA taxpayer may not deduct any amount paid for
new buildings or permanent improvements or betterments made to increase the value of
any property.
a.
b.
c.
d.

Tangible Property (land, building);


Intangible Property (stocks, bonds, etc.);
Cost of real estate broker/other brokerage commissions;
Costs of construction for new building, including architect fees, permits, and other
incidentals;
e. Costs of equipment, even if equipment would normally qualify for depreciation
deductions in the taxpayers business.
Treas. Regs 1.263-1 - 1.263(f)-1: Items Not Deductible
Repairs Vs. Improvements
Midland Empire Packing Co. v. Commissioner
-Midland Empire owned several underground storage rooms, which it used to cure ham
and bacon. After 25 years in use, oil from the neighboring refinery began to seep through
the walls into the basement, threatening the product. A federal inspector required Midland
to seal the walls. In response, Midland Empire added a concrete lining to the walls and
floor, the costs of which they deducted as an ordinary and necessary expense. The

Commissioner argued that the expenses were indeed necessary, but were not ordinary;
rather, the Commissioner contended that the expenses were a capital investment.
Held: The expenses were both necessary and ordinary. No expansion or improvement of
the property occurred. The basement functioned exactly the same way, both before and
after the addition of the seal. This was a mere repair, and therefore ordinary.
Amounts Paid to Acquire or Create Intangible Property
INDOPCO, Inc. v. Commissioner, United States Supreme Court (1992)
Unilever approached INDOPCO about a friendly takeover. INDOPCO hired an
investment banker and lawyer to appraise and guide the process. INDOPCO paid $2.25M
to the banker, $500K to the lawyers, and $150K in related expenses. The company
deducted the $2.25M as a business expense, subsequently the Commissioner asserted a
deficiency. INDOPCO counterclaimed, asserting that, in addition to the $2.25M, they
should also be able to take a deduction on the $500K and the $150K.
Result: This was a non-deductible capital expenditure. The transaction produced
significant benefits to INDOPCO that extended beyond the tax year in question.
-It had long-term value. Is it useful in the current year or something that adds value over
time?
Big Question: Is it now or is it later?
Problem 1 pg. 310
Landlord incurs the following expenses during the current year on a ten-unit apartment
complex. Is each expenditure a currently deductible repair on a capital expenditure?
a. $500 for painting three rooms of one of the apartments. Deductible
repair. Routine maintenance to keep in its normal working condition.
b. $4,000 for replacing the roof over an apartment. The roof had suffered
termite damage. Depends on how much of the roof. If its the whole
roof, then probably a capital expenditure. If it was just a portion then
probably a deductible repair.
c. $1,000 for patching the entire asphalt parking lot area. Probably a
normal repair to patch the parking lot area.
d. $3,000 for adding a carport to an apartment. Capitalization, it
improves the property as a whole. It adds value.
e. $100 for advertising for a tenant to occupy an empty apartment.
Advertising is an exception to the rule. They are consistently
deductible.
3. CARRYING ON BUSINESS

In order for the costs to be deductible, they must be incurred in the participation of the
business, and not just costs incurred in investigating participation in a new business.
Morton Frank v. Commissioner, Tax Court (1953)
Frank, a career newspaper man, sought to purchase a newspaper to run. They traveled
around the country (Pennsylvania, Ohio, Indiana, Michigan, Minnesota, Wisconsin,
Oklahoma, New Mexico, California, Texas, and Arizona). They eventually settled in
Phoenix, Arizona were Frank worked as an employee of the Arizona Times. He still
wished to purchase a newspaper, so he continued to make trips to find a paper to
purchase. Eventually, he purchased the Canton Repository. Mr. Frank deducted his travel,
telephone and other expenses expended in finding a paper to purchase.
Held: They cant take these deductions under 162(a), 162(a) requires carrying on a
trade or business, and Frank was spending money to find a trade or business, he was not
carrying on so there can be no deduction taken under 162(a).
START-UP COSTS:
General Rule: No deduction for startup costs, unless they are listed in 195.
Section 195, Start-Up Expenditures
a. 195 permits a taxpayer to elect to deduct up to $5,000 of start-up expenditures
in the year in which the business begins, reduced by the amount of start-up
expenditures that exceed $50,000 (thus phased out at $55,000).
b. The remaining start-up costs are amortized over a period of not less than 180
months from the month in which the business begins.
c. One must actually be successful in entering a trade or business to elect 195.
d. Start-up expenditures are only those incurred with respect to:
a. Investigating creation/acquisition of active trade or business;
b. Creating an active trade or business; or
c. Activities engaged in for profit before the day on which the active trade or
business begins, in anticipation of such activity becoming an active trade
or business.
e. The expenditures must be of the type that would be allowable as a deduction if
paid or incurred by an existing trade or business.
Problems on pg. 317
1. Determine the deductibility under 162 and 195 of expenses incurred in the following
situations:
(a) Tycoon, a doctor, unexpectedly inherited a sizeable amount of money from an
eccentric millionaire. Tycoon decided to invest a part of her fortune in the development
of industrial properties and she incurred expenses in making a preliminary investigation.
Not deductible under 162 because she is not actively carrying on business and she is
only looking into it. However, could be a start up expense under 195.

(b) The facts are the same as in (a) above, except that Tycoon, rather than having been a
doctor, was a successful developer of residential and shopping center properties.
Deductible under 162 because he is already carrying on the business.
(c) The facts are the same as in (b) above, except that Tycoon, desiring to diversify her
investments, incurs expenses in investigating the possibility of purchasing a professional
sports team. Not deductible under 162, sports teams are different than real property. but
maybe under 195.
(d) The facts are the same as in (c) above and Tycoon purchases a sports team. However,
after two years Tycoons fortunes turn sour and she sells the team at a loss. What
happens to the deferred investigation expenses? Yes under 195?

C. Specific Business Deduction


- Under the basic rule, section 162 allows as a deduction all the ordinary and necessary
expenses paid or incurred during the taxable year in carrying on any trade or business.
When do certain expenses like salaries, travel away from home, rental expenses, and
educational costs meet this requirement for deductibility?
1. REASONABLE SALARIES
-A taxpayer is permitted a deduction for a reasonable allowance for salaries or other
compensation for personal services actually rendered.
Treas. Reg 1.162-7: (a)There may be included among the ordinary and necessary
expenses paid or incurred in carrying on any trade or business a reasonable allowance for
salaries or other compensation for personal services actually rendered. The test of
deductibility in the case of compensation payments is whether they are reasonable and
are in fact payments purely for services.
(b) The test set forth in paragraph (a) of this section and its practical application may be
further stated and illustrated as follows:
(1) Any amount paid in the form of compensation, but not in fact as the purchase price of
services, is not deductible
(2) The form or method of fixing compensation is not decisive as to deductibility. While
any form of contingent compensation invites scrutiny as a possible distribution of
earnings of the enterprise, it does not follow that payments on a contingent basis are to be
treated fundamentally on any basis different from that applying to compensation at a flat
rate. Generally speaking, if contingent compensation is paid pursuant to a free bargain
between the employer and the individual made before the services are rendered, not
influenced by any consideration on the part of the employer other than that of securing on
fair and advantageous terms the services of the individual, it should be allowed as a
deduction even though in the actual working out of the contract it may prove to be greater
than the amount which would ordinarily be paid.

(3) In any event the allowance for the compensation paid may not exceed what is
reasonable under all the circumstances

NOTES: If really isnt payment for services then its not deductible. The form of
compensation is not decisive to determine if it is reasonable or not. Doesnt matter what
you call it. Are they disguised dividends? Contingent compensation is closely looked at
by IRS to make sure not disguised dividends.

Exacto Spring Corporation v. Commissioner, 7th Circuit (1999)In 1993 and 1994,
Exacto paid CEO $1.3M and $1.0M respectively. The company deducted these amounts
from its income tax returns. The IRS challenged the deductions, finding that CEOs
reasonable salary would have been only $381K and $400K respectively. The Tax Court
agreed that the salary was excessive, but reduced it only to $900K and $700K
respectively.
Held: CEOs salary was sufficiently related to the rate of return he generated. Judge
Posner embraced an independent investor test.
Independent Investor Test: An executives salary is reasonable so long as it is
reasonably related to the rate of return (adjusted for risk) that the executive can generate.
If the Commissioner can prove that a salary is inflated by dividends or that it does not
compensate for services, then it can nullify the deduction.
Harolds Club v. Commissioner, 9th Circuit (1965)From 1952-1956 Harolds Club, a
Nevada casino, paid Smith annual salaries ranging from $350k to $560k. Smiths sons
owned Harolds, and Smith was the brains of the operation. Profits during the period
ranged from $1.3M to $2M, and the sons received a salary of less than $100k.
Holding: The salary was unreasonable, the evidence failed to support the existence of a
free bargain between Smith and the owners (his sons). Smith was the father and head of
the family and he dominated both the business and the relationship with the sons. The
Court found that the control, which led to the absence of a free bargain, nullified the
deductions.

Section 162(m): Congress has imposed a $1 Million dollar ceiling on the amount of
compensation that a publicly held corporation may deduct in any year as remuneration
for services performed by a covered employee.
Problem 1 pg. 330
1.
Employee is the majority shareholder (248 of 250 outstanding shares) and
president of Corporation. Shortly after Corporation was incorporated, its Directors

adopted a resolution establishing a contingent compensation contract for Employee. The


plan provided for Corporation to pay Employee a nominal salary plus an annual bonus
based on a percentage of Corporations net income. In the early years of the plan,
payments to Employee averaged $50,000 annually. In recent years, Corporations profits
have increased substantially and, as a consequence, Employee has received payments
averaging more than $200,000 per year.
(a)
What are the Corporations possible alternative tax treatments for the payments?
Is it salary or dividend?
(b)
What factors should be considered in determining the proper tax treatment for the
payments? Reasonable or not.
(c)The problem assumes Employee always owned 248 of the Corporations 250 shares.
Might it be important to learn that the compensation contract was made at a time when
Employee held only 10 out of the 250 outstanding shares? -* Yes, a minority SH is more
likely to think it is compensation as opposed to dividends. It would make it appear as
though it was really a freely entered into contract at the time. Would have looked more
like an arms link contract.

2. TRAVEL AWAY FROM HOME


-Section 162 allows a deduction for traveling expenses (including amounts expended for
meals and lodging other than amounts which are lavish or extravagant under the
circumstances) while away from home in the pursuit of a trade or business. What is
included (and excluded) under this standard?
Rosenpan v. United States, 2nd Circuit (1971)Rosenpan was a traveling jewelry
salesman for New York based jewelers. He spent an average of 300 days a year on the
road, and when he did return to N.Y.C. he usually staid in hotels. Between 1948 and 1961
Rosenpan maintained a permanent address at his brothers residence in Brooklyn,
although he rarely stayed there. In 1961, he registered his automobile using a cousins
address in Ohio. But, in reality he just traveled, he had no home. For 1962-64 the I.R.S.
disallowed his extensive deductions for travel away from home, arguing that Rosenpan
had no home from which to be away from.
Holding: The deductions were disallowed. Home means home and Rosepan simply
hasnt got a home, therefore he was never away from home. To be eligible for the
deduction 3 conditions must be satisfied:
1. Expenses must be reasonable and necessary traveling expenses, as the term is
usually understood. Includes: transportation, food, and lodging.
2. Expense must be incurred while away from home.

3. Expense must be incurred in pursuit of business. Requires a direct connection


between the expenditure and the trade or business of the employer.
Andrews v. Commissioner, 1st Circuit (1991)Andrews lived in Mass., where he had a
swimming pool construction business. In 1974, Andrews started a horse breeding
operation in Florida. In 1983, Andrews purchased a home in Florida near his horse
breeding farm. He conducted substantial business at the Florida home and spent a part of
the year there. He took deductions on his expenses while in Florida. The Commissioner
protested, considering both the Mass. and Florida houses to be homes for tax purposes.
Held: The Florida expenses are deductible. The cost of maintaining the Florida home was
a duplicative expense. But, they were only deductible for the period of time he actually
spent there, the time in which he was using the property to produce income. *Only have a
one primary home, can deduct the house you were at home less which was connected to
business.

Revenue Ruling 99-7, Daily Transportation Expenses:


1. HomeBusiness: Costs of commuting between the taxpayers residence and his
place of business are not deductible personal expenses.
2. BusinessBusiness: Costs of commuting between the taxpayers business and
another business are deductible business expenses. If the taxpayer works out of
his/her home, then costs of commuting from the home/business to other
businesses may be deducted.
3. HomeTemp. Business Outside Area: A taxpayer may deduct daily
transportation expenses incurred in commuting between his residence and any
temporary work location outside the metro area where the taxpayer lives and
normally works.
a. Exception 1: Multiple Work Locations: If a taxpayer has one or more
regular work locations away from the taxpayers residence, the taxpayer
may deduct daily transportation expenses incurred in going between the
taxpayers residence and any temporary work location in the same trade or
business, regardless of the distance.
b. Exception 2: Home Office: If a taxpayers residence is the taxpayers
principal place of business within the meaning of the Code, the taxpayer
may deduct daily transportation expenses incurred in going between the
residence and any other work location in the same trade or business,
regardless of whether the other location is regular or temporary and
regardless of the distance.

4. TemporaryIf employment at a work location is realistically expected to last


(and does in fact last) for one year or less, the employment is temporary.
5. Not TemporaryIf employment at a work location is realistically expected to
last for more than one year or there is no realistic expectation that the work will
last for less than one year, the employment is not temporary, regardless of the
actual duration.
HybridIf work is initially expected only to last less than a year (i.e. be temporary) then
at some later point expectations change and it is reasonably expected to last for more than
one year (i.e. be not temporary), the employment is treated as temporary up to the date
where the expectations changed, and after that point it is treated as not temporary.
Problems pg. 346
1. Commuter owns a home in Suburb of City and drives to work in City each day. He
eats lunch in various restaurants in city.
a. May Commuter deduct his costs of transportation and/or meals? Not a deduction for
the transportation, and the meals generally not deductible, unless it was like a meeting or
something.
b. Same as (a) above, but Commuter is an attorney and often must travel between his
office and the City Court House to file papers, try cases, etc. May Commuter deduct all
or any of his costs of transportation and meals? Can deduct his transportation expenses,
but probably not the meals.
c. Commuter resides and woks in City, but occasionally must fly to Other City on
business for his employer. He eats lunch in Other City and returns home in the late
afternoon or early evening. May he deduct all or a part of his costs? All travel is
deductible. Meals are only deductible if your away from home long enough where sleep
or rest is required, so probably not.

3. NECESSARY RENTAL AND SIMILAR PAYMENTS


Section 162 allows a deduction for rentals or other payments required to be made as a
condition to the continued use or possession, for purposes of the trade or business, of
property to which the taxpayer has not taken or is not taking title or in which he has no
equity. What is included (and excluded) under this standard? When is a rental not really
a rental?
Starrs Estate v. Commissioner, 9th Circuit (1959)Sales couched as rentals are not
deductible. Automatic Sprinklers installed a fire system at Gross Manufacturing
Company, owned by Starr. Automatic leased the system to Gross for rent of $1240 for the
first 5 years and $32 for the 5 years after that.
Deductible? No. This was not really a rental, rather it was a non-deductible capital
expenditure disguised as a rental. You cant deduct this, rather you must treat it as a
purchase and depreciate it over its useful life

4. EXPENSES FOR EDUCATION


Expenses for education that are ordinary and necessary in connection with the taxpayers
business are deductible; personal educational expenses are not deductible under 162.
Generally personal expenses for education are non-deductible.
*Later in the course well learn that some of these deductions are allowable, they may be
limited by other sections.
Hill v. Commissioner, 4th Circuit (1950)Nora Hill was a Virginia schoolteacher. She
held the highest certification the State of Virginia gave to teachers. The certification was
set to expire. In order to renew it she was required to either take further coursework or
take an exam. She chose the coursework, and enrolled in summer courses at Columbia.
She deducted her expenses. The Commissioner came in and asserted a deficiency. The
Tax Court disallowed the deduction, characterizing it as a personal expense.
Holding: Reversed. This is a deductible business expense, ordinary and necessary and in
the course of carrying on a trade or business.
Coughlin v. Commissioner, 2nd Circuit (1953)Coughlin was a tax lawyer. In 1946,
Coughlin deducted $305 in travel and lodging expenses associated with his trip to a
Federal Taxation Seminar in Manhattan. The Commissioner and Tax Court said no to the
deduction.
Held: Deductible. Coughlin had a professional need to incur the expenses. Coughlin had
a business need to stay up to date on federal tax matters.
Problems pg. 358
1. Alice, Barbara, Cathy, and Denise were college roommates who after graduating went
on to become a doctor, a dentist, an accountant (CPA) and a lawyer, respectively. In the
current year, after some time in practice as an orthopedic surgeon, Alice, who was often
called upon to give medical testimony in malpractice suits, decided to go to law school so
as to better understand this aspect of her medical practice. Barbara enrolled in a course
of postgraduate study in orthodontics, intending to restrict her dental practice to that
specialty in the future. Cathy enrolled part time in law school (with eventual prospects of
attaining a degree) so as better to perform her accounting duties in areas in which law and
accounting tend to overlap. And Denise took a leave of absence from her firm to enroll in
an LL.M. course in taxation, intending to practice exclusively in the tax area. Which, if
any, is incurring deductible expenses of education?
-Alice: Not deductible, because it doesnt relate to her current trade or business, it would
be a new trade or business.
-Barbara: Yes, it is deductible. It relates to her current field of work and is refining on
specializing in her already chosen line of work.

-Cathy: Not deductible. It is a new career.


-Denise: The cases are split. Dirusso says it probably should be deductible.

D. Miscellaneous Business Deductions


1. INTRODUCTION
Common and important business expense deductions, which may have special rules,
include:
-Business Meals and Entertainment
-Uniforms
-Advertising
-Dues
-Lobbying Expenses
Business Meals And Entertainment
Business meals and entertainment are deductible only subject to the limitations that:

The meal or activity must be directly related to or associated with the


taxpayers trade or business.

The meal is not lavish or extravagant and the taxpayer (or employee of the
taxpayer) attends the meal.

Only 50% of the cost may be deducted.

Expenses related to entertainment facilities are strictly limited.

Substantiation of expenses is required. Keep good records/receipts.

(Theres more fine print and detail too, but these are the high points.)

Uniforms
Deductions relating to employee uniforms qualify only if:

The uniform is required as a condition of employment and

The uniform is not adaptable to general use (can take the place of regular
clothing).
Advertising
-Generally, advertising expenses of a business are deducible in the year in which they are
incurred or paid (even if the benefits extend over time).
However, an expenditure related to advertising may also be a capital expenditure, which
is not deductible. Example: The purchase of property or the construction of a billboard or
advertising sign is a non-deductible capital expenditure.

Dues
-Generally, dues paid to organizations directly related to ones business are deductible.
Examples: ABA, BBA etc
Lobbying Expenses
Generally, lobbying expenses are NOT deductible as a business expense.
Lobbying expenses include costs such as those incurred in influencing legislation,
participation in political campaigns, influencing the public with respect to elections or
legislative matters, or direct communications attempting to influence executive branch
officials.
*Exceptions: The limitation does not apply to (1) influencing legislation at the local level,
(2) in-house lobbying expenses of up to $2,000 per year, or (3) lobbying expenses
incurred by professional lobbyists directly on behalf of another person.

Problem pg. 365


1. Employee spends $100 taking 3 business clients to lunch at a local restaurant to
discuss a particular business matter. The $100 cost includes $5 in tax and $15 for a tip.
They each have two martinis before lunch.
(a) To what extent are Employees expenses deductible? 50% of $100 meal/tip/tax is
deductible, as long as its not lavish or extravagant.
(b) To what extent are the meals deductible if the lunch is merely to touch base with the
clients? Not directly related to a business purpose or matter.
(c) What result if Employee merely sends the three clients to lunch without going herself
but picks up their $75 tab? Not deductible, because there was no business rep there.
(d) What result in (a) above if, in addition, Employee incurs a $15 cab fare to transport
the clients to lunch? The transportation costs are deductible and not subject to the 50%
entertainment restriction. So it is fully deductible.
(e) What result in (a) above if Employer reimburses Employee for the $100 tab? *Check
this answer.
3. Airline Pilot incurs the following expenses in the current year:
(1) $250 for the cost of a new uniform. Deductible.
(2) $30 for dry cleaning the uniform. Ordinary and necessary to upkeep of uniform, so
deductible.

(3) $100 in newspaper ads to acquire a job as a property manager in his spare time. Not
deductible because not related to current trade or business.
(4) $200 in union dues. Deductible.
(5) $50 in political contributions to his local legislator who he hopes will push
legislation beneficial to airline pilots. Local mean state? So probably deductible.
(6) $500 in fees to a local gym to keep in physical shape for flying. Not deductible.
What is the total or Pilots deductible 162 expenses? $480.

2. BUSINESS LOSSES
Qualifying business losses may be deductible. Section 165(c)(1) permits the deduction
by an individual of any loss incurred in a trade or business.
Many other kinds of losses are disallowed may be experienced by the taxpayer, but not
recognized for tax purposes.
165. Losses (a) General rule.--There shall be allowed as a deduction any loss
sustained during the taxable year and not compensated for by insurance or otherwise.
(b) Amount of deduction.--For purposes of subsection (a), the basis for determining the
amount of the deduction for any loss shall be the adjusted basis provided in section 1011
for determining the loss from the sale or other disposition of property.
(c) Limitation on losses of individuals.--In the case of an individual, the deduction
under subsection (a) shall be limited to-(1) losses incurred in a trade or business;
(2) losses incurred in any transaction entered into for profit, though not connected
with a trade or business; and
(3) except as provided in subsection (h), losses of property not connected with a
trade or business or a transaction entered into for profit, if such losses arise from
fire, storm, shipwreck, or other casualty, or from theft
280B. Demolition of structures: In the case of the demolition of any structure--

(1) no deduction otherwise allowable under this chapter shall be allowed to the owner or
lessee of such structure for-(A) any amount expended for such demolition, or
(B) any loss sustained on account of such demolition; and
(2) amounts described in paragraph (1) shall be treated as properly chargeable to capital
account with respect to the land on which the demolished structure was located.
Problem pg. 368
1. Taxpayer has an automobile used exclusively in Taxpayers business, which was
purchased for $40,000 and, as a result of depreciation deductions, has an adjusted basis of
$22,000. When the automobile was worth $30,000, it was totally destroyed in an
accident and Taxpayer received $15,000 of insurance proceeds.
(a) What is Taxpayers deductible loss under Section 165? Car was used in trade or
business. Adjusted Basis (22k)-15k=7kLoss.
(b) What result in (a) above if the automobile had not been totally destroyed but was
worth $10,000 after the accident? Choose the lower of basis or change in value. The
change in value was 20k which is lower than the adjusted basis, so go with the lower of
20k. So 20k-15k=5k
(c) What is Taxpayers adjusted basis in the automobile in (b) above if Taxpayer incurs
$17,000 fixing the automobile? 2k basis increased by 17k basis son basis is now 19k.
*LEFT OFF HERE, CONCETATE ON E1.

E. DEPRECIATION
**Only responsible for the basic introduction.**
1. INTRODUCTION

Certain assets that are not eligible for current business deductions may
instead be eligible for depreciation deductions.

Depreciation deductions reflect the declining value of property and allow a


taxpayer to take into account his investment in property over time.

Depreciation deductions are calculated based on the taxpayers basis


(more on how later). Amounts taken as depreciation trigger an adjustment
in basis (the taxpayer must subtract the depreciation amount from the
basis), so that the taxpayer is not credited with return of capital twice.

Depreciation Basics:

Eligibility for Deduction: Only property used in a trade or business or


held for the production of income that is subject to exhaustion, wear, or
obsolescence is entitled to a depreciation deduction.

Useful Life: Assets are depreciated over the time they are expected to last
(or some time period estimated to be such or deliberately underestimated
to allow faster recovery of investment).

Depreciation Methods: Most depreciation methods are variations on


straight-line depreciation, where flat amounts are deducted each year, or
declining balance depreciation, where percentages of remaining basis are
deducted each year. Often you get to pick your method, but sometimes a
particular method is mandatory, like land. More on these methods later.

167. Depreciation
(a) General rule.--There shall be allowed as a depreciation deduction a
reasonable allowance for the exhaustion, wear and tear (including a reasonable
allowance for obsolescence)-(1) of property used in the trade or business, or
(2) of property held for the production of income.
(c) Basis for depreciation.-(1) In general.--The basis on which exhaustion, wear and tear, and
obsolescence are to be allowed in respect of any property shall be the adjusted
basis provided in section 1011, for the purpose of determining the gain on the sale
or other disposition of such property.
(2) Special rule for property subject to lease.--If any property is
acquired subject to a lease-(A) no portion of the adjusted basis shall be allocated to the leasehold
interest, and

(B) the entire adjusted basis shall be taken into account in determining the
depreciation deduction (if any) with respect to the property subject to the
lease.
Section 1016: Depreciation lowers your basis.

Sharp v. US

FACTS: Sharp and Sharp were business partners who bought an airplane costing
about $54k.

They used the plane 74% of the time for personal use, and 26% of the time
for business use.

For a few years, Sharp and Sharp claimed a deduction on their taxes for
depreciation of the airplane.

The total depreciation claimed was about $13.7k.

Finally, they sold the airplane for $35.3k. When they filed their taxes, they claimed a loss
on the sale. The IRS disagreed and found that they had made a gain on the sale.

Sharp and Sharp argued that their adjusted basis in the plane was $54k $13.7k = $40.5k. Since they sold the airplane for only $35.3k, they lost
$5k.

The IRS had an alternate method of calculating. They argued that


depreciation is only allowed on business expenses, which means that only
the 26% of the airplane that was used for business qualified.

So, the business part of the airplane was only worth 26% of the
$54k, or about $14.2k. When you subtract out the $13.7k of
depreciation, the adjusted basis of the business part of the airplane
was only $500.

When they sold the airplane, the business part was sold for $35k x
26% = $9300.

Therefore, there was a gain of $9300 - $500 = $8800.

Since gains or losses on the personal use part of the airplane are
not deductible, the partnership owes taxes on a gain or $8800.

The Trial Court found for the IRS. Sharp and Sharp appealed.

The Trial Court found that Sharp's logic would result in non-uniformity
between taxes owed on properties used exclusively for business, and those
used for a combination of business and personal uses.

The Appellate Court affirmed.


CLAS NOTES: *Only exclusive to business use property.
depreicated.

They originally over

Simon v. Commissioner
The Simons, Petitioners, purchased two violin bows made in the 19th century by Francois
Tourte, a renowned bow maker. They were purchased in 1985 and were virtually unused
at that time. A violin bow will play out eventually and no longer be useful. However,
the Tourte bows retain value as collectors items. The bows at issue were appraised in
1990 at $45,000 and $35,000. At that time they had deteriorated since their purchase by
Petitioners. Petitioners paid $30,000 and $21,500 for the bows. Petitioners used the bows
for their trade as musicians and in 1989, the tax year at issue, they played in four concerts
per week. They claimed depreciation on the bows in the amount of $6,300 and $4,515.
The Tax Court allowed the deductions, and the Commissioner appealed.
Holding for Petitioners and finding that the bows did not have to have a determinable
useful life to qualify for a depreciation deduction, and were deductible.
The Court of Appeals found that the test is whether property will suffer exhaustion, wear
and tear, or obsolescence in its use by a business. The Court points out that the law was
intended by Congress to encourage economic in business property. The Commissioner
argued that businesses could use the law to purchase wasteful antiques. However, the
Court found that the test articulate should prevent abuse by businesses because it requires
demonstrable wear and tear in it business.
CLASS NOTES: Must have a property that is subject to wear and tear.

Problems Pg. 391


1. If the Share divided airplane was purchased in the current year and was used for
non-entertainment business purposes, would the facility limitation of 274(a)(1)(B) deny
the depreciation deduction?
-Not relevant here. Airplane is primarily used for transportation and 274 doesnt apply to
transportation.
2. Section 263(a) provides no deduction shall be allowed for any amount paid out for
new buildings Thus if one pays for salaries or for painting, etc. in construction of a
building, the expenses are capitalized and the total cost of the building is then depreciated
assuming the 167 or 168 requirements are met.

(a) If Company owns trucks that are used during the year exclusively in constructing a
new storage plant for the Company, is Company allowed a depreciation deduction for the
trucks during the year? Consider what tax alternative there might be and then see
Commissioner v. Idaho Power Co.
-No depreciation because they exclusively used in only construction. You capitalized
construction costs.
(b) Are interest on loans connected with property and taxes on the property which are
paid during the construction period currently deductible? See 263A.
-*Check answer. No depreciation

***E2 and E3 will not be tested*********


2. Special Depreciation Rules on Personal Property
3.
**Left off here.

CH. 15 DEDUCTIONS FOR PROFIT-MAKING, NONBUSINESS ACTIVITIES


A. SECTION 212 EXPENSES
*Like the deduction for business expenses, certain expenses incurred in connection with
the pursuit of income, although not part of a trade or business, are entitled to an
analogous deduction.
Section 212, Expenses for Production of Incomein the case of an individual, there
shall be allowed as a deduction all the ordinary and necessary expenses paid or incurred
during the taxable year
1. For the production or collection of income;
2. For the management, conservation, or maintenance of property held for the
production of income; or
3. In connection with the determination, collection, or refund of any tax.
*Treas. Reg. 1.212-1: It has to be taxable income. Has to be ordinary and necessary.
Certain fees are deductible, investment counsel, custodial fees, clerical help but must be
reasonable.
-Just trying to prove you own something is not deductible, so litigation over who
is the owner is not deductible. It is just a cost of obtaining the asset.

-Expenses incurred for tax counsel or tax preparation are deductible.


-Expenses in defending lawsuits are not deductible.
Higgins v. Commissioner, United States Supreme Court (1941)
Higgins was a wealthy investor. He lived in Paris, France and maintained an office in
New York, complete with staff, to supervise his extensive portfolio of real estate and
securities. In 1932 and 1933, he deducted the salaries and expenses of these operations as
business expense ( 162). The Commissioner denied the deduction, asserting that Higgins
was not carrying on a trade or business.
Holding: The Supreme Court agreed with the Commissioner. Higgins was not eligible to
deduct the expenses because the management of his investments did not constitute
carrying on a trade or business.
**THIS is an example of why we now have 212.
Bowers v. Lumpkin, 4th Circuit (1944)Mrs. Lumpkin had a life interest under a trust,
created by her now deceased husband, in one-half of the stock of a corporation that
owned valuable rights in the sale and distribution of Coca-Cola syrup in the State of
South Carolina. She purchased the remaining stock for $255k from trustees to whom it
has been bequeathed to establish and maintain an orphanage. The South Carolina
Attorney General sued to invalidate the sale; LUMPKIN WON. In connection with the
litigation, Lumpkin incurred nearly $27k in expenses, which she deducted from her gross
income under 212. The Commissioner vigorously protested, disallowing the deduction
and asserting a deficiency against Mrs. Lumpkin.
Holding: She could not deduct her expenses under 212. Defending the lawsuit
involving her stock was not managing, conserving, or maintaining property held for the
production of income. Mrs. Lumpkin was merely litigating over the right to title to a
piece of property.
CLASS NOTES: Has to be an expense for the production of income, not to prove your
rights to property.
Surasky v. United States, 5th Circuit (1963)Mr. Surasky purchased 4,000 shares of
Montgomery Ward, for nearly $300k. He, along with several of his fellow stockholders,
formed a committee to revamp the ailing, struggling company, fronting $17k in expenses
for the committee in tax year 1955. The committee successfully won three seats on the
board of directors and toppled the Chairman and the President of the Company. In the
year following, profits soared. Surasky sold his stock for a $50k profit after taking home
$30k in dividends. Suraskey sued to recover the $17k deduction on his 1955 taxes. The
District Court said no, because the expenses were not sufficiently related to production of
income.
Held: The statute does not require proximate cause between the expense and a return of
income. It was sufficient that Surasky expended the money with anticipation that profit
would return.

CLASS NOTES: Spent money on new leadership, to earn more money.


Revenue Ruling 64-236: The Service holds that expenditures are deductible by a
stockholder under Section 212 if such expenditures are proximately related to either the
production or collection of income or the management, conservation, or maintenance of
property held for the production of income. This largely aligns with Surasky somewhat,
but not to the extent that Surasky did not require the proximate relationship.
*Proxy fights are potentially, but youre going to have to show that relationship.

Meyer J. Fleischman v. Commissioner, Tax Court (1966)Meyer and his wife Joan had
an ante-nuptial agreement, under which upon divorce Meyer would pay Joan $5k to
release all claims to Meyers property. Joan filed for divorce seeking division of property.
While the suit was pending, she filed a separate action to invalidate the ante-nuptial
agreement. Meyer prevailed on the invalidation suit, and the divorce settled.
Procedure: Meyer did not deduct the expenses of the divorce proceedings (good, cause
they are not deductible, they are personal expenses) but he did deduct expenses for
defending the invalidation suit. The Commissioner asserted a deficiency.
Held: No deduction can be allowed here. The invalidation suit arose entirely from a
personal issue. A suit against a taxpayer must be directly connected with or proximately
result from his business before it is a business/profit-making expense.
CLASS NOTES: Personal costs are not deductible.
Problems 1,3 pg. 425
1. Speculator buys 100 shares of Sound Company stock for $3,000, paying her broker a
commission of $50 on the purchase. Fourteen months later she sells the shares for $4,000
paying a commission of $60 on the sale.
a.) She would like to treat $110 paid as commissions as 212 expenses. Why? Can
she? Not deductible because the commissions are part of the cost.
b.) What result in (a) above if instead she sells the shares for $2500 paying a $45
commission on the sale? See 165(c)(2). Doesnt matter that it was a gain or loss
so same answer.
c.) Speculator owned only one-tenth of one percent of the Sound Company stock
(worth $3000) but being an eager investor during the time she owned the stock,
she incurred $500 of transportation, meals, and lodging expenses in traveling
1000 miles to New York City to attend Sounds annual shareholder meeting. May
she deduct her travel under 212(2)? No it is not ordinary for someone with such a
small stake in the company to do. Not deductible.

d.) What result in (c) above if instead Speculator owned 10% of the total
outstanding Sound stock, worth $300,000? It is deductible, he could influence
the company with that significant amount.
e.) What result to Speculator if she incurred the expense in (c) above to attend a
seminar on investments? Not deductible. Educational expenses are personal.
f.) Speculator owns 10% of Sounds stock worth $300,000 and she incurs $10,000
in legal fees and personal costs investigating the operation of the business after
the business has some serious set-backs. Is the $10,000 deductible? Maybe.
Probably need more facts. Is it likely to produce more return? Ordinary and
necessary?
3. Planner consults his attorneys with respect to his estate plan. They decide to make
various inter vivos gifts and draft his will. To what extent, if any, are Planners legal fees
deductible under 212(3)? Legal fees are deductible for determining tax consequences but
not deductible to not to anything unrelated to tax.

B. CHARGES ARISING OUT OF TRANSACTIOND ENTERED INTO FOR


PROFIT
To what extent are charges, costs, and losses taken into account?
IRC 165: Losses: *Get this from slide.
IRC 167: Depreciation: deductions for the exhaustion wear and tear, for property
held for the production of income.
Treas Reg. 1.212-1 Nontrade or non-business expense: If its your own place or own
home you cant take deductions. If it is your summer home or something and you rent it
and you USED to live there, can deduct the expenses.
William C. Horrman v. Commissioner, Tax Court (1951)Horrman inherited a house
from his mother in 1940 worth $60k. He redecorated the house and used it as his
principal place of residence until 1942, at which time he abandoned the house. He
attempted to rent the house, but to no avail. In 1945, he sold the house for $45k.
Issues: The Tax Court took up three issues:
(i)
(ii)

Deduction for deprecation on the property in 1943, 44, and 45.


Deduction for expenses incurred in maintenance and conservation of the
property.
(iii)
Deduction for loss on the sale of the property.
Holding:
(i)

Horrman was entitled to the deductions for depreciation, because the property was
held for the production of income. His efforts to rent the property, albeit
unsuccessful, were sufficient.

(ii)
(iii)

Horrman was entitled to a deduction for the expenses incurred in maintaining and
conserving the property for production of income. The improvements he made
were designed to make the property more attractive to potential renters.
Horrman was not entitled to a deduction for the loss on the sale of the property
because no facts supported a shift from (1) his holding the property for the
production of income to (2) a transaction entered into for profit. Simply putting
the house on the market doesnt support the designation; he had the ability to pull
it off the market at any time. A more substantial, definitive event would be
required.

Lowry v. United States, District Court for the District of New Hampshire (1974)
Lowry put up his vacation home for sale and unsuccessfully attempted to rent it while it
remained on the market for several years. Lowry deducted the expenses of caring for and
maintaining the residence. The Service disallowed the deduction and levied a deficiency.
Lawry paid and sued for a refund.
Held: Refund granted, Lowry was entitled to the deduction. The Court held that the facts
supported that Lowry abandoned the home as a residence and transformed the property
into one held for the production of income. The statute permitting a deduction for the care
and maintenance of such property does not require that it actually produce any income,
rather, it requires a factual determination of how the owners regarded and treated the
property. In this case, the court found that it was clear that Lowry was holding the
property for the production of profit.
*Can convert personal property to profit seeking property.

Problem 1 pg. 433


1. Recall the Frank case in Chapter 14 at pg.311 supra.
a.) Should Franks expenses have been deductible under 212 or 165(c)(2)? Not
deductible under either. 212 he has to have an existing interest. 165 expenses in a
preliminary investigation are included. He didnt buy anything.
b.) If Frank had decided to buy the newspaper and incurred capital expenditures to begin
operations, but then abandoned his plans, would he have been allowed a deduction? Yes.
If you do enter into the transaction seeking profit and you receive a loss, then you can
recognize a loss.
c.) If Frank entered the business and elected to use 195 but ceased operations before
deducting all of his start-up expenses, to what extent could he take a 165(c) loss?
*Check answer

CH. 16 DEDUCTIONS NOT LIMITED TO BUSINESS


OR PROFIT SEEKING ACTIVITIES
A. INTRODUCTION.

B. INTEREST
-To what extent are payments of interest deductible? Sometimes but not always.
IRC 163. INTEREST
(a) General rule.--There shall be allowed as a deduction all interest paid or accrued
within the taxable year on indebtedness.
(h) Disallowance of deduction for personal interest.-(1) In general.--In the case of a taxpayer other than a corporation, no deduction
shall be allowed under this chapter for personal interest paid or accrued during the
taxable year.
(2) Personal interest. --For purposes of this subsection, the term personal
interest means any interest allowable as a deduction under this chapter other than
(What is deductible:)
(A) interest paid or accrued on indebtedness properly allocable to a trade
or business (other than the trade or business of performing services as an
employee),
(B) any investment interest (within the meaning of subsection (d) ),
(C) any interest which is taken into account under section 469 in
computing income or loss from a passive activity of the taxpayer,
(D) any qualified residence interest (within the meaning of paragraph (3)),
*This is the biggie. Whether you own a house or not.
(E) any interest payable under section 6601 on any unpaid portion of the
tax imposed by section 2001 for the period during which an extension of
time for payment of such tax is in effect under section 6163, and
(F) any interest allowable as a deduction under section 221 (relating to
interest on educational loans).
(3) Qualified residence interest.--For purposes of this subsection--

(A) In general.--The term qualified residence interest means any


interest which is paid or accrued during the taxable year on-(i) acquisition indebtedness with respect to any qualified residence
of the taxpayer, or
(ii) home equity indebtedness with respect to any qualified
residence of the taxpayer.
For purposes of the preceding sentence, the determination of
whether any property is a qualified residence of the taxpayer shall
be made as of the time the interest is accrued.
(B) Acquisition indebtedness.-(i) In general.--The term acquisition indebtedness means any
indebtedness which-(I) is incurred in acquiring, constructing, or substantially
improving any qualified residence of the taxpayer, and
(II) is secured by such residence.
Such term also includes any indebtedness secured by such
residence resulting from the refinancing of indebtedness
meeting the requirements of the preceding sentence (or this
sentence); but only to the extent the amount of the
indebtedness resulting from such refinancing does not
exceed the amount of the refinanced indebtedness.
(ii) $1,000,000 Limitation.--The aggregate amount treated as
acquisition indebtedness for any period shall not exceed
$1,000,000 ($500,000 in the case of a married individual filing a
separate return).
(C) Home equity indebtedness.-(i) In general.--The term home equity indebtedness means any
indebtedness (other than acquisition indebtedness) secured by a
qualified residence to the extent the aggregate amount of such
indebtedness does not exceed-(I) the fair market value of such qualified residence,
reduced by
(II) the amount of acquisition indebtedness with respect to
such residence.

(ii) Limitation.--The aggregate amount treated as home equity


indebtedness for any period shall not exceed $100,000 ($50,000 in the
case of a separate return by a married individual).
(4) Other definitions and special rules.--For purposes of this subsection-(A) Qualified residence.-(i) In general.--The term qualified residence means-(I) the principal residence (within the meaning of section 121) of the
taxpayer, and
(II) 1 other residence of the taxpayer which is selected by the taxpayer for
purposes of this subsection for the taxable year and which is used by the
taxpayer as a residence (within the meaning of section 280A(d)(1)).
(ii) Married individuals filing separate returns.--If a married couple does not
file a joint return for the taxable year-(I) such couple shall be treated as 1 taxpayer for purposes of clause
(i), and
(II) each individual shall be entitled to take into account 1
residence unless both individuals consent in writing to 1 individual
taking into account the principal residence and 1 other residence.
(iii) Residence not rented.--For purposes of clause (i)(II), notwithstanding
section 280A(d)(1), if the taxpayer does not rent a dwelling unit at any time
during a taxable year, such unit may be treated as a residence for such taxable
year.

Revenue Ruling 69-188


-Taxpayer wants to purchase a building, and takes out a mortgage with a lender. In
addition to the annual interest, the taxpayer was to pay 70x dollars as a loan processing
fee prior to the receipt of the loan proceeds. Taxpayer wants to deduct the 70x as interest
paid.
HELD: This is considered interest and is deductible. Doesnt matter what you name it as
long as it is an amount paid as compensation for the use or forbearance of money; a
negotiated bonus or premium paid by a borrower to a lender in order to obtain a loan.
Qualifications for a payment to be considered as an "interest" - Interest is an amount paid
as compensation for the use or forbearance of money; a negotiated bonus or premium
paid by a borrower to a lender in order to obtain a loan; The payment of interest must be
incidental to an unconditional and legally enforceable obligation of the taxpayer claiming
the deduction; There must be a "separate check" to pay the interest, so not from the

loaned money; The payment must not be for specific services that the lender performs in
connection with the borrower's account; It is sufficient that the payment is a "prerequisite
to obtaining borrowed capital.

J. Simpson Dean v. Commissioner


- Petitioners, J. Simpson Dean and Paulina Dean, filed joint tax returns as husband and
wife from 1955 and 1956. Petitioners owned the stock of the Nemours Corporation. The
Petitioners gifted 2000 shares of Nemours stock to a trust for their children. Petitioners
owed large amounts to Nemours on non-interest bearing loans. The Commissioner of
Internal Revenue sought deficiencies alleging Petitioner received income from the
interest free loans.
Issue: Whether Petitioners realized taxable income from the alleged economic benefit
from the interest free use of funds, which they borrowed from the family corporation,
which they controlled?
HELD: Interest-free loans cant be considered taxable income. The Tax Court points out
that had the interest been charged, it would have been fully deductible by Petitioners.
Further, the Tax Court notes that an interest free loan is not the same as use of property
rent free, which would be taxable income.
Not on the exam**Highlights of 7872: Interest Free Loans, ***Not on the exam.
Gifts: Generally, an interest-free loan is treated as if the borrower paid the lender interest
at the AFR, triggering taxable interest income to the lender and a potentially deductible
interest expense to the borrower, and the lender made a tax-free gift of the foregone
interest to the borrower, triggering neither income nor deductions.
Compensation Loans: Generally, an interest-free loan is treated as if the employer paid
the employee compensation in the amount of the foregone interest, triggering a business
expense deduction, and the employee paid the employer interest at the AFR, triggering
taxable interest income to the lender and a potentially deductible interest expense to the
borrower.
Shareholder Loans: Generally, an interest-free loan is treated as if the company paid the
shareholder a dividend in the amount of the foregone interest, triggering inclusion in the
shareholders income and no deduction to the company, and the shareholder paid the
company interest at the AFR, triggering taxable interest income to the company and a
potentially deductible interest expense to the shareholder.
Valuation and calculation of the foregone interest varies depending on whether the loan is
a gift loan or a non-gift loan and a demand loan or a term loan.

For example, for gift term loans, the value of the constructive interest is the amount
loaned less the present value of the principal (and all actual interest payments to be made,
if any).
For gift demand loans, the value of the constructive interest is the difference between the
interest paid, if any, and the amount that would have been paid under the AFR that year.
7872 does not apply to gift loans or compensation-related loans that are cumulatively
under $10,000. It also does limits constructive interest to net investment income for
loans cumulatively under $100,000.

Revenue Ruling 2010-25


-Taxpayer purchased a 1.5 million dollar hose, paid 300k down and financed the
remaining 1.2 million. Taxpayer paid interest on the indebtedness that year.
-163 Qualified residences. 1 million dollar limit.
HELD: Taxpayer can deduct interest paid on 1million of the 1.2 million indebtedness
used to acquire the principal residence.

Soppy v. Commissioner
- The Tax Court has held that unmarried co-owners of two personal residences were not
each allowed to deduct interest on $1.1 million of acquisition and home-equity
indebtedness because the debt limitations are residence based, rather than taxpayer based.
CLASS NOTES: Per-residence basis, not per taxpayer basis.

Qualified Education Loans


Among the limited types of interest expenses that qualify is interest on qualified higher
education expenses of a student, spouse, or dependent.
The deduction is limited to $2500 and phases out beginning between $50,000 and
$65,000 AGI ($100,000/$130,000 MFJ).
Problems on pg. 465
3. Taxpayers purchase a home in the current year, which they use as their principal
residence. Unless otherwise stated, they obtain a loan secured by the residence and use
the proceeds to acquire the residence. What portion of the interest paid on such loan may
Taxpayers deduct in the following situations?

(a) The purchase price and fair market value of the home is $350,000. Taxpayers obtain a
mortgage for $250,000 of the purchase price. They can deduct all of it. It can go up to
1million.
(b) The facts are the same as in (a), above, except that in 2 years Taxpayers have reduced
the outstanding principal balance of the mortgage to $200,000 and the fair market value
of the residence has increased to $400,000. In the later year, Taxpayers take out a 2nd
mortgage for $100,000 secured by their residence to add a 4th bedroom and a den to the
residence. Yes, either under home equity or acquisition.
(c) The facts are the same as (b), above, except that Taxpayers use the proceeds of the
$100,000 mortgage to buy a Ferrari. Yes, under the home equity indebtedness. You can
buy whatever you want.

4. Single taxpayer, T, who graduated from law school, pays $3,000 of interest in the
current year on qualified educational loans.
(a) If T has $40,000 of modified adjusted gross income in the current year, what amount
of interest can T deduct? -2,500.
(b) Same as (a), above, except T has $56,000 of modified adjusted gross income in the
current year. Phase out. Exceeded by 6k over 15k, which is 2/5. So you lose 2/5 of it but
you can keep 3/5 of it.
(c) Same as (a), above, except T is married and T and Spouse file a joint return and have
$140,000 of modified adjusted gross income in the current year. None. No deduction.
Exceeds the phase out.
(d) Same as (c), above, except that T and Spouse delay paying the $3,000 of interest
(along with a $300 penalty) from the current year to succeeding year when their modified
adjusted gross income is $80,000 (because Spouse ceases working) and when no other
interest payments are made. Only 2,500 is deductible. Can choose to defer, and have a
deduction the year you qualify. *Check.
(e) Same as (a), above, except that F, Ts father makes the $3,000 payment. It is
deductible if there is a parent paying on the behalf of them.

C. TAXES
- To what extent are payments of taxes deductible?

164. Taxes (a) General rule.--Except as otherwise provided in this section, the
following taxes shall be allowed as a deduction for the taxable year within which paid or
accrued:

(1) State and local, and foreign, real property taxes.


(2) State and local personal property taxes.
(3) State and local, and foreign, income, war profits, and excess profits taxes.
(4) The GST tax imposed on income distributions.
(5) The environmental tax imposed by section 59A.
(6) Qualified motor vehicle taxes.
In addition, there shall be allowed as a deduction State and local, and foreign, taxes not
described in the preceding sentence which are paid or accrued within the taxable year in
carrying on a trade or business or an activity described in section 212 (relating to
expenses for production of income). Notwithstanding the preceding sentence, any tax
(not described in the first sentence of this subsection) which is paid or accrued by the
taxpayer in connection with an acquisition or disposition of property shall be treated as
part of the cost of the acquired property or, in the case of a disposition, as a reduction in
the amount realized on the disposition
(b)(5) General sales taxes.--For purposes of subsection (a)-(A) Election to deduct State and local sales taxes in lieu of State and local
income taxes.--At the election of the taxpayer for the taxable year, subsection (a)
shall be applied-(i) without regard to the reference to State and local income taxes, and
(ii) as if State and local general sales taxes were referred to in a paragraph
thereof.

Cramer v. Commissioner

Cramer sold her house to Osborn under a "land sale contract."

That's like an installment plan, where Osborn was to pay Cramer monthly
payments and also pay the property taxes.

Osborn was a deadbeat who didn't pay. Eventually, Cramer paid the property taxes
herself and foreclosed the property.

Later that same year, Cramer sold the house to someone else for no gain.

At the same time, Cramer's mother was sick, so Cramer was looking after her
house and paying the property taxes there as well.

Also at the same time, Cramer had bought herself a new house, and was paying
the property taxes there as well.

When Cramer filed her taxes, she deducted the amounts paid in property taxes on
all three residences.

The IRS denied the deduction. Cramer appealed.

The IRS argued that only the property taxes paid on Cramer's new house
were deductible.

The Tax Court found mostly for the IRS.

The Tax Court found that the property tax Cramer paid for her Mom's
house was not deductible.

The Court looked to 26 U.S.C. 164, which allows a deduction for


real property taxes, but they are, in general, "deductible only by the
person upon whom they are imposed."

Since the house wasn't owned by Cramer, and she wasn't liable to
pay the taxes, she couldn't claim the deduction even though she
was the one who paid the taxes.

The Court found that the property tax Cramer paid for the house she sold
to Osborn was partially deductible.

The Court looked to 164(d)(1), which says that property taxes


paid on property that was bought or sold is prorated.

In this case, the Court determined that Cramer owned the property for 48 out of the 365
days of the tax year in question, so she is allowed to deduct 48/365th of the amount she
paid in property taxes on that house.
CLASS NOTES: To take the deductions you have to be legally obliged to pay the taxes.
Problems pg. 469
1. Which of the following taxes would be deductible as such under 164?
(a) A state sales tax imposed at a single rate on sellers but required to be separately stated
and paid by purchasers to sellers, applicable to retail sales of any property except food,
clothing, and medicine. It is impossible for us to know at this point in time.
(b) A state real property tax of $1,000 for which A became liable as owner of Blackacre
on January 1st but which B agreed to pay half of when he acquired Blackacre from A on
July 1st. Only deduct the $500. Half to each.
(c) A state income tax. Deductible.
(d) The federal income tax. Not deductible.
(e) A state gasoline tax imposed on customers. Not Deductible.
3. Son who is still in college owns substantial securities. Father, when paying his own
intangibles tax to state X, pays the intangibles tax due by Son.
(a) May Father deduct the tax paid? No. Father has to be legally obliged.

(b) Is it deductible by Son? Yes.

4. Dr. Medic employs Charles to work for her as receptionist. She pays Charles salary
but withholds X dollars to which she adds Y dollars all of which she pays to the federal
government under the Federal Insurance Contributions Act (for social security).
(a) Can Dr. Medic deduct amount X? Amount Y? X + Y? Yes. Cost of doing business.
(b) Is Charles entitled to a deduction for the payments? No. Cant deduct payroll taxes
from federal income taxes.
D. BAD DEBTS, CHARITABLE CONTRIBUTIONS, AND CASUALTY AND THEFT LOSSES
-Will cover more later.

CH. 17 RESTRICTIONS ON DEDUCTIONS


A. INTRODUCTION
-Even if a deduction is expressly authorized in the Code, it may elsewhere be restricted,
limited, qualified, phased out, capped, subjected to a ceiling, or otherwise not all that and
a bag of chips. There is often fine print
B. DEDUCTIONS LIMITED TO AMOUNT AT RISK
-To protect against taxpayers sheltering income with deductions in investments or
business that create losses, there are limitations on loss deductions. Notably, a taxpayer
may only claim a deduction or loss to the extent of the amount the taxpayer has at risk.
IRC 465 Deductions limited to amount at risk
(a) Limitation to amount at risk
(1) In general In the case of
(A) an individual, and
(B) a C corporation with respect to which the stock ownership requirement of
paragraph (2) of section 542(a) is met,
engaged in an activity to which this section applies, any loss from such activity for the
taxable year shall be allowed only to the extent of the aggregate amount with respect to
which the taxpayer is at risk (within the meaning of subsection (b)) for such activity at
the close of the taxable year.
(2) Deduction in succeeding year

Any loss from an activity to which this section applies not allowed under this section for
the taxable year shall be treated as a deduction allocable to such activity in the first
succeeding taxable year.
(b) Amounts considered at risk
(1) In general
For purposes of this section, a taxpayer shall be considered at risk for an activity with
respect to amounts including
(A)the amount of money and the adjusted basis of other property contributed by
the taxpayer to the activity, and
(B)amounts borrowed with respect to such activity (as determined under
paragraph (2)).
(2) Borrowed amounts
For purposes of this section, a taxpayer shall be considered at risk with respect to
amounts borrowed for use in an activity to the extent that he
(A) is personally liable for the repayment of such amounts, or
(B) has pledged property, other than property used in such activity, as security for
such borrowed amount (to the extent of the net fair market value of the taxpayers
interest in such property).
No property shall be taken into account as security if such property is directly or
indirectly financed by indebtedness, which is secured by property described in paragraph
(1).
(3) Certain borrowed amounts excluded
(A) In general
Except to the extent provided in regulations, for purposes of paragraph (1)(B),
amounts borrowed shall not be considered to be at risk with respect to an activity
if such amounts are borrowed from any person who has an interest in such activity
or from a related person to a person (other than the taxpayer) having such an
interest. **Has to actually be recourse liability, not just disguised.
(d) Definition of loss
For purposes of this section, the term loss means the excess of the deductions allowable
under this chapter for the taxable year (determined without regard to the first sentence of
subsection (a)) and allocable to an activity to which this section applies over the income
received or accrued by the taxpayer during the taxable year from such activity
(determined without regard to subsection (e)(1)(A)).
..

(e) Recapture of losses where amount at risk is less than zero


(1) In general If zero exceeds the amount for which the taxpayer is at risk in any activity
at the close of any taxable year
(A)the taxpayer shall include in his gross income for such taxable year (as income
from such activity) an amount equal to such excess, and
(B)an amount equal to the amount so included in gross income shall be treated as
a deduction allocable to such activity for the first succeeding taxable year.
(2) Limitation
The excess referred to in paragraph (1) shall not exceed
(A)the aggregate amount of the reductions required by subsection (b)(5) with
respect to the activity by reason of losses for all prior taxable years beginning
after December 31, 1978, reduced by
(B)the amounts previously included in gross income with respect to such activity
under this subsection.
NOTES: Only deduct what you have at risk. Non-recourse liability is not included in
your at risk total. (Unlike in basis, where non-recourse liability can be included in basis).
Recapture: If you have taken a loss and then convert to nonrecourse, amount at risk can
be negative. So there is a recapture where they take it back.
Problem 1 pg. 477
1. Discuss the extent to which 465 limits Taxpayers loss deductions, generates
recapture income out of a previously allowed loss deductions, or allows the use of a loss
carryover in the following situations.
a. Taxpayer purchased a farm for $50,000 cash and his personal note for $400,000
secured by a mortgage. In the first two years of operation he put in an additional $50,000
each year, by way of cash and personal loans, for feed, fertilizer and other supplies; but
things did not go well. In the first year of operations his loss was $80,000 and he had
another $80,000 loss in the second year of operations. No principal was paid on the
liability of either year. So can deduct all losses, 160k. Hes got recourse liability and
enough on the line to take the deductions.
b. The facts are the same as in (a), above, except that the farm was acquired for $50,000
cash and $400,000 of nonrecourse financing. Can take a loss for the first year. Only 70k
loss in the second year. The last 10k isnt currently eligible because he doesnt have
sufficient assets at risk, maybe next year if he puts more money in.

c. The facts are the same as in (a) except that in the third year of operations when the
farm broke even, Taxpayer converted his personal liability of $400,000 to a non-recourse
loan. Recapture applies. So has to recapture 10k of income.
d. The facts are the same as in (b) except that Taxpayer pays off $10,000 of the
nonrecourse loan in year two. Increase in amount at risk by paying off principal of the
nonrecourse loan. 160k at risk, and 160k worth of deduction. So no recapture.
e. The facts are the same as in (b) except that the farm breaks even in year three and
Taxpayer pays off $10,000 of the non-recourse loan in year three. Can re-deduct it late
because he put more at risk.

C. ACTIVITIES NOT ENGAGED IN FOR PROFIT


-Remember that certain deductions and benefits are available with respect to transactions
entered into for profit, but not for personal transactions. If a loss is incurred in the
participation of a hobby or another activity not entered into for profit, the loss or
deduction is disallowed, except to the extent the hobby produces income to offset. Cant
deduct the cost over your gain.

*Add notes:
CLASS NOTES:
IRC 183: Disallowance of hobby activities. But can still deduct things like property
taxes. You can deduct only the losses up to the extent of your gains.
-Determining whether it is an activity entered into for profit is a question of fact. Use an
objective test. Presumption it is if it has a history of being profitable.
Treas. Reg. 1.183-3 Activity not engaged in for profit defined.
Relevant Factors: 9 of them. See notes.

Engdahl v. Commissioner
-Retired orthodontist and wife decided to enter the horse-breeding business after
consulting with trainers, vets, and other people within the business. They purchased a
ranch to board the horses and worked 35 to 55 hours a week caring for the horses and
working on the ranch. They maintained books and records of their horse operation. From
1964- 1975 they had operating losses from 1964 through 1975. Eventually they ended the
business and sold the ranch.

-They deducted substantial losses for 1971, 1972, and 1973. The losses were not allowed
based because the hose breeding operation was considered not to be an activity engaged
in for profit.
Holding: It was an activity engaged in for profit. Court looks at the 9 factors listed by the
regs. They include the manner in which the taxpayer carried on the activity; the expertise
of the taxpayer or his advisers; the financial success or failures of the activity; the
financial statues of the taxpayer.

Problem 1 pg. 486


1. Goggle, who owns a successful high-tech start-up business, develops an interest in
winemaking. Goggle purchases land in neighboring wine country and buys a home and
land surrounding it and grows grapes on the land. Regrettably, Goggles grape-growing
endeavors are not as successful as the high-tech endeavors. Goggle has net losses in each
of the first 2 years of grape-growing operations. In the 3rd year, Goggle earns $55,000
from sales of grapes, but Goggle also incurs $10,000 of property taxes, $15,000 of
interest on a mortgage on the property, $20,000 of salaries, $20,000 in other operating
expenses, and $15,000 of depreciation. Discuss the tax consequences to Goggle in year 3.
If it is for profit, then the expenses are fully deductible. If not for profit, can still deduct
losses up to your income and can deduct expenses otherwise deductible in other parts
like property taxes and interest. Hobby losses are completely disallowed.

D. RESTRICTIONS ON DEDUCTIONS OF HOMES


A taxpayers ability to take deductions for the business or profit-making use of the home
is limited. In general, only portions of rental expenses will be deductible for a home also
used as a residence. Home office expenses are also regulated and limited.

*Triggered if you are renting a vacation home, and the home office limitation.
Exceptions: Allows for deductions for home office if it is used exclusively and on a
regular basis as his principal place of business. Portion of the home has to be used.
IRC 280A. Disallowance of certain expenses in connection with business use of
home, rental of vacation homes, etc.
- (a) General rule.--Except as otherwise provided in this section, in the case of a
taxpayer who is an individual or an S corporation, no deduction otherwise allowable
under this chapter shall be allowed with respect to the use of a dwelling unit which is
used by the taxpayer during the taxable year as a residence.

(b) Exception for interest, taxes, casualty losses, etc.--Subsection (a) shall not apply to
any deduction allowable to the taxpayer without regard to its connection with his trade or
business (or with his income-producing activity).
(c) Exceptions for certain business or rental use; limitation on deductions for such
use.-(1) Certain business use.--Subsection (a) shall not apply to any item to the extent
such item is allocable to a portion of the dwelling unit which is exclusively used
on a regular basis-(A) as the principal place of business for any trade or business of the
taxpayer,
(B) as a place of business which is used by patients, clients, or customers
in meeting or dealing with the taxpayer in the normal course of his trade or
business, or
(C) in the case of a separate structure which is not attached to the dwelling
unit, in connection with the taxpayer's trade or business.
In the case of an employee, the preceding sentence shall apply only if the exclusive use
referred to in the preceding sentence is for the convenience of his employer. For purposes
of subparagraph (A), the term principal place of business includes a place of business
which is used by the taxpayer for the administrative or management activities of any
trade or business of the taxpayer if there is no other fixed location of such trade or
business where the taxpayer conducts substantial administrative or management activities
of such trade or business.
(c) Exceptions for certain business or rental use; limitation on deductions for such
use.
(2) Certain storage use
(3) Rental use.--Subsection (a) shall not apply to any item which is attributable to the
rental of the dwelling unit or portion thereof (determined after the application of
subsection (e)).
(4) Use in providing day care services
(5) Limitation on deductions.--In the case of a use described in paragraph (1), (2), or
(4), and in the case of a use described in paragraph (3) where the dwelling unit is used by
the taxpayer during the taxable year as a residence, the deductions allowed under this
chapter for the taxable year by reason of being attributed to such use shall not exceed the
excess of-(A) the gross income derived from such use for the taxable year, over
(B) the sum of--

(i) the deductions allocable to such use which are allowable under this chapter for the
taxable year whether or not such unit (or portion thereof) was so used, and
(ii) the deductions allocable to the trade or business (or rental activity) in which such use
occurs (but which are not allocable to such use) for such taxable year.
Any amount not allowable as a deduction under this chapter by reason of the preceding
sentence shall be taken into account as a deduction (allocable to such use) under this
chapter for the succeeding taxable year. Any amount taken into account for any taxable
year under the preceding sentence shall be subject to the limitation of the 1st sentence of
this paragraph whether or not the dwelling unit is used as a residence during such taxable
year.

Problem 2 pg. 491


2. Taxpayer operates a consulting business out of his home. He uses an office in his
home, exclusively and on a regular basis, as the principal place of business for his
consulting business. Taxpayer has $2,000 gross income from his consulting business. He
has business deductions of $1,600 for supplies and secretary expenses. Mortgage interest
and real estate taxes allocable to his office total $400. Utilities and depreciation allocable
to his office are $200 and $150, respectively. How much of the utility expense and
depreciation is deductible?
-The $200 and $150 would have to be rolled over because it would otherwise exceed the
gross income for the year.

E. PASSIVE ACTIVITY LIMITATIONS


-As a general rule, a taxpayer may only use deductions or credits attributable to passive
participation in an activity to offset income or gains from a passive activity.
-Suspended losses may be recognized in later years (when there is income from a passive
activity to offset) or upon qualifying dispositions of the taxpayers interest.
-An activity is passive when the taxpayer does not materially participate in the trade
or business or transaction entered into for profit.
IRS Publication 925 provides helpful detail and examples.
http://www.irs.gov/publications/p925/ar02.html#en_US_2013_publink1000104565
IRC 469. Passive activity losses and credits limited
A passive activity is a trade or business in which the t/p does not materially participate
1.
Material participation is one who is involved in the operations on a
regular, continuous and substantial basis. 469(h) & 1.469-5T
a)
500 or more hours per year
b)
Does all the work of activity

c)
d)
2.
3.

>100 hours but no one else worked more than that


It is a significant participation activity and aggregate amount of
all sign. part. act is >500 hours
e)
Materially participated for 5 out of the last 10 years
Limited partnerships, and
Rental real estate activities are per se passive activities
a)
Exception - $25,000 offset for rental activities if the t/p:
(1)
Is in a rental real estate activity
(2)
actively participates
(a)
Involved in management of property in a bona
fide sense.
(b)
Limited partnership does not satisfy active
participation for Rental RE

B.
C.
D.
E.
F.

(3)
minimum of 10% ownership
b) $25,000 offset is reduced by 50% of the gross income of the t/p
exceeding $100,000.
Portfolio income is defined as interest dividends and royalties 469(e)(1)(A)
Separates active portfolio passive income
Cannot use passive losses to offset active or portfolio income (Except after
disposition)
Must carry forward to offset passive gains in subsequent years
Losses are allowed in the following order:
1.
Income from that activity for the year
2.
Over any net income or gain from all other passive activities
3.
Against active activity or portfolio activity on disposition (See 1d)
G.
If a passive activity has a loss in one year then
becomes an active activity in the next with a gain, the
passive loss can offset the active (passive) gain.

Problems pg. 502


1. Lawyer earns $200,000 of taxable income from her practice in the current year.
Discuss the extent to which the following transactions affect her taxable income in the
current year.
(a) Lawyer also has $10,000 of dividends and interest in the year. She invests as a
limited partner in a partnership that films and distributes movies. Her share of the
partnerships movie losses for the year is $50,000. The limited partnership is considered
a passive activity. So no deduction for a passive loss and the 50,000 is carried over to the
next year. *Can only deduct passive losses against passive gains.
(b) Same as (a) above except that Lawyer also has a $30,000 gain from her limited
partnership investment in a windmill power tax shelter. Can deduct 30,000 of the loss
from the movie partnership. The 30,000 of gain is offset by 30,000 of the losses from the
movie partnership. 20,000 of loss is carried forward.

(c) Same as (a) above except that in the succeeding year the movie limited partnership
makes a gain of $90,000 as a result of a successful move, Alligator Allee. -50,000 of
loss offsets the 90,000 gain, so only 40,000 in gain needs to be recognized.
(d) Same as (a) above, except that Lawyer sells her movie limited partnership interest at
the beginning of the succeeding year at a gain. Can deduct the prior losses against the
gain from the sale of the interest.

2. (a) In the current year Grocer purchases a grocery store and spends 35 hours per week
operating it to the exclusion of all other business and investment activities. Grocers loss
from the grocery store business is $50,000. How much of his loss is deductible? All of
the 50,000 is deductible because it is a passive activity. Material participation.
(b) Same as (a) above except Grocer is only irregularly involved in the operation of the
grocery store. Intermittently, Grocer makes significant management decision. Have to
decide whether he is materially participating. Book says that an active but only
intermittent role in management doesnt establish material participation. So probably
passive. Probably not enough,
(c) Same as (a) above except that Grocer who is retired purchases the grocery store and
hires a manager who has carte blanche power to make all business decisions. Passive
activity. Outsourced all you material participation.
(d) Grocer, upset with the $50,000 loss in (a) above, fires manager at the end of the year
and in the succeeding year, Grocer manages the store on a full-time basis and in that year
makes a $60,000 profit. What tax consequences to Grocer for the succeeding year?
Passive activity was converted into active activity. The 50,000 loss from the prior passive
year is carried over and offsets the 60,000 gain in the now active year. So 10,000 of
taxable income.

F. ILLEGALITY OF IMPROPRIETY
Certain expenses that might otherwise be deductible may be denied on public policy
grounds; such as that incurring the expense was illegal or improper.
IRC 162. Trade or business expenses (c) Illegal bribes, kickbacks, and other
payments.-(1) Illegal payments to government officials or employees.--No deduction shall be
allowed under subsection (a) for any payment made, directly or indirectly, to an official
or employee of any government, or of any agency or instrumentality of any government,
if the payment constitutes an illegal bribe or kickback or, if the payment is to an official
or employee of a foreign government, the payment is unlawful under the Foreign Corrupt
Practices Act of 1977

(2) Other illegal payments.--No deduction shall be allowed under subsection (a) for any
payment (other than a payment described in paragraph (1)) made, directly or indirectly, to
any person, if the payment constitutes an illegal bribe, illegal kickback, or other illegal
payment under any law of the United States, or under any law of a State (but only if such
State law is generally enforced), which subjects the payor to a criminal penalty or the loss
of license or privilege to engage in a trade or business
(3) Kickbacks, rebates, and bribes under medicare and Medicaid
(f) Fines and penalties.--No deduction shall be allowed under subsection (a) for any fine
or similar penalty

Commissioner v. Tellier
-Taxpayer was in the business of buying and selling securities. Was later charged and
found guilty for violating the 1933 Securities Act 1933 and mail fraud statute.
-Incurred about $23,000 in legal fees and claimed a deduction.
-Commissioner conceded that the payments were business expenses and were ordinary
and necessary. However, disallowed the deduction on the ground of public policy.
-HELD: Supreme Court disagreed with the Tax Court and said the deductions should be
allowed. The Court noted that criminals are still expected to pay taxes on income from
criminal activities. So it would only be fair to allow them to claim the same deductions as
would normally apply to legal activities
- Only in extremely limited circumstances has the Court upheld a disallowance, and this
did not fall under that category.
-No public policy is offended when a person hires an attorney to help him defend himself
against serious charges.

Ch. 18 Deductions for Individuals Only


A. THE CONCEPT OF ADJUSTED GROSS INCOME
- Certain deductions are permitted in arriving at a baseline level of income, called AGI.
AGI is used as a common threshold for phaseouts and eligibility determinations for other
tax benefits.
Other deductions, not permitted before arriving at AGI, may be taken (in whole or in
part) as itemized deductions, as an alternative to the standard deduction.

**ATL are most valuable deductions, and BTL are second tier.

****SHORT ANSWER WILL BE BASED ON THIS FOR FINAL****


[(Ordinary Income
Above-the-line Deductions
Personal Exemptions
The Larger of Itemized Deductions OR Standard Deduction)
X Ordinary Income Rate]
+
[Net Capital Gain X Capital Gain Rate]
Credits
= Tax Liability or Refund

IRC 62: Defines Adjusted Gross Income and lists the above the line deductions:
(1) Trade and business deductions.--The deductions allowed by this chapter (other than
by part VII of this subchapter) which are attributable to a trade or business carried on by
the taxpayer, if such trade or business does not consist of the performance of services by
the taxpayer as an employee.
*If your employed they are not ATL deductions, they are BTL deductions.
Ex.) Bar dues for solo practitioner are ATL deductions.
(2) Certain trade and business deductions of employees [excluding unreimbursed
employee business expenses]
(3) Losses from sale or exchange of property.--The deductions allowed by part VI (sec.
161 and following) as losses from the sale or exchange of property.
(4) Deductions attributable to rents and royalties.--The deductions allowed by part VI
(sec. 161 and following), by section 212 (relating to expenses for production of income),
and by section 611 (relating to depletion) which are attributable to property held for the
production of rents or royalties.
(5) Certain deductions of life tenants and income beneficiaries of property
(6) Pension, profit-sharing, and annuity plans of self-employed individuals.--In the
case of an individual who is an employee within the meaning of section 401(c)(1), the
deduction allowed by section 404.

(7) Retirement savings.--The deduction allowed by section 219 (relating to deduction of


certain retirement savings).
[(8) Repealed]
(9) Penalties forfeited because of premature withdrawal of funds from time savings
accounts or deposits
(10) Alimony.--The deduction allowed by section 215.
(11) Reforestation expenses.--The deduction allowed by section 194.
(12) Certain required repayments of supplemental unemployment compensation
benefits.
(13) Jury duty pay remitted to employer.y.
(14) Deduction for clean-fuel vehicles and certain refueling property.--The deduction
allowed by section 179A.
(15) Moving expenses.--The deduction allowed by section 217.
(16) Archer MSAs.--The deduction allowed by section 220.
(17) Interest on education loans.--The deduction allowed by section 221.
(18) Higher education expenses.--The deduction allowed by section 222.
(19) Health savings accounts.--The deduction allowed by section 223.
(20) Costs involving discrimination suits, etc
(21) Attorneys fees relating to awards to whistleblowers.
Nothing in this section shall permit the same item to be deducted more than once
(2) Certain trade and business deductions of employees.-(A) Reimbursed expenses of employees.--The deductions allowed by part VI (section
161 and following) which consist of expenses paid or incurred by the taxpayer, in
connection with the performance by him of services as an employee, under a
reimbursement or other expense allowance arrangement with his employer. The fact that
the reimbursement may be provided by a third party shall not be determinative of whether
or not the preceding sentence applies.
(B) Certain expenses of performing artists.--The deductions allowed by section 162
which consist of expenses paid or incurred by a qualified performing artist in connection
with the performances by him of services in the performing arts as an employee.
(C) Certain expenses of officials.--The deductions allowed by section 162 which
consist of expenses paid or incurred with respect to services performed by an official as
an employee of a State or a political subdivision thereof in a position compensated in
whole or in part on a fee basis.
(D) Certain expenses of elementary and secondary school teachers.--In the case of
taxable years beginning during 2002, 2003, 2004, 2005, 2006, 2007, 2008, 2009, 2010,
2011, 2012, 2013, or 2014, the deductions allowed by section 162 which consist of
expenses, not in excess of $250, paid or incurred by an eligible educator in connection
with books, supplies (other than nonathletic supplies for courses of instruction in health
or physical education), computer equipment (including related software and services) and

other equipment, and supplementary materials used by the eligible educator in the
classroom.

IRC 63 Taxable Income Defined = Gross Income ATL deductions.


-Road forks to itemizers and non-itemizers.
-Lists the itemized deductions.

Problem 1 pg. 513


1. Assume the following expenses are properly deductible. Does the deduction fall under
62, or may it be claimed only as a 63 deduction. Is it ATL or itemized BTL deduction?
(a) Employee, a policeman, purchases a new uniform at his own expense. BTL
deduction.
(b) Employee salesman (not outside) pays the cost of entertaining purchasers in social
circumstances that are directly related to her trade or business and is not reimbursed by
Employer. BTL deduction. Have to be the boss to take these trade or business deductions.
(c) Same as (b), above, except that Employer reimburses Employee for the exact cost
incurred.
(1) How should Employee treat the expenses and reimbursement on her return? What
result to Employer? Compensation, because it is reimbursed so its an ATL deduction.
Results in a wash. Employer can deduct it as ATL deduction.
(2) What if the amount paid to Employee as reimbursement exceeds her actual expenses?
Amount she paid deductible, the excess is just compensation.
(3) What if actual expenses exceed her reimbursement? BTL deduction.
(d) Same as (b), above, except that Employer, an individual, rather than Employee,
entertained the purchasers. ATL deduction because they are the boss/employer and its a
business expense.
(e) Employee, at his own expense, pays $500 tuition for a refresher course in Home
Town to bring himself up to date on current business techniques relating to his
employment. Probably BTL deduction?

(f) Employee makes payments for medical expenses and charitable contributions and for
taxes on her residence and interest on a note secured by a mortgage on the residence. BTL
deductions. Traditional itemized deductions.
(g) Same as (f), above, except that the taxes and interest relate to a residence that
Employee rents to Tenant. If directly related to rental income the ATL, otherwise it
would be BTL.
(h) Employee has a loss on the sale of some stock that he held for investment. ATL
(i) Employee deducts $1,000 of interest on student loans. ATL
(j) Employer, whose business is unincorporated, pays his state income taxes. BTL. Not a
business tax, his personal income tax.
(k) Employer pays accountant $400 to prepare his federal income tax return, $150 of
which is allocable to preparing the schedule related to the income from his sole
proprietorship business. Costs specifically related to business ($400) is ATL, and the
personal tax would be BTL.
(l) Single pays ex-spouse $6,000 in alimony. Is this within the common thread of 62
deductions? ATL.
(m) Employee incurs properly deductible moving expenses. ATL.
(n) Employee is an elementary school teacher who buys $350 worth of classroom
materials in the current year. ATL, up to $250. Congress hasnt acted yet for this year
though.

B. MOVING EXPENSES
-Taxpayers are permitted, as an ATL deduction, to deduct certain moving expenses. For
new places of employment. Cant be for the search of work.
IRC 217. Moving expenses
(a) Deduction allowed.--There shall be allowed as a deduction moving expenses paid or
incurred during the taxable year in connection with the commencement of work by the
taxpayer as an employee or as a self-employed individual at a new principal place of
work.
(c) Conditions for allowance.--No deduction shall be allowed under this section unless-(1) the taxpayer's new principal place of work--

(A) is at least 50 miles farther from his former residence than was his
former principal place of work, or
(B) if he had no former principal place of work, is at least 50 miles from
his former residence, and
(2) either-(A) during the 12-month period immediately following his arrival in the
general location of his new principal place of work, the taxpayer is a fulltime employee, in such general location, during at least 39 weeks, or
(B) during the 24-month period immediately following his arrival in the
general location of his new principal place of work, the taxpayer is a fulltime employee or performs services as a self-employed individual on a
full-time basis, in such general location, during at least 78 weeks, of which
not less than 39 weeks are during the 12-month period referred to in
subparagraph (A).
For purposes of paragraph (1), the distance between two points shall be the shortest of
the more commonly traveled routes between such two points.

Problems pg. 517


1. Lawyer has been practicing law in Town X and he and his family live in Suburb of
Town X ten miles away. He decides to open an office in Town Y. Consequently he
moves himself and his family to a home in Town Y.
(a) How far away from Suburb must Town Y be located in order for Lawyer to be
allowed a moving expense deduction? 50 miles.
(b) How far away from Suburb must Town Y be located in order for Lawyer to be
allowed a moving expense deduction if Layer has just graduated from law school in
Town X and he was not employed? 50 miles from former residence.
(c) Assuming Lawyer is a sole practitioner what time requirements are imposed on him
or order for 217 to apply? Looks at part B of 217.
(d) What difference in result in (c) above if Lawyer joins a firm in Town Y as a partner?
Still self employed, so no difference.
(e) What difference in result in (c) above if Lawyer goes to work for a firm in Town Y
but as an associate rather than a partner? Employee, so only 39 weeks?
(f) Assuming the necessary time and distance requirements are met, and that a joint
return is filed, what is the amount of Lawyers 217 deduction if he incurs the following
expenses: $400 in moving his familys belongings, $150 in transporting his family; and

$100 in lodging and $200 in meals in conjunction with transporting the family?
Everything but the meals so $650.
(g) Is there any difference in the result in (f) above if Lawyers spouse also takes a job in
Town Y and meets the necessary time and distance requirements? No doesnt make any
difference at all as long as either spouse qualifies.
(h) If Lawyers firm reimburses Lawyer for $850 of his expenses in the year they are
incurred, what tax consequence will the reimbursement have? $650 is deductible, but
$200 doesnt qualify.

C. EXTRAORDINARY MEDICAL EXPENSES


- Taxpayers may deduct, as an itemized deduction, qualifying medical expenses that
exceed 10% of the taxpayers AGI (7.5% for taxpayers over age 65 through 2016).
-Only matters if you have a really low AGI or extremely high medical costs.
-So can only deduct the expenses that EXCEED 10% of the taxpayers AGI.
IRC 213: Medical, dental Expenses
(a) Allowance of deduction.--There shall be allowed as a deduction the expenses paid
during the taxable year, not compensated for by insurance or otherwise, for medical care
of the taxpayer, his spouse, or a dependent (as defined in section 152, determined without
regard to subsections (b)(1), (b)(2), and (d)(1)(B) thereof), to the extent that such
expenses exceed 10 percent of adjusted gross income.
(b) Limitation with respect to medicine and drugs.--An amount paid during the
taxable year for medicine or a drug shall be taken into account under subsection (a) only
if such medicine or drug is a prescribed drug or is insulin.
(d) Definitions.--For purposes of this section-(1) The term medical care means amounts paid-(A) For the diagnosis, cure, mitigation, treatment, or prevention of disease, or for
the purpose of affecting any structure or function of the body,
(B) For transportation primarily for and essential to medical care referred to in
subparagraph (A),
(C) For qualified long-term care services (as defined in section 7702B(c)), or
(D) For insurance (including amounts paid as premiums under part B of title
XVIII of the Social Security Act, relating to supplementary medical insurance for
the aged) covering medical care referred to in subparagraphs (A) and (B) or for
any qualified long-term care insurance contract (as defined in section 7702B(b)).
Raymond Gerard v. Commissioner

-Gerard's daughter had cystic fibrosis, which meant it was dangerous for her to
be exposed to dry, dusty air. On the recommendation of her doctor, Gerard
installed an air-conditioning system costing $1300 to make his daughter feel
better.
When he filed his taxes, Gerard deducted the $1300. The IRS denied the
deduction. Gerard appealed. Gerard argued that the $1300 was deductible under
26 U.S.C. 213, which allows for deductions for expenses related to medical care.
The IRS argued that this wasn't a medical expense, it was an improvement to
Gerard's house, and that's a non-deductible capital expenditure.
HELD: The Tax Court split the difference. The Tax Court found that the air
conditioner was an expenditure for medical care, and so was covered within the
scope of 213. However, the Court also found that 26 U.S.C. 263(a)(1) does not
allow deductions for any amount paid for permanent improvements or betterments
made to increase the value of any property.
In this case, the Court found that the air conditioner increased the value of
Gerard's property by $800. Therefore, Gerard was only allowed to deduct $1300$800 = $500.

Rev. Rul. 2002-19


-Weight loss programs deductible? Could deduct expenditures for the meetings and other
stuff but the meal plans couldnt be deducte.d

Problem 1 pg. 527


1. Divorced Homeowner, age 55, who received neither alimony nor other support
payments from her former husband, fully supported her 20 year old Daughter who had no
income, lived with Homeowner and was a dependent of Homeowner under 152. In the
current year, Homeowner installed a central air conditioning system at a cost of $10,000,
Dr. Watson said was an elementary requirement in caring for Daughters respiratory
problems. After installation, Homeowners home had increased in value by $4,000. Other
medical expenses paid during the year by Homeowner and Daughter consisted of
prescription medicine in the amount of $500 and doctors bills in the amount of $1,000.
Late in the year, she also paid $3,000 in premiums for health insurance but received no
reimbursements under the policy that year.
(a) If Homeowners adjusted gross income is $100,000 for the year, what will be the
amount of her medical expense deduction? Can include the daughter. $10k can be
deductible to the extent ii didnt add value to the house so $6,000. The medicine is

deductible and the health insurance. So total is $10,500, can only deduct the amount it
exceeds the 10% of her AGI so the answer is $500.
(b) Would it make better sense and, if so, be possible under the present statute to allow a
deduction of $1,200 per year for the air conditioning expenditure, assuming the system
has a 5-year life? No. Make it harder to break that 10% qualifier.
(c) If in the current year Homeowner incurs maintenance expenses of $300 on the air
conditioning system can that be taken into account as a medical expense? Would a $150
deduction for those expenses be more supportable assuming, of course, Daughter is still
there and still asthmatic? What about an estimate that $400 of the years electricity bill is
attributed to running the air conditioning system? Maintaining the equipment is still a
medical cost.

D. QUALIFIED TUITION AND RELATED EXPENSES


Taxpayers may deduct certain higher education expenses as an ATL deduction.
-Taxpayer is the student, or the child or spouse is the student.
-Costs of enrollment and attendance. Doesnt include money for books and housing.
-Cant deduct it if you elect a Section 25A tax credit with respect to the expenses. So you
have to choose between the deduction or the tax credit.
-Ceiling limitation:

The amount allowed as a deduction for a taxpayer whose AGI does not
exceed $65K ($130K in case of joint returns is $4,000.

The amount allowed for a taxpayer whose AGI is between $65K and
$80K ($130K and $160K in case of a joint return) is $2,000.

For taxpayers over $80k (or $160k in case of a joint return), ZERO.

Problems pg. 529


1. Law Student, who is single, has $30,000 of adjusted gross income prior to
consideration of Section 222, and pays $10,000 of law school tuition in the current year.
(a) What is the amount of Law Students 222 deduction? $4000?
(b) Same as (a), above, except that Law Student uses $8,000 of a 117 scholarship to pay
$8,000 of the tuition.

(c) Same as (a), above, except that Law Student is married filing a joint return and Law
Student and Spouse have $140,000 of adjusted gross income prior to consideration of
222.
E. PERSONAL AND DEPENDENCY EXEMPTIONS
-Taxpayers are generally entitled to a personal exemption for the self, the spouse, and
qualifying dependents, so long as those exemptions are not claimed by another taxpayer.
-Almost every taxpayer is allowed an exemption for himself. If another taxpayer claims
the individual as a dependent, the individual is denied a personal exemption.
-Husband and wife filing a joint tax return count as taxpayers and can get two personal
exemptions. If they dont file a joint return, a spouse may claim an allowance for the
other spouse only if the other has no gross income and is not a dependent of any other
taxpayer.
-Personal Exemption Amount is 4k.
-Also allows exemptions for dependents.
The Dependent: Qualifying Child or Qualifying Relative
152 (a) In general.--For purposes of this subtitle, the term dependent means-(1) a qualifying child, or
(2) a qualifying relative.
Phaseout: Personal exemptions are phased out for wealthier people. Lose 2% of the 4k
for each 2,500 you are over the applicable amount. Its a gradual phase-out.

Qualifying Child:
In general, to be a taxpayers qualifying child, a person must satisfy four tests:
Relationship the taxpayers child or stepchild (whether by blood or adoption), foster
child, sibling or stepsibling, or a descendant of one of these.
Residence has the same principal residence as the taxpayer for more than half the tax
year. Exceptions apply, in certain cases, for children of divorced or separated parents,
kidnapped children, temporary absences, and for children who were born or died during
the year.
Age must be under the age of 19 at the end of the tax year, or under the age of 24 if a
full-time student for at least five months of the year, or be permanently and totally
disabled at any time during the year.
Support did not provide more than one-half of his/her own support for the year.

(from http://www.irs.gov/uac/A-%E2%80%9CQualifying-Child%E2%80%9D)
Qualifying Relative
Four criteria must be met for a person to be your qualifying relative.
1.

The individual cannot be a qualifying child.

2.

The person must either

(a) live with you all year as a member of your household, or


(b) be related to you in one of the ways listed (child, stepchild, foster child, or a
descendant of any of them; brother, sister, half brother, half sister, stepbrother, or
stepsister; father, mother, grandparent, or other direct ancestor, but not foster parent;
stepfather or stepmother; son or daughter of your brother or sister; son or daughter of
your half brother or half sister; brother or sister of your father or mother; son-in-law,
daughter-in-law, father-in-law, mother-in-law, brother-in-law, or sister-in-law).
3.

The persons gross income for the year must be less than $3,900,

and

4.
The taxpayer generally must provide more than half of a person's total support
during.

Problems pg. 533


1. In the following parts of this question, state the # of deductions for personal
exemptions available. The following facts may be assumed unless otherwise indicated: T
was married; Ts spouse, S, had no gross income during the year and was not a dependent
of any other person; and T files a separate return. Treat each part separately unless
otherwise indicated.
(a) T married S on December 31st, and Ss only income for the year was $50 of interest
on tax-exempt bonds. Claim two exemptions, one on himself and one on his spouse
because she has no taxable income.
(b) Same as (a), above, except that on December 31st S also received $100 as a wedding
present from Uncle U. Gifts dont count, so the same answer.
(c) Same as (a), above, except that the $50 was a gain from the sale of the bonds in (a),
above. T would have his own exemption but cant have one for his wife, due to the
taxable income.
(d) Under the facts of (c), above, may T claim a dependency exemption for S if a spousal
exemption is foreclosed? No because she is a spouse. Spouse cannot qualify as a
spouse.
(e) Same as (c), above, except that T and S file a joint return. Exemption for himself
and one for her because they are filing jointly.

2. In each of the following parts, determine whether T, a calendar year taxpayer, is


entitled to a dependency exemption for the particular person, X, involved.
(a) X is Ts child who was born on December 31st of the current year. Yes.
(b) X is Ts child who was born on December 31st 19 years ago and who lives at home
and works part time. T provides for Xs meals and lodging which constitute more than
of Xs support. Have to see if qualifying relative rather than qualifying child, so need
more information. Need to know how much money they have.
(c) Same as (b), above, except that instead of working part time, X is a full-time student
at Embraceable U which is located in another state of where T lives. Probably, just need
to know if they share a household, but temporary absences from home dont count.
(d) Same as (c), above, except that X married Y during the year with whom she files a
joint return. No longer a dependent on someone elses return if you claim yourself on
your own return.
(e) Same as (c), above, except that X also receives $5,000 as a scholarship which X uses
for Xs support and which constitutes more than 1/2 of Xs total support. Still should
qualify as giving support.
(f) Same as (e), above, except that X is Ts brother not Ts child. Not a child of the
taxpayer, so you do take into account the scholarship. So no.

F. THE STANDARD DEDUCTION


-In determining taxable income, taxpayers may deduct the greater of the standard
deduction and itemized deductions (although in limited circumstances, an election is not
permitted).
-Itemized Deductions: Section 67 imposes a 2% floor under many itemized deductions.
For most itemized deductions can only take it to the point that it exceeds 2% of your AGI.
So most wont do you any good unless you have enough deductions.
It lists the one you can take without worrying about the 2% floor:
(1) the deduction under section 163 (relating to interest),
(2) the deduction under section 164 (relating to taxes),
(3) the deduction under section 165(a) for casualty or theft losses described in paragraph
(2) or (3) of section 165(c) or for losses described in section 165(d),
(4) the deductions under section 170 (relating to charitable, etc., contributions and gifts)
and section 642(c) (relating to deduction for amounts paid or permanently set aside for a
charitable purpose),
(5) the deduction under section 213 (relating to medical, dental, etc., expenses),

(6) any deduction allowable for impairment-related work expenses,


(7) the deduction under section 691(c) (relating to deduction for estate tax in case of
income in respect of the decedent),
(8) any deduction allowable in connection with personal property used in a short sale,
(9) the deduction under section 1341 (relating to computation of tax where taxpayer
restores substantial amount held under claim of right),
(10) the deduction under section 72(b)(3) (relating to deduction where annuity payments
cease before investment recovered),
(11) the deduction under section 171 (relating to deduction for amortizable bond
premium), and
(12) the deduction under section 216 (relating to deductions in connection with
cooperative housing
Section 68: Overall limitation on itemized deductions
Commissioner v. Banks
-The issue presented to the United States Supreme Court was whether the portion of a
money judgment or settlement paid to a taxpayer's attorney under a contingent-fee
agreement is income to the taxpayer for federal income tax purposes.
HELD: Should be included in the taxes of the Plaintiff.
Problem pg. 543
1. Single Taxpayer in the current year has $20,000 of adjusted gross income, a single
personal exemption, and the following allowable itemized deductions: $1,000 in interest,
$500 in state income taxes, $1,500 in unreimbursed employee travel expenses, $200 in
tax preparation fees, and $300 bar association dues. For simplicity, assume that there are
no inflation adjustments after 1988.
(a) What is Taxpayers taxable income for the current year? What is the AGI- Itemized
deductions personal exemptions. AGI is 20k. Itemized deductions: 1000 in interest and
500 in income taxes (not subject to 2%). The others are subject to the 2% floor,2% times
20,000=400. 2,000-400= 1,600. 2,000 for personal exemption.
2,000+1,500+1,600=5,100. 20,000-5,100= Taxable income is 14,900.
(b) What difference in result for (a), above, if Taxpayers 65th birthday is January 1st of
the succeeding year. Additional standard deduction if your are older than 65, blind, or
something else.
(c) What difference in result under the facts of (a), above, if Taxpayer is a married couple
filing a joint return? They would have two personal exemptions, which would be $4000.
Also a higher standard deductions for married couples and makes the standard deduction
higher than the itemized deductions.

(d) What difference in results under the facts of (c), above, if Taxpayers deductible
interest is $4,000 rather than $1,000? It would make the itemized deduction higher

CH. 23B: THE CHARITABLE DEDUCTION


-In general, an individual taxpayer may take qualified donations to charity within the tax
year as an itemized deduction.
-Most contributions are capped at 50% of a taxpayers AGI, although there are carryforward provisions.
-Certain other contributions, such as those to private foundations or those consisting of
certain types of property or with built-in gain, may instead face 30% or 20% limitations.
IRC 170. Charitable, etc., contributions and gifts
(a) Allowance of deduction.-(1) General rule.--There shall be allowed as a deduction any charitable contribution (as
defined in subsection (c) payment of which is made within the taxable year. A charitable
contribution shall be allowable as a deduction only if verified under regulations
prescribed by the Secretary.
[Okay, so this code section is 27 pages, and there are dozens of regulations on point too.
We are just going to hit some of the key points here, and know that there is more detail
than we have covered (for instance, section (n) relating to expenses paid by certain
whaling captains in support of Native Alaskan subsistence whaling]
Percentage Limitations: Most types are subject to the 50% rule.
*Cash transfer is fully deductible; property transfer is deductible at its FMV.
*Has to be a qualified organization or charity. 170c lists these:
170 (c) Charitable contribution defined.--For purposes of this section, the term
charitable contribution means a contribution or gift to or for the use of-(1) A State, a possession of the United States, or any political subdivision of any of the
foregoing, or the United States or the District of Columbia, but only if the contribution or
gift is made for exclusively public purposes.
(2) A corporation, trust, or community chest, fund, or foundation--

(A) created or organized in the United States or in any possession thereof, or


under the law of the United States, any State, the District of Columbia, or any
possession of the United States;
(B) organized and operated exclusively for religious, charitable, scientific,
literary, or educational purposes, or to foster national or international amateur
sports competition (but only if no part of its activities involve the provision of
athletic facilities or equipment), or for the prevention of cruelty to children or
animals;
(C) no part of the net earnings of which inures to the benefit of any private
shareholder or individual; and
(D) which is not disqualified for tax exemption under section 501(c)(3) by reason
of attempting to influence legislation, and which does not participate in, or
intervene in (including the publishing or distributing of statements), any political
campaign on behalf of (or in opposition to) any candidate for public office
(3) A post or organization of war veterans, or an auxiliary unit or society of, or trust or
foundation for, any such post or organization.
(4) In the case of a contribution or gift by an individual, a domestic fraternal society,
order, or association, operating under the lodge system, but only if such contribution or
gift is to be used exclusively for religious, charitable, scientific, literary, or educational
purposes, or for the prevention of cruelty to children or animals.
(5) A cemetery company
For purposes of this section, the term charitable contribution also means an amount
treated under subsection (g) as paid for the use of an organization described in paragraph
(2), (3), or (4).
170(d): Carryover provision. If you exceed your AGI limitations you can carry it over up
to five years.
Rev. Ruling 83-104
-Tuition payments to school not deductible. There can be no expectation of a benefit.
Cant disguise paying for something as a charitable contribution.
Rev. Ruling 67-346
-Payment to a charity has to be gift. Has to payment without consideration. Ticket prices/
Take the FMV of the ticket then subtract it from the amount that would be considered a
charitable contribution.
Charitable Contributions
- Generally, property is deducted at its fair market value, although sometimes the
deduction is limited to the taxpayers basis or the price at which the charity sells the item.

The value of services cannot be deducted, although some costs incurred in connection
with volunteering may be (mileage etc).
A transfer for partial consideration may result in a part-gift, part-sale transaction.
Quid pro quo received by the donor must be netted out, except for certain de minimus
benefits.
Substantiation of donations is required. Have to provide evidence of donation made.
Partial Interests in Property
-Only certain partial interests in property qualify for the charitable deduction.

Problem pg. 755


1. Ts contribution base for the year of the following gifts is $150,000. During the year T
makes contributions to Suntan U., an organization within 170(b)(1)(A)(2) and (c)(2), or
to Private Foundation, which is within 170(c)(2) but not within 170(b)(1)(A)(7). In each
of the following circumstances assume all verification requirements are met and
determine Ts 170 deduction for the current year, and what effect, if any, will have:
(a) T gives $100,000 cash to Suntan U. -$ Since Ts contribution base is $150,000, and
this is what could classified as a public charity under 170(b)(1)(A), T can deduct up to
50% of his contribution base. Here he could deduction up to $75,000 ($150K * 50%).
The remaining $25,000 of his contribution will roll over to the next year and can
continue to roll over for 5 years.
(b) T gives $100,000 cash to Private Foundation. Since Ts contention base is
$150,000, and this is a privation foundation that would be labeled under 170(b)(1)(B), T
can deduct up to 30% of his contribution base. Here he could deduction $45,000 ($150K
* 50%). The remaining $55k will roll over to the next year and continue to roll over up to
5 years.
(c) T gives $60,000 cash to Suntan U, and $40,000 to Private Foundation.
Its important to note that you cant exceed 50% of the contribution base in any year. So
our limit is going to be $75,000 (150k * 50%). First we want to look at the private
contribution. It will still be limited to $45,000 (150k * 30%), so since he has contributed
$40,000 that will all be deductible. His public contribution is limited to $75k (150K *
50%) however keep in mind that we have already deducted $40k, so since we can exceed
50% in any year (i.e. $75k), he will only be able to deduct $35K of his public
contribution. The remaining $25K of his public contribution can roll over to the next year
for up to 5 years.

(d) T gives $20,000 to Suntan U, and $80,000 to Private Foundation.


Again keep in mind that he cant exceed $75K in any year. His private contribution is
still limited to $45k so that will be all the he can deduct for the private contribution, the
rest will roll over. He can also include the $20K to public entity because he would still be
under his $75K cap. So his total amount of charity deduction would be $65K.
(e) T has a freshman daughter who attends Suntan U, and T pays $3,000 tuition for her
and makes a $10,000 Sponsors Club contribution. Children of members of the Sponsors
Club are automatically admitted to Suntan U.
The $3k is specifically considered tuition and the $10K will be considered tuition as well,
to the extent that it is determined to be disguised tuition. Any remaining amount could be
deductible.

CHAPTER 21 CAPITAL GAINS AND LOSSES


A. INTRODUCTION
-Certain transactions involving the transfer of property may trigger capital gain or loss
rather than ordinary income gain or loss. These rules apply to capital assets.
-Capital gain rates are generally lower and intended to promote investment.
-The holding period of the asset is usually necessary to determine whether the
investments have been held long enough to take advantage of preferential rate treatment.

A netting process (more on this later) is required to determine the balance of long-term
and short-term capital gain and loss.
IRC 1. Tax imposed
(c) Unmarried individuals (other than surviving spouses and heads of households).-There is hereby imposed on the taxable income of every individual (other than a
surviving spouse as defined in section 2(a) or the head of a household as defined in
section 2(b)) who is not a married individual (as defined in section 7703) a tax
determined in accordance with the following table:
****LOOK UP RATES

Common Capital Gain Rates


0% -- If taxpayers ordinary income tax rate is less than 25% (15%)
15%

-- If taxpayers ordinary income tax rate is 25% - 39.6% (28%, 31%, or 36%)

20%

-- If taxpayers ordinary income tax rate is 39.6%

There are a few other exceptions where capital gains may be taxed at rates greater than
20%:
The portion of any unrecaptured section 1250 gain from selling section 1250 real
property is taxed at a maximum 25% rate
The taxable part of a gain from selling section 1202 qualified small business stock is
taxed at a maximum 28% rate.
Net capital gains from selling collectibles (like coins or art) are taxed at a maximum 28%
rate.

B.

The Mechanics of Capital Gains

To calculate capital gain or loss:


First, net STCG against STCL.
Next, net LTCG against LTCL.
Then, net net-LT against net-ST to determine whether you have a gain or loss for the year.
If you have STCG, it is taxed at ordinary income tax rates.

If you have LTCG, it is taxed at (lower) capital gain rates


IRC 1222. Other terms relating to capital gains and losses
(1) Short-term capital gain.--The term short-term capital gain means gain from the sale or
exchange of a capital asset held for not more than 1 year, if and to the extent such gain is taken
into account in computing gross income.
(2) Short-term capital loss.--The term short-term capital loss means loss from the sale or
exchange of a capital asset held for not more than 1 year, if and to the extent that such loss is
taken into account in computing taxable income.
(3) Long-term capital gain.--The term long-term capital gain means gain from the sale or
exchange of a capital asset held for more than 1 year, if and to the extent such gain is taken into
account in computing gross income.
(4) Long-term capital loss.--The term long-term capital loss means loss from the sale or
exchange of a capital asset held for more than 1 year, if and to the extent that such loss is taken
into account in computing taxable income.
(5) Net short-term capital gain.--The term net short-term capital gain means the excess of
short-term capital gains for the taxable year over the short-term capital losses for such year.
(6) Net short-term capital loss.--The term net short-term capital loss means the excess of
short-term capital losses for the taxable year over the short-term capital gains for such year.
(7) Net long-term capital gain.--The term net long-term capital gain means the excess of longterm capital gains for the taxable year over the long-term capital losses for such year.
(8) Net long-term capital loss.--The term net long-term capital loss means the excess of longterm capital losses for the taxable year over the long-term capital gains for such year.
(9) Capital gain net income.--The term capital gain net income means the excess of the gains
from sales or exchanges of capital assets over the losses from such sales or exchanges.
(10) Net capital loss.--The term net capital loss means the excess of the losses from sales or
exchanges of capital assets over the sum allowed under section 1211. In the case of a corporation,
for the purpose of determining losses under this paragraph, amounts which are short-term capital
losses under section 1212(a)(1) shall be excluded.
(11) Net capital gain.--The term net capital gain means the excess of the net long-term capital
gain for the taxable year over the net short-term capital loss for such year.
For purposes of this subtitle, in the case of futures transactions in any commodity subject to the
rules of a board of trade or commodity exchange, the length of the holding period taken into
account under this section or under any other section amended by section 1402 of the Tax Reform
Act of 1976 shall be determined without regard to the amendments made by subsections (a) and
(b) of such section 1402.

Problem pg. 647

1. T, a single taxpayer, has a salary of $50,000 in the current year. T also has the
following transactions all involving the sale of capital assets: (1) a gain of $15,000 on a
collectible held for 2 years; and (2) a gain of $20,000 on stock held for 1 year and 3
months.
(a) Determine the amount of Ts net capital gain. Overall capital gain is $35,000.
(b) At what rate will the components of Ts net capital gain be taxed? Collectible is
28% and stock is 20%
(c) Assuming there is a flat 30% tax on ordinary income and disregarding any deductions
(including the standard deduction and personal exemption), what is Ts tax liability in the
current year? *Check.

2. S, a single taxpayer, has a salary of $500,000 in the current year. S also has the
following transactions involving the sale of capital assets: (1) a gain of $120,000 on stock
held for 1 year and 3 months and (2) a loss of $20,000 on stock held for 3 years. Assume
there is a flat 30% tax on ordinary income and disregarding any deductions (including the
standard deduction and personal exemption), what is Ss tax liability under Section 1 in
the current year?
His salary of $500K will subject to the 30% tax, so that would be $150,000.
His capital gains would be $100K and they would be subject to 20% capital against rate,
so that would be $20,000.
Which would bring his total tax liability to $170,000.
C. CAPITAL LOSSES
-Capital losses may be used to offset capital gains. To the extent capital losses exceed
capital gains, $3000 may be deducted against ordinary income, and the balance may be
carried forward into future years until gain arises to offset the loss (or until death).
IRC 1211. Limitation on capital losses
(b) Other taxpayers.--In the case of a taxpayer other than a corporation, losses from
sales or exchanges of capital assets shall be allowed only to the extent of the gains from
such sales or exchanges, plus (if such losses exceed such gains) the lower of-(1) $3,000 ($1,500 in the case of a married individual filing a separate return), or
(2) the excess of such losses over such gains.
Problem pg. 651
1.
Here are two questions on capital losses incurred in the current year. The figure
for taxable income given in column A reflects a single taxpayers taxable income for each
of two years without regard to his capital gains and losses. Note that in computing gross

income (as adjusted) on the return no gains will be included, since capital losses exceed
capital gains and the 1211(b) excess amount will be a reduction.
Taxable Income
1. $10,000
2. $10,000

LTCG
$2,000
$2,000

LTCL
$6,000
$10,000

STCG
$2,600
$2,000

STCL
$1,000
$4,000

For each year separately, without regard to computations for other years, determine the
amount of the taxpayers capital loss that is allowed as a deduction from ordinary income
under 1211 (b)(1) or (2) and the amount and character of his capital loss carryover, if any,
under 1212(b).
ANSWER:
Taxable Income
1. $10,000

LTCG
$2,000

LTCL
$6,000

$2,600

STCG
$1,000

STCL

First, net long term gains and long term losses:


2000-6000 = -$4000 ($4000 LTC loss)
Next, net short-term gains and short term losses:
2600-1000 = $1600 STC Gain
Finally, net short term and long term
$1600 STCG - $4000 LTC Loss = - $2400 ($2400 loss)
Taxpayer can deduct capital losses from ordinary income up to the SMALLER of the
actual loss and $3000.
Here, $2400<$3000, so T can deduct $2400 from his $10,000 ordinary income.
No capital loss carryover, since he used the loss in this year.

Taxable Income
2. $10,000

LTCG
$2,000

LTCL
$10,000

STCG
$2,000

First, net long term gains and long term losses:


2000-10000 = -$8000 ($8000 LTC loss)
Next, net short-term gains and short term losses:
2000-4000 = -$2000 STC Loss
Finally, net short term and long term
-$2000 STCL - $8000 LTC Loss = - $10000 ($10000 loss)

STCL
$4,000

Taxpayer can deduct capital losses from ordinary income up to the SMALLER of the
actual loss and $3000.
Here, $10000>$3000, so T can deduct $3000 from his $10,000 ordinary income.
The $7000 in unused loss is net capital loss carryover and can be used next year (at the
rate of $3000 + gain offset).

D. THE MEANING OF CAPITAL ASSET


-A capital asset is defined in the negative. It includes any asset held by the taxpayer,
except those on the excluded list. Most things are capital assets. The excluded list is
substantial, though, and notably includes:
-Inventory or property held by the taxpayer primarily for sale to customers in the ordinary
course of his trade or business.
-Trade or business property of a character, which is subject to the depreciation allowance
or real property used in trade or business;
-Supplies of a type regularly used or consumed by the taxpayer in the ordinary course of
a trade or business;
-Certain intellectual property;
-Certain derivative investments.
Mauldin v. Commissioner

-Maudlin bought 160 acres of land in New Mexico with the intention of using it
for cattle grazing. But then he decided a cattle ranch was a bad investment, so he
did nothing with the land. The land was way outside the city limits, and wasn't
very valuable for building houses.

Years later, the town had grown and the land was now inside the city limits. The
city built some roads on the land, and charged Maudlin for the service. To pay the
bill, Maudlin split up the land into lots and started selling them off to people who
wanted to build houses. As soon as Maudlin sold enough lots to cover the cost of
the roads, he stopped actively marketing the lots. But over the next 10 years,
unsolicited offers came in and he eventually sold off most of the lots, making a
good profit.

He claimed that the lots he sold were a long-time capital asset (26 U.S.C. 1221)
and paid taxes on the profits at that rate. The IRS denied Maudlin's claim and
found that the profit was ordinary income (now 26 U.S.C. 61(a)), which is taxed
at a higher rate. Maudlin appealed. Maudlin argued that his profits shouldn't be
considered ordinary income unless he was in the trade or business of selling real

estate, but he wasn't, he was only trying to get rid of this land he didn't want
anymore.
The Tax Court found that Maudlin was in the "trade or business" of selling real estate,
therefore the profits were ordinary income.
The Appellate Court found that there is no rule of thumb for when property sales
amount to a trade or business. Evidence the courts should consider includes:

The purposes for which the property was acquired

Whether acquisition was for resale or for investment

The continuity and frequency of sales.

In this case, the Court found that the evidence weighed in favor of Maudlin being
engaged in a trade or business.
Malat v. Riddell

-Malat was a participant in a joint venture that bought some land. They were
originally going to build some apartments to rent or houses to sell, but they had
some problems with financing and zoning, so they eventually gave up and just
sold off the land. When Malat filed his taxes, he claimed that the profits from the
land sale were a long-time capital asset (26 U.S.C. 1221) and paid taxes on the
profits at that rate. The IRS denied Malat's claim and found that the profit was
ordinary income (now 26 U.S.C. 61(a)), which is taxed at a higher rate. Malat
appealed.

Malat argued that the land was primarily to be used for building
apartments, so he was in the "trade or business" of renting apartments, He
was not in the trade or business of selling real estate.

The IRS argued that Malat was in the trade or business of selling property,
and so any profits he made were ordinary income.

The Trial Court found for the IRS. Malat appealed. The Appellate Court affirmed.
Malat appealed. The US Supreme Court remanded.

The US Supreme Court looked at 1221(a)(1), which said that the


definition of a capital asset does not include "property held by the
taxpayer primarily for sale to customers in the ordinary course of his trade
or business."

The Court found that the word "primarily" should be read to mean "of first
importance" or "principally. The IRS had been arguing that "primarily"
should be read to mean "substantially."

The Court sent the case back to the Trial Court using the proper definition of "primarily."

E. THE SALE OR EXCHANGE REQUIREMENT


- Gain or loss is only recognized upon the sale or exchange of a capital asset. Other
transactions involving an asset may not be sufficient to trigger the recognition of gain or
loss (it may still be pending, awaiting its later sale or disposition). For example: A gift is
not a sale or exchange. Need a sale or exchange for there to be capital gain or loss.
Kenan v. Commissioner

Bingham died and her will left Wise $5M to be paid on her 40th birthday, which
was payable in either cash or securities (aka stocks and bonds). Bingham's
trustees paid Wise partially in cash and partially in securities.

The securities had been purchased by Bingham years earlier and had
significantly appreciated in value between the time Bingham died and
when Wise received the inheritance.

The IRS ordered the trustees to pay $367k in taxes on the securities as a capital
gain.

The IRS argued that if the trustees had sold the securities and given the cash to Wise,
then the sale would have been a realized gain and thus taxable.
The trustees argued that the securities were an inheritance and should not be
taxable because they had not been sold and no gain was realized.
Basically the trustees were saying that there was no realized gain until Wise sold
the securities. They argued that this was more like an exchange of property. If
Bingham had given Wise a house in her will, the estate would not be responsible
for paying taxes on the increased value of the house.

The Tax Court found for the IRS. The trustees appealed.

The Appellate Court affirmed.

The Appellate Court looked to the relevant section of the tax code (then 26
U.S.C. 117, now 26 U.S.C. 1222) and found that capital gain is realized
whenever there is a "sale or exchange."

In this case, the Court found that there was an "exchange" when the
securities were transferred to Wise. Therefore, the gain was realized.

The Court found that since Bingham's will offered the trustees a choice of paying Wise in
cash or securities, it was not the same as if Bingham had made a bequest of property.

Hudson v. Commissioner

-Harahan won a lawsuit against Cole and was awarded $75k. However Cole
didn't pay. Eventually Harahan sold her interest in the Cole judgment to Hudson

and Taylor for $11k. Hudson and Taylor then negotiated with Cole and settled the
judgment for $21k. When they filed their taxes, Hudson and Taylor reported their
profit on the deal (~$5k each) as a long-term capital gain. The IRS denied the
claim. Hudson and Taylor appealed.

The IRS argued that it was not a long-term capital gain but instead was
ordinary income (which was taxed at a higher rate).

Hudson and Taylor argued that the judgment was a capital asset, and the
settlement was a "sale or judgment of a capital asset" which met the
definition of the relevant section of the tax code (then 26 U.S.C. 117,
now 26 U.S.C. 1222).

The Tax Court found for the IRS.

The Tax Court found that the judgment was not 'property' but just a debt.
Hudson and Taylor did not recover the money as the result of any sale or
exchange but only as a collection of settlement of the judgment.

The Court found that since the settlement of a debt is not a "sale or exchange," then the
tax code provisions related to capital gains are inapplicable and the profits are to be
treated as ordinary income.
CLAS NOTES: Distribution of the judgment funds is ordinary income.
Problem 1 pg. 666
1. Creditor purchased an individual Debtors $5,000 interest bearing note from a
third party as an investment at a cost of $4,000 in 1996 (see 1271(a)(1) and (b)(1)
(A)). Several years later, Debtor pays off the principal of the note using General
Motors stock which Debtor purchased several years ago at a cost of $2,000 which
is now worth $5,000. What are the tax consequences to Creditor and Debtor?
For the debtor, he purchased the stock for $2,000, therefore that is his basis, he
has an amount realized of $5,000 because that is the amount he owed on the note.
Therefore, he has a capital gain of $3,000 because the stock would be considered
a capital asset.
For the creditor, purchased the note obligation for $4,000 therefore that is his
basis, his amount realized is $5,000 because that is the value of the stock used to
satisfy the note obligation. Therefore he will report $1,000 to income because this
would not be considered a capital asset.

Note: 1271(a)(1), effective in 1997, provides that Amounts received by the holder on
retirement of any debt instrument shall be considered as amounts received in exchange
therefor. 1271(b)(1(A) provides that 1271 does not apply to pre-June 1997 obligations.

F. THE HOLDING PERIOD


- As a general rule, a capital asset held for a year or less results in short-term gain or loss,
and a capital asset held for over a year results in long-term gain or loss.
Sometimes Congress fiddles with the time periods.
-Incentive to hold onto your investments.
These revenue rulings tell you how to count days:
Rev. Rul. 66-7
-Start counting the day after you acquire the asset. If you acquire it on the last day of the
month, must be sold by or after the first day of the 7th month. (This was when the 6month was the holding period.)
Rev. Rul. 66-97
-How to count trading dates. Dont count the day on which you got them, do count the
day on which they are sold.
Problem 1 pg. 671
1. Taxpayer, a cash method, calendar year taxpayer, engaged in the following transactions
in shares of stock. Consider the amount and character of Ts gain or loss in each
transaction:
(a) T bought 100 shares of stock on January 15th of year 1 at a cost of $50 per share. T
sold them on January 16th of year 2 at $60 per share. Here the capital asset was held for
more than one year so it is long-term. His basis in the asset was 5,000. His A/R was
$6,000. So he has $1,000 long-term capital gain.
(b) T bought 100 shares of stock on February 28th of year 1 at a cost of $50 per share. T
sold them on February 29th of year 2, a leap year, for $60 per share. Since this is a leap
year, he would have had to held it until March 1st. Doesnt matter if the month has less
than 31 days. So here its short term captal gain of $1,000.
(c) T bought 100 shares at $50 per share on February 10th of year 1 and another 100
shares at $50 per share on March 10th of year 1. T sold 100 of the shares on February 15th
of year 2 for $60 per share. If you are able to distinguish between the stocks have to treat
them separately, so Feb 10th stocks would be long term and March 10th would be short
term. If you cant distinguish can treat them all as long-term.

(d) T told Ts broker to purchase 100 shares of stock on December 29th of year 1 at a time
when its price was $50 per share. The stock was delivered to T on January 3rd of year 2
when it was selling for $52 per share. T told Ts broker to sell the stock on December 30th

of year two when it sold for $60 per share, and it was delivered to Buyer on January 4th of
year 3 when it was selling for $63 per share. You use the trade dates. The date purchased
(Dec. 29th at $50 per share) and the date sold (Dec 30th at $60 per share). Since it was for
longer than one year, he has a long term capital gain of $1,000.

(e) Same as (d), above, except that the value of the stock on December 30th of year 2 was
$45 per share and on January 4th of year 3 was $48 per share. Still use the date they were
sold, however now there would be a long term capital loss of ($500).

(f) Ts father bought 100 shares of stock on January 10th of year 1 at $30 per share. On
March 10th of year 1 when they were worth $40 per share he gave them to T who sold
them on January 15th of year 2 for $60 per share. Gifts. 3k LTC gain. Can stack the
holding periods on top of each other.
(g) Ts father purchased 1,000 shares of stock for $10 per share several years ago. The
stock was worth $50 per share on March 1st of year 1, the date of Fathers death. The
stock was distributed to T by the ex ecutor on January 5th of year 2 and T sold it for $60
per share on January 15th of year 2. Can stack the holding periods on top. So LTC gain.
10k LTC gain.
(h) Same as (g), above, except that T was executor of Ts fathers estate and as such T
sold the stock on January 15th of year 2 for $60 per share to pay the estates
administration expenses. Still a LTC gain.

Ch. 27 Computations
A. TAX RATES
Once taxable income has been determined (by subtracting appropriate deductions and
exemptions from gross income), the taxpayer must apply the appropriate tax rate for
his/her/their filing status to determine tentative tax liability.

Classifications of Taxpayers
An individual taxpayer is classified into one of the following statuses:
1.

Married Filing Jointly/Surviving Spouse

2.

Head of Household (Unmarried but provide support for dependents)

3.

Unmarried Individual

4.

Married Filing Separately

*Same rules apply for same sex marriages.


*Although the same tax rates apply to each status, the dollar amount at which the tax
tiers increases varies. They have different schedules for different classifications.
Bonus Taxes for High-Income Taxpayers ****NOT ON EXAM***
Personal Exemption Phase-outs (151).
Itemized Deduction Reductions (68 called the Pease limitations after former Rep.
Pease).
Net Investment Income Tax (1411)

Personal Exemption Phase-outs (PEP -- 151) **NOT ON EXAM**


Apples only to taxpayers who exceed the threshold amount.
Reduces value of exemption by 2% for each $2500 by which the taxpayers AGI exceeds
the threshold amount.
In 2015, phase-outs for personal exemption amounts (sometimes called PEP) begin
with adjusted gross incomes (AGI) of $258,250 for individuals and $309,900 for married
couples filing jointly; the personal exemptions phase out completely at $380,750 for
individual taxpayers ($432,400 for married couples filing jointly.)
Itemized Deduction Reductions (68 Pease) *NOT ON EXAM*
Apples only to taxpayers who exceed the threshold amount.
Reduces value of MOST itemized deductions by the lesser of (a) 80% of their total value
and (b) 3%
Phase-outs for itemized deductions claimed on individual returns for tax year 2015 will
begin with incomes of $258,200 or more ($309,900 for married couples filing jointly).
Net Investment Income Tax (1411) **NOT ON EXAM**
Additional 3.8% tax on net investment income (interest, dividends, annuities, royalties,
rents, capital gains, passive activities, etc.).
Applies to lesser of net investment income or the amount by which taxpayers AGI
exceeds the threshold amount which means it does not apply to taxpayers below the
threshold amount.

Threshold amount is $250,000 for married filing jointly, $125,00 for married filing
separately, $200,000 for single, and $200,000 for head of household.
The Kiddie Tax
- The net unearned income of a child (under age 18 or a supported student under age 24)
that exceeds $1,000 (in 2015, $2,100) is taxed at the parents marginal tax rate.

Problems pg. 871


11. T is a calendar year taxpayer. In each of the following subparts you are to
compute Ts tax liability (before credits) assuming t has $223,050 of taxable income
under the rate schedules for the year 2013 which appear above.
(a) T is unmarried and has no special status.
T's tax liability under 1(c) is $57,737.25. As is pointed out in the Note at page 863 of
the Text, the special rates for non-surviving spouse, non-head-of-household singles were
first added under the 1969 Act.
Over $183,250 but not over $398,350: $44,603.25 + 33% of [$223,050-$183,250] = :
$44,603.25 + 33% of $39,800 = $44,603.25 + $13,134 = $57,737.25.
(b) On December 31st of the current year T married Spouse, a calendar year
taxpayer who has no income for the year, and they file a joint return.
T's tax liability under 1(a) is $49,919.50. If married individuals meet the 6013
requirements, they may file a joint return. Under 7703(a)(1) marital status is determined
at the end of the year and therefore T and Spouse may file a joint return.
Over $223,050 but not over $398,350: $49,919.50 + 33% of excess over $223,050.
(c) T was married and two minor children supported by T lived with T and Spouse,
but Spouse, who had no income in the year, died on January 15th of the current year.
T's tax liability under 1(a) is $49,919.50.
The exception clause of 7703 (a)(1) applies, and T is still treated as married in the year
of Spouse's death. Spouse's executor or administrator and T may file a joint return for the
year of Spouse's death. See 6013(a)(3).
(d) Same as (c), above, except that Spouse died on December 31st of the prior year.

T's tax liability under 1(a) is still $49,919.50.


In this situation, T is a "surviving spouse" as defined in 2(a) and T may continue to use
the 1(a) rates. The following requirements of the 2(a) definition of "surviving spouse"
are met, viz:
(1) Spouse died in one of the two years immediately preceding the taxable year; 2(a)(1)
(A).
(2) T maintains a household providing over half of the cost of maintenance. 2(a)(1) (last
sentence).
(3) T's household constitutes the principal place of abode for 151 dependents of T who
are children of T. The children are qualified dependents of T under 152(a)(1) for whom
T can claim deductions under 151(c)(1). 2(a)(1)(B)(i) and (ii).
(4) It is assumed that none of 2(a)(2) limitations is violated.
(e) Same as (c), above, except that spouse died on December 31st 3 years ago.
T's tax liability under 1(b) is $54,188.00. T is a "head of household" as defined in
2(b), because:
(1) T is not married at the close of the taxable year. 2(b)(1).
(2) T is not a surviving spouse; see 2(a). As Spouse did not die in one of the two
immediately preceding taxable years, T does not qualify as a surviving spouse. 2(a)(1)
(A).
(3) T maintains a household and provides over half of the cost of maintenance. 2(b)(1)
(last sentence).
(4) T maintains as T's home a household which constitutes the principal place of abode of
T's children for more than one-half of the year. 2(b)(1)(A)(i).
(5) T does not fall within any of the 2(b)(3) limitations.
Over $203,150 but not over $398,350: $47,621 + 33% of excess over $203,150.
So $47,621 + 33% [$223,050 - $203,150] = $47,621 + 33% [$19,900] = $47,621 +
$6,567 = $54,188
(f) Same as (c), above, except that T remarried on December 31st of the current year
and T and New Spouse file separate returns for the current year.
T's tax liability under 1(d) is $62,240.50. Even though T supports two minor children in
T's home, T does not qualify for the favorable "head of household" rate of 1(b) because
T is married at the close of the taxable year. See 2(b)(1). See also 1(d) and 7703(a)
(1) concerning the time for determining marital status. If, however, T and New Spouse
had lived apart during the entire taxable year, T could use the 1(b) rates. See 2(c)
and 7703(b).
It may also be useful to compare the amount of T's liability as an unmarried person,
computed in problem 1(a), above, with the higher amount T must pay now that T is

married but filing separately. It might also be informative to point out that, under 1(e),
an estate or trust with
$223,050 of taxable income would be liable for $86,685.60 of taxes.
Over $199,175 but not over $225,000: $53,884.25 + 35% of excess over $199,175.
So $53,884.25 + 35% [$223,050 - $199,175] = $53,884.25 + 35% [$23,875] =
$53,884.25 + $8,356.25 = $62,240.50.

2. Husband and Wife, both under 65 and with good eyesight, have 2 dependent
children. In the year 2013, whose rate schedules appear above, the file a joint return.
They have no 62 deductions. Using the rate tables and assuming the exemption
amount is $2,000 and the standard deduction for marrieds filing jointly is $6,000,
compute their tax liability before credits if:
(a) They have $100,000 of gross income, $2,000 in state property taxes, and $5,500 in
miscellaneous employee business expenses.
Husband and Wife have taxable income of $86,000. That amount of taxable income is
computed as follows:
Gross income $100,000
- 62 deductions - 0
Adjusted Gross Income $100,000
- Standard deduction - 6,000
- Personal exemptions - 8,000
Equals taxable income of $ 86,000
They would use the standard deduction of $6,000 (see 63(c)(2)(A)), as it exceeds their
total itemized deductions of $5,500, the total of the miscellaneous itemized deductions of
$5,500 which are subject to a $2,000 (2% of adjusted gross income, see 67(a)) floor
netting $3,500 plus the $2,000 of itemized deductions which are not subject to the 2%
floor (see 67(b)(2) and 164(a)(1)). The personal exemptions amount is 4 x $2,000 or
$8,000.
Husband and Wife have $13,357.50 of tax liability computed under 1(a) as follows:
$9,982.50 plus $3,375 (25% of $13,500) equals $13,357.50.
(b) They have $100,000 of gross income, $3,000 in state property taxes, and $7,000 in
miscellaneous employee business expenses.
Husband and Wife have taxable income of $84,000. That amount of taxable income is
computed as follows:

Gross income $100,000


- 62 deduction - 0
Adjusted Gross Income $100,000
- Itemized deductions - 8,000
(see below)
- Personal exemptions - 8,000
Equals taxable income of $ 84,000
They would elect to itemize their deductions. Their total itemized deductions after the 2%
floor ($2,000) would be $7,000 less $2,000 plus $3,000 or $8,000, which exceeds the
$6,000 standard deduction.
Husband and Wife would have tax liability of $12,857.50, computed under 1(a) as
follows: $9,982.50 plus $2,875 (25% of $11,500) equals $12,857.50.

B. CREDITS AGAINST TAX


Credits function as dollar-for-dollar reduction of a taxpayers tax liability. Most credits
are nonrefundable (can reduce tax liability, but not below zero), but a few are refundable
and can result in payments to the taxpayer.
1. PERSONAL CREDITS
Some of the primary credits for which taxpayers may be eligible include:
Credit for Dependent Care Services (21)
Credit for Qualified Adoption Expenses (23)
Child Tax Credit (24)
Hope and Lifetime Learning Credits (25A)
Earned Income Credit (32)
General Business Credit (38)
Dependent Care Services
Limited credit can offset costs of dependent care necessary for taxpayers employment
Allowed for employment related expenses for household services or day care of
qualifying individual (qualifying child under 13 or disabled spouse or dependent of
taxpayer)
Credit varies from 20% to 35% of costs up to a cap
Cap of costs are $3000 for one child or $6000 for two or more

Maximum credit is 35% of $3000 or $6000 ($1050 or $2100)


Adoption Expense Credit (23)
Limited credit can offset costs of adoption of child by taxpayer
Up to $10,000 in costs ($13,400 in 2015).
Flat $10,000 ($13,400 in 2015 for child with special needs.
Credit phased out for higher-income taxpayers (between $40,000 and $150,000)
($201,010-$241,010 in 2015)
Child Tax Credit (24)
Credit for each qualifying child under age 17.
Generally $1000 per child.
Partially refundable (generally up to 15% of taxpayers earned income over $3000).
Credit phased out for higher-income taxpayers ($110,000 MFJ).
Earned income Tax Credit
-*See slides.
2. CREDITS FOR PREPAID TAXES
-Taxpayers who have prepaid taxes, whether through employer withholding or quarterly
estimated payments of tax, are eligible to have overpayments refunded.
*Most common. You authorize companies to withhold when you fill out a W2.

Problem pg. 879


Taxpayer has $106,000 of tax liability in the current year prior to consideration of certain
credits. Taxpayer has two children (ages 10 and 12). None of the credits is subject to the
passive activity credit limitation. Taxpayer isnt subject to the alternative minimum tax.
Determine tax payable or refundable after the credits and whether Taxpayer is allowed
any carry backs or carryovers of credits. The credits are:
21

Child care credit

31
38

Withholding credit
Credits potentially qualifying for the
general business credit

$ 1,000
24,000
90,000

Exam Short Answer Questions


Consider the following facts relevant in determining X and Ys tax liability, step by step,
as instructed below. List the items of receipt or expense as prompted as well as the value
at which each would be included on an income tax return. Use your answers for
Questions 26-53 in responding to the questions that follow. For purposes of the questions
below, assume that X and Ys filing status is married filing jointly.

54.
What items are included in gross income (excluding capital gain property)? At
what value?

55.

What above-the-line deductions apply, if any? At what value?

56.

What is Xs Adjusted Gross Income?

57.
What are each of Xs itemized deductions and what is the total of Xs itemized
deductions? You should take into account the 2% limitation of Section 67, if any.

58.
Assume the standard deduction for married taxpayers filing jointly is $10,000.
Should X take the itemized deductions or the standard deduction? (Whichever one is
bigger)

59.
How much can X deduct for personal exemptions, assuming each exemption is
$5,000?

60.
What is Xs net taxable income? (AGI ITEMIZED/STD DEDUCTION
EXEMPTIONS)

61.
Using the table in the edited code section below, for married couples filing jointly,
what is the tax on Xs net taxable income?

If taxable income is:

The tax is:

Not over $50,000

15% of gross income

Over $50,000 but not over $100,000

$7,500 plus 28% of the excess over $50,000

Over $100,000 but not over $150,000


$100,000

$21,500 plus 31% of the excess over

Over $150,000 but not over $250,000


$150,000

$62,500 plus 36% of the excess over

Over $250,000

$98,500 plus 40% of the excess over


$250,000

62.
Considering your answers for 26-35, and assuming a capital gain tax rate of 20%,
what is the tax on Xs net capital gain (assuming the holding period is long-term)?

63.

What is Xs total tax liability, before credits?

62.
Considering your answers for 46-49, what credits should be deducted, and at what
value?

65.

Considering all of the above, what should Xs final tax bill be?

You might also like